You are on page 1of 123

Bookkeeper and accountant civil service examination instruction;

250 questions and answers including problems and solutions &


500 specimen ques. ... Board of education questions for license
to teach bookkeeping in high schools and evening schools, N.Y.
city; digest of uniform acounting systems prescribed by the Public
service commission, First district, New York; certified public accountant,
answers to New York state examination questions; practice questions
and answers on bookkeeping, commercial practice and terms. By
Mark Wolff.
Wolff, Mark.
New York, Civil service chronicle, c1917.

https://hdl.handle.net/2027/uc1.$b306362

Public Domain, Google-digitized


http://www.hathitrust.org/access_use#pd-google

We have determined this work to be in the public domain,


meaning that it is not subject to copyright. Users are
free to copy, use, and redistribute the work in part or
in whole. It is possible that current copyright holders,
heirs or the estate of the authors of individual portions
of the work, such as illustrations or photographs, assert
copyrights over these portions. Depending on the nature
of subsequent use that is made, additional rights may
need to be obtained independently of anything we can
address. The digital images and OCR of this work were
produced by Google, Inc. (indicated by a watermark
on each page in the PageTurner). Google requests that
the images and OCR not be re-hosted, redistributed
or used commercially. The images are provided for
educational, scholarly, non-commercial purposes.
H.
Bººl.
W.
|
iii.
*B B.D. L. E LE
-
INFORMATION REGARDING EXAMINATIONS:::FOR:..
IBOOKIKEEPER AND ACCOUNTAINT POSITIONS
IN THE NEW YORK CITY, NEW YORK STATE,
INEW JIERSIEY AND UNITED STATES
SERVICES.

There are excellent opportunities for ap $6,000, 1 Accountant at $4,000, 1 Bookkeeper


pointment and advancement for those quali at $4,000. Auditors of Accounts: 13 at
fied to pass examinations for Bookkeeper $3,000, 1 at $2,500. 1 Examiner of Accounts
or Accountant in the New York City, New of Institutions at $5,000. Accountants: 1 at
York State and United States Services. $2,400, 1 at $1,800. Bookkeepers: 1 at $3,150,
Examinations are held frequently, as Book 1 at $2,580, 1 at $2,400, 1 at $2,250, 1 at
keepers and Accountants are always in de $1,800, 1 at $1,680, 2 at $1,650, 4 at $1,560,
mand, especially at the present time. The 5 at $1,440, 1 at $1,350, 2 at $1,320, 13 at
Federal Government during the war has $1,200.
been unable to secure enough Bookkeepers Bureau of Accounting.—1 Chief Auditor
and Accountants to handle the great vol at $6,000. Expert Accountants: 1 at $3,420,
ume of business resulting from war con 1 at $2,280, 1 at $2,100, 1 at $1,860. Ac
tracts, which mount into billions of dollars. countants: 1 at $3,000, 2 at $2,520, 1 at
City and State governments throughout $2,400, 1 at $2,100, 1 at $1,800, 1 at $1,740.
the country are applying themselves to new Bookkeepers: 1 at $1,680, 1 at $1,620, 2 at
systems of accounting. Leader in this di $1,320.
rection is the Federal Government, which Many Bookkeepers also are appointed for
recently started a new Cost Accounting temporary periods at the rate of $1,200 per
Department at Washington. It has been an int1nn.
prophesied that this department will become There are hundreds of Bookkeepers and
a model one to be copied by municipalities Accountants in other city departments.
throughout the country.
The line of promotion in the Municipal
Service of the City of New York is excel BOOKKEEPER,
lent, affording Bookkeepers and Account
NEW YORK CITY
ants opportunity to rise to positions paying
SERVICE, GR. 2–$840 TO $1,200.
as high as $6,000 a year. The payroll of The lowest grade examination for Book
the Finance Department may be taken by
keeper in the New York City Service is
way of illustration to show the number of
Bookkeepers and Accountants employed at that for Bookkeeper, Grade 2. Statistics
on this test follow:
various salaries. In the budget for the year
Last examination held, July 10, 1917.
1918 there is provision for the following: Took test: 882 candidates. -
Bureau of Law and Adjustment.—1 Chief,
$6,000; 1 Auditor of Accounts, $5,000; 1 Au Number passing: 168 males, 106 females.
List established: September 27, 1917.
ditor at $4,000, 3 Auditors at $3,000, 1 Dep
uty Auditor at $2,500. Examiners: 1 at This examination was for positions pay
$3,500. 1 at $2,520, 1 at $2,340, 2 at $2,280,
ing from $840 to $1,200. It was open to
males and females not less than 18 years
1 at $2,100, 3 at $1,920, 3 at $1,800, 3 at
$1,620, 1 at $1,560. Examining Inspectors: of age. The examination consisted of a
paper on Duties (Bookkeeping), weight,
1 at $1,800, 3 at $1,620. 1 Bookkeeper at 6; Arithmetic, 2; Handwriting and Neatness,
$1,200.
2. Seventy per cent was required on the
Bureau of Stocks and Bonds.-Auditors
Duties paper and 70 per cent required on
of Accounts: 2 at $2,100, 1 at $1,800. Book
keepers: 2 at $1,560, 1 at $1,320.
all. No previous experience required. The
official announcement stated the duties to
Bureau of Municipal Investigation and be as follows:
Statistics.-1 Supervising Statistician and “Duties: To make simple postings of ac
Examiner at $6,000; 1 Accountant at $4,260. counting information from one book to an
Expert Accountants: 1 at $2,700, 2 at $2,400, other or from documents or schedules to
2 at $2,280, 1 at $2,100. Accountants: 1 at accounting records, to balance or adjust
$3,180, 5 at $2,940, 2 at $2,700, 2 at $2,460, accounts, to take trial balances, to make
3 at $2,400, 3 at $2,280, 3 at $2,100, 1 at journal entries, to write statements or bills,
$1,950, 4 at $1,920, 2 at $1,800. Bookkeep and to perform other bookkeeping work of
ers: 1 at $2,400, 1 at $1,560, 1 at $1,320. a similar character and standard.”
Bureau of Assessments and Arrears.
Bookkeepers: 1 at $2,160, 1 at $1,560, 1 at
$1,500, 1 at $1,440, 3 at $1,320, 1 at $1,200. BOOKKEEPER, GRADE 3, NEW YORK
Bureauof Markets and Sundry Revenues.
—Bookkeepers: 1 at $1,800, 1 at $1,320. CITY SERVICE, $1,200 TO $1,800.
Bureau of Audits, Accounts and Disburse The next higher grade examination in the
ments.-1 Chief Auditor of Accounts at New York City Service is that for Book
/
H P 5.4/
M/7
skéhº...Kirºde::3.: #prºuation regarding list. The Duties of a 5th Grade Account
* which is as follows: ant are as follows:
Last examination held, July 9, 1914.
“Duties: 1. To conduct audits, examina
Applications filed: 1,941. tions and investigations
Took test: 883. of the various city
departments and to report thereon;
Passed: 430 males, 45 females.
“2. To devise and install new and im
List established: December 2, 1914. proved methods of accounting;
This position pays from $1,200 to $1,800. “3. To devise daily, periodical and annual
The subjects and weights were: Duties report, forms, and to analyze and classify
(Bookkeeping), 5; Arithmetic, 3; Handwrit
expenditures for budget purposes;
ing and Neatness, 2. Seventy per cent was
required on the Duties paper and 70 per “4. ...To supervise the work of bookkeep
cent on all. The minimum age was 21 ing divisions in the larger city depart
ments.”
years. Salary, $1,200 to $1,800. No previ
ous experience as a Bookkeeper required.
The official requirements follow:
“Applicants must present evidence of at
least two years' experience in accounting
work of a grade equal to that outlined un
ACCOUNTANT, GRADE 4, NEW YORK der ‘Duties' either in large industrial or
governmental organizations, or in the offices
CITY SERVICE, $1,800 TO $2,400. of certified public accountants. A knowl
Going up another rung in the ladder the edge of auditing and the theory of accounts
candidate may take an open competitive or is necessary, as well as the ability to solve
promotion Accountant, problems in theoretical practical account
examination to
Grade 4, which pays from $1,800 to $2,400. ing. Candidates may also be called upon
The last open competitive examination for to answer questions relative to the elements
this position was held on July 2, 1917. There of business law and to municipal account
were 112 competitors, of which number 80 ing as practiced in the City of New York.”
passed. The list was established on August The minimum age was 25 years.
8, 1917. The subjects and weights of the
examination were: Experience, 3 (70 per
cent required); Technical, 7 (75 per cent
required). The official announcement for
this examination gave the Duties and Re EFFICIENCY ACCOUNTANT, NEW
quirements as follows: YORK CITY SERVICE, SALARY
“Duties: The duties of the position are $3,000 TO $4,000.
to conduct, under general directions, audits,
examinations and investigations of the vari On the same plane as 5th Grade Account
ous city departments; to assist in the in ant is the position of Efficiency Account
stallation of new and improved methods of ant, Grade 5, which pays the same salary.
accounting; to analyze and classify expen The position is largely that of a Cost Ac
ditures for budget purposes, and to exam countant. The last examination was held
ine and report upon the bookkeeping meth on August 10, 1915. Thirty-two candidates
ods of large city departments. took the test, and 13 attained the list, which
“Requirements: Applicants must present was established on September 1, 1915. The
evidence of at least one year's experience Duties as set forth in the official announce
in accounting work of a grade equal to that ment follow:
outlined under ‘Duties' either in large in “1. The working out of cost data for
dustrial or government organizations or in budget purposes and administrative control;
the offices of certified public accountants. “2. The determination and introduction
A knowledge of auditing and the theory of improvements in organization, methods
of accounts is necessary, as well as the abil and procedure
ity to solve problems in theory and prac based upon fundamental
principles of scientific management as ap
tical accounting.” plied in the municipal field;
The minimum age was 24 years.
“3.

The supervision cost calculations,


of
of

the tabulation units work and the


of

compilation
of

the reports
of

unit costs.”
ACCOUNTANT, GRADE 5, NEW YORK The requirements follow:
CITY SERVICE, $2,400 AND UP. “Applicants must present evidence
at
of

least two years' experience cost ac


as
a

The highest open competitive examina countant either large industrial or gov
in

tion for Accountant in the service of the ernmental organizations,


or
in

the offices
City of New York is that for Accountant,
of

certified public accountants.


Grade 5. This position pays $2,400 and over,
be

“Candidates should acquainted with


and usually $3,000, $3,250, $3,500 and $4,000. the principles and practice
of

scientific man
The last examination was held on Feb agement and should be familiar with the
ruary 2, 1916, in which 149 candidates par use mechanical sorting and tabulating
of

ticipated. The list was established on June machines.”


14, 1916. Only 19 candidates attained the The minimum age was years.
21
BOOKKEEPER, GRADES 5 AND 6, JUNIOR ACCOUNTANT, NEW YORK
NEW YORK STATE SERVICE, EN STATE SERVICE, SALARY
TRANCE SALARY $721 TO $1,200. $1,201 TO $1,500.

The demand for Bookkeepers and Ac Advancing higher a candidate may then
countants in the service of the State of take the test for Junior Accountant in the
New York has been so great that it has New York State Service. Information con
been necessary to hold examinations at fre cerning this test follows:
quent intervals. Last test held December 8, 1917. Most
The lowest grade of examination is that appointments are made to the Public Ser
for Bookkeeper, Grade 4. Details of this vice Commission, First District, but the list
examination follow: is also used for other State Departments.
Appointments are made to State Depart Salary, $1,201 to $1,500. Age limits, 20 to
ments and Institutions. Entrance salary, 30 years.
$721 per annum, or $600 and maintenance. Candidates should have a good training
Minimum age, 18 years. Subjects and in the theory and principles of economics,
weights of examination: Spelling, 1; Pen including accounting, finance and statistics.
manship, 1; Letter-writing, 1; Theory and Subjects and weights of examination:
Practice of Bookkeeping and Commer Discussion of one of several topics, such
cial Arithmetic, 4; Experience, special cred as valuation, depreciation, capitalization, re
it being given for actual experience in book turn on investment, especially as related to
keeping and general office work, 3. public service problems, 4; Questions on
Last
test held on October 6, 1917. Accounting Principles, 3; Education, Ex
perience and Personal Qualifications, 3.
Candidates were summoned also for an oral
interview with the examiners in connection
with the rating on personal qualifications.
BOOKKEEPER – STENOGRAPHER - Candidates were expected to have an edu
cation substantially equivalent to a univer
CLERK, NEW YORK STATE SER sity course of four years' duration. The
VICE, SALARY $720 TO $1,000. test was open to non-residents.

The State Civil Service Commission also


held an examination for Bookkeeper-Ste ACCOUNTANT, NEW YORK STATE
nographer-Clerk, a new title of position, on SERVICE, SALARY $1,801 TO $2,400.
November 3, 1917. Details of this test fol
low: A full-fledged Accountant may take the
Salary, $720 to $1,000. Subjects and examination for Accountant, information
weights: Stenography and Typewriting, 4; concerning which follows:
Bookkeeping, 3; Letter-writing, 1; Penman Last test held April 8, 1916. Most ap
ship, 1; Commercial Arithmetic, 1. Appli pointments made to the Public Service
cants who have qualified as Stenographer Commission, but the list is also used for
or Bookkeeper in a State Civil Service ex other State Departments. Salary, $1,801 to
amination within the past three years will $2,400. (These are entrance salaries only.
not be required to be re-examined in such Some of the present staff of the Public Ser
subjects, but may omit that part, taking vice Commission have been advanced to
the remainder of the examination. The ex about $3,500.)
amination in Stenography will comprise the The position requires knowledge of the
transcription of matter dictated at 60 and principles and methods of accounting and
90 words per minute, and an exercise in of their application to the accounts of pub
Plain Copy (Typewriting). lic service corporations; also actual experi
ence as an Accountant.
Subjects and weights: Economics, Statis
tics and Corporation Finance, 3; Principles
and Methods of Accounting, with especial
BOOKKEEPER, GRADES 5 AND 6, reference to the systems of accounts pre
scribed by the Public Service Commissions,
NEW YORK STATE SERVICE, 4; Education, Training and Experience, 3.
SALARY $721 TO $1,200. Facility in the correct use of the English
language must be evidenced in the papers
The step next higher in the New York
State Service is the test for Bookkeeper, submitted by candidates.
Grades 5 and 6, details of which follow:
Appointments made to State Departments
and Institutions. Salary, $721 to $1,200. EXAMINER OF MUNICIPAL AC
Subjects and weights: Spelling, 1; Penman COUNTS, NEW YORK STATE SER
ship, 1; Test in Rapid Addition and Com VICE, SALARY $5 TO $15 A DAY
mercial Arithmetic, 2; Theory and Practice AND EXPENSES.
of Bookkeeping, 3; Business Experience
with special reference to Bookkeeping and The highest accountancy position in the
Office Work, 3. Last examination held No New York State Service is that of Exam
vember 3, 1917. iner of Municipal Accounts; details follow:
Last test held September 8, 1917. Ap BOOKKEEPER, U. S. FIELD SERVICE.
pointments made to State Comptroller's
office, at $5 to $15 a day, when employed, This examination is held only when eligi
and necessary traveling expenses in the bles are needed and will be announced by
field. The duties are to inspect and exam the District Secretary in whose district the
ine the accounts of counties, cities, towns, vacancy exists.
villages and school districts, pursuant to
the provisions of Article 3 of the General
Municipal Law. The position requires ac BOOKKEEPER, PANAMA
counting skill and a knowledge of the laws
CANAL
SERVICE.
covering financial transactions in municipal
itleS. This examination has been discontinued.
Subjects and weights: Municipal Ac Bookkeeping is given as an optional subject
counting and Laws relating thereto, 6; Edu
in the examination for Clerk, Panama Canal,
cation, Experience, Personal Qualifications examination.
and Adaptability to the work, 4. An oral
test was held.
Most of the bookkeeping and account
ancy examinations are open to both men BOOKKEEPER, PHILIPPINE SER
and women. VICE.
Age 20 to 40 years; men only; Applica
BOOKKEEPER, UNITED STATES tion Form, 2, including the medical certifi
cate; non-apportioned service.
SERVICE. Time allowed, eight hours. Subjects of
The examination for Bookkeeper in the examination and relative weights of sub
jects on a scale of 100: Spelling, 5; Arith
United States Service in the past has been metic, 10; Penmanship, 5; Report Writing,
held annually, but owing to war conditions
10; Copying and Correcting Manuscript, 5
the test is being held very frequently, the
United States Civil Service Commission (all First Grade); Practice of Bookkeeping,
being unable to secure a sufficient number embracing tests in journalizing and prepa
ration of balance sheet in mercantile trans
of eligibles. Accountants also are in great actions, 40; Elements of Auditing and Dis
demand at present. The examinations are bursing, 10; Training and Experience, 15.
being held under various titles. The de
tails follow: Competitors who fail to attain an aver
age rating of at least 70 in the sixth and
seventh subjects, with the relative weights
given above, will not be eligible for ap
BOOKKEEPER, U.S. DEPARTMENTAL pointment and the remaining subjects will
SERVICE. not be rated.

Age, 18 years or over; men only; Appli


cation Form, 304; medical certificate not
required; usually apportioned service, rare BOOKKEEPER, NEW JERSEY SER
ly non-apportioned. Usual salary, $1,000. VICE, SALARY, 1st GRADE, $1,500
Time allowed, seven hours. Subjects of GRADE,
examination and relative weights of sub TO $2,000; 2nd $1,000
jects on a scale of 100: Spelling, 10; Arith TO $1,500.
metic, 15; Penmanship, 10; Report Writing,
15; Copying and Correcting Manuscript, Subjects of the examination and relative
weights: Experience, 3; Commercial Arith
10 (the foregoing all are regular First Grade
metic, 4; Penmanship, 2; Theory and Prac
Clerk subjects); Practice of Bookkeeping,
embracing tests in journalizing and prepara tice of Bookkeeping, 4; Spelling, 2; total,
tion of balance sheet in mercantile trans 15.
actions, 40. Competitors who fail to re
ceive a rating of at least 70 in the subject
of Practice of Bookkeeping will not be
eligible for appointment, and the remain
ing subjects will not be rated. ACCOUNTANT AND STATISTICIAN,
Residents of all States and Territories NEW JERSEY SERVICE, SALARY
and the District of Columbia, who meet $2,000.
the requirements, will be admitted to this
examination, but eligibles whose bona fide Subjects of the examination and relative
residence is in the District of Columbia weights: Experience, 5; Accounting, 3;
will be certified for temporary employment Commercial Arithmetic, 2; Economics, Cor
or for employment in the non-apportioned poration Finances and Statistics, 5; total,
service only. 15.
BOOKKEEPER, NEW YORK CITY SERVICE.

Answers to Examination Questions and Specimen Ques


tions for 2nd, 3rd, 4th and 5th Grades.

Municipal Civil Service Commission, N. Y. Sold George White, City, furniture


amounting to $675 and received his check
BOOKKEEPER, 2nd GRADE. for $250, balance due in 30 days. Paid H.
D. French, rent for month of June, $175.
June 8. Paid freight on invoice of furni
(Salary, $840 to $1,200.) ture from R. D. Langley, $17.65. Cash sales
amounting to $376.48 were made. Paid for
Date: July 10, 1917. cleaning windows, $4.75. Sent R. D. Lang
ley check for amount of invoice of the 1st.

DUTIES (BOOKKEEPING)—Weight 6. June 14. Bought of Grand Rapids Fur


niture Company, Grand Rapids, Mich., fur
HANDWRITING AND NEATNESS— niture amounting to $1,296.43, terms 4
months’ note. Sold D. E. Sayles, City, bill
Weight 2, will be rated on this paper. of furniture amounting to $678.93, terms
2/5, n/30. John Doane drew $150 from the
(To be finished at 1 P. M.) business for personal use.
June 18. Received check from D. E.
Sayles for bill of goods sold him on the
For Questions 1 to 6 use journal, cash
book, sales journal (or sales book) and pur 11th. Gave Saunders & Company a note
chase journal (or purchase book). at 30 days for $967.81, being amount of in
voice of furniture received from them May
Special columns are required in the jour 16. Draft made by John Doane, June 15,
nal for Accounts Payable, Dr. and Accounts on E. M. Griffin, Binghamton, N. Y., at 30
Receivable, Cr. days in favor of G. H. Staley for $348.72,
Special columns are required in the cash balance of account due from Griffin has been
book: on the debit side, for Accounts Re received with aceptance.
ceivable and Sales; on the credit side, for On June 30, A. K. Morgan failed in busi
Accounts Payable and Expense. ness and compromised with his creditors on
the basis of 75 cents on the dollar. In ac
Ques. 1 to 6:—John Doane of Utica, cordance with this settlement we received
N. Y., is engaged in the furniture business his check for $566.22 in full of his account
and keeps his books by the double entry of $754.96. Cash sales this day $641.92. Paid
system. On the 31st of May, 1917, his cash the following: Salaries of clerks for month,
balance was $2,476.93. Under date of June $275; bill for gas and electric light for
1, open a cash book with this balance and month, $21.69.
record the following selected transactions Prepare the books of original entry for
in the proper books of original entry: posting, assuming that controlling accounts
June 1. Bought of R. D. Langley, Buf for sales and purchase ledgers are to be
falo, N. Y., a lot of furniture invoiced at used. Indicate the balance of cash.
$840.25, terms 2/10, n/30.
ANSWER:
DR. CASH (Left-hand page)
Disct. & Accts.
Cash Allow- Re- Cash Sun
Date L.F. aſc Credited Explanation Received ances ceivable Sales dries
1917
June 1
1 Geo. White
Balance 2,476.93 2,476.93 /
on a/c 250.00 250.00
8 Moise. Sales Cash Sales 376.48 376.48
18 D. E. Sayles Inv. 6/14 665.35 13.58 678.93
30 A.K.Morgan (See Journal
& Co. entry) 566.22 566.22
30 Mdse. Sales Cash Sales 641.92 641.92

4,976.90 13.58 1,495.15 1,018.40 2,476.93

4,976.90

Summary Journal Entry.


Dr. Cr.
Cash 2,499.97
Discount & Allowances 13.58

2,513.55
Accounts Receivable (Cont. a/c) 1,495.15
V Cash Sales 1,018.40

2,513.55

(Right-hand page) CASH CR.

Discts. &
- Cash Allow- Accts. Ex- Sun
Date L.F. a/c Debited Explanation Paid ances Payable pense dries
1917
June 1 Rent June, 1917 175.00 175.00
8 Moise. Purch. Fr’t Inv. R. D.
Langley 17.65 17.65
8 Gen'l Expense Window Clean'g 4.75 4.75
8 R. D. Langley Ck., Inv. June 1 823.45 16.80 840.25
14 John Doane Personal aſc 150.00 150.00
30 Salaries June, 1917 275.00 275.00
30 Gas & Elect. June, 1917 21.69 21.69

1,467.54 16.80 840.25 179.75 464.34


30 Balance 3,509.36

4,976.90

Journal Entry.
- Dr. Cr.
Accts. Payable (Controlling a/c) 840.25
V Expense 179.75
V Sundries 464.34

1,484.34
Cash 1,467.54
Discounts and Allowances 16.80
-
1,484.34
SALES BOOK
Cash Credit
1917 -
[une 1 Geo. White City (See Cash Book) 675.00
--
8 Cash Sales 376.48
44
14 D. E. Sayles --
2/5 N/30 678.93
30 Cash Sales 641.92

1,018.40 1,353.93

-
Dr. Cr.
Accounts Receivable (Controlling aſc) 1,353.93
Cash 1,018.40
Mdse. Sales 2,372.33

PURCHASE BOOK
Cash Credit

1917
June 1 R. D. Langley Buffalo, N. Y. 2/10 N/30 840.25
14 Grand Rapids. Furniture Co. Grand Rapids, Mich., note 4 mos. 1,296.43

2,136.68

Journal Entry.
Dr. Cr.
Mdse. Purchases 2,136.68
Accounts Payable 2,136.68

BILLS PAYABLE
When Drawer & Drawee In Whose For What Where When When and How
Favor Given Payable Date Time Due L.F. Amt. Disposed of
Given Endorser & Maker

5/14/17
- - - - - - - - - John Doane Grand Rapids Infull of Inv. Bank 6/14/17 4 mos. 10/14/17 . . . . 1,296.43 ............
Furn. Co. June 14/17
John Doane Saunders&Co. Inv. of May Bank 30 das. ............

-
- 6/18/17 7/18/17 .... 967.81
5/18/17 .. .. .. .. .
- 14/17

DR. JOURNAL CR.


June, 1917.

Accts. Accts. Re- Accts. Re- Accts.


Payable ceivable L.F. ceivable General Payable
General
—14—
1,296.43 Grand Rapids Furn. Co.
Notes Payable 1,296.43
Note for 4 months in payment of
June 14, 1917, invoice.
—18—
967.81 Saunders & Co.
Notes Payable 967.81
Note for 30 days in settlement
of May 16, 1917, invoice.
—18—
348.72 G. H. Staley
E. M. Griffin 348.72
Draft accepted by Griffin in fa
vor of Staley.
—30—
188.74 P. & L. a/c
A. K. Morgan Co. 188.74
Being 25% on the amount due,
lost through failure and set
tlement on 75% basis.

2,612.96 188.74 537.46 2,264.24


Ques. 7:—Reconcile the following bank
account: Balance per check book, $1,076.48;

standing, $278.39,

interest on the
$78.42, $67.43,

exchange on out of town checks, $3.46.


ANSWER:
Balance
Add Interest credited but not en-
.-
balance per pass book, $2,01751; checks out-
$323.98,
$179.56; items not entered in the check book,

º: and

as per check book. . . . . . . . $1,076.48


*::::

tered . . . . . . . . . . . . . . . . . . . . . . . . . 16.71 (
ºn
Fixed Assets.

. ... ... .. . ... . ... .. .... .

.......................’.......
-

. .. .. .

- - -- - --

$........
' ' ' '' ''' ' ' ' ' '

Fixed Liabilities.
- - --- --
. $........
$........

$1,098.19 ... ... ... .. ... ... ... . . .. . .. . .... .


Less Exchange charges. . . . . . . . . . . . 3.46 y
Proprietorship.
True Balance . . . . . . . . . . . . . . . . $1,089.73 × .. .. ... . .. ... . ... ... .. ... $.......
Balance as per Bank.............. $2,017.51
Checks outstanding— $278.39 .. . .. .. . ... . ... .. .. . .. ... $.......
78.42 .. .. . . . .. . .. ... .. .. . ... . . . . .. . ..
67.43 --~
323.98 ... . . . .. . . .. .. . .. .. .. ... . . ... . ... -
179.56 927.78 - - - - - - - - - - - - - - - - - - - - - - - - -$ - - - - - - -

Balance as per check book. . . . . . . . $1,089.73 .... .. ... ... . .. .. .. . ... . . ... ....

Ques. 8:—(a) Write the note called for in $. . . . . . . .


transaction of June 14 (Questions 1-6).
(b) Write the draft mentioned in the ANSWER:
transaction of June (Questions 1-6). Current Assets.
ANSWER (a):
A..."...
ºcj ".
Cash on hand and in bank
Y.,

14,

$1,296.43
Utica, N. June

.
1917. Va le

D
Four months after date promise pay
to

.
.
.
.
.
.
.
.
.
.
.
Bills
I

e
The Grand Rapids Furniture
to

of

the order

-
-
-
-
-
-
-
-
-
-
-
-
-
Co. Twelve hundred and ninety-six and º
Merchandi Se 111Velltory
int

.
.
.
.
.
.
.
.
.
.
.
.
.
.
.
.
.
.

my office, Pine Street, Fixed Assets.


35
at

43/100 dollars
Utica, N. Y. Land and Buildings.
.
.
.
.
.
.
.
.
.
.
.
.
Value received. Machinery and Equipment

.
.
.
.
.
.
.
.
.
.
.
.
.
.
.
JOHN DOANE. Current Liabilities.
No. Due Oct. 14, 1917.
3.

Notes Payable

§
....... ........
.
.
.
.
.
.
.
.
.
.
.
.
.
.
.
.
.
.
(b) Accounts Payable........
Utica, N. Y., June
;

15, 1917 Fixed Liabilities.


-
Mort
Mortgage Payable
Payable
Thirty days after date, pay
.
.
.
.
.
.
.

.
.
.
.
.
.
.
.
to

the order -
Proprietorship.
-

H. Staley.
of
G.

.
.
.
.
.
.
.
.
.
.
.
.
.
.
.
.
.
.
.
.
.
.
.
.
.
.

Three hundred forty-eight and 72/100 dol- Samuel Higgins aſc-Bal

-
lars. Value received, and charge ance June
1,

account
to

1916.
.
.
.
.
in .
.
.
.
.
.
.
.

of JOHN DOANE. Deduct Drawings


salary.
of

To E. M. Griffin excess
.
.
.
.
.
.
.
.
.
.

Binghamton, N. Y.
Accepted,
June 17, 1917, Add Profit for period.
*

.
.
.
.
.
.
.
.
.
.
.
.
.

E. M. Griffin. Frank Jasper aſc-Bal


Ques. 9:—The following
of

an outline ance June


1,
is

1916.
Drawings
.
.
.
.
in.
.
.
.
.
.
.
.

Samuel Higgins and Deduct


of

the balance sheet


Frank Jasper, partners. Copy the outline salary.
of

excess
.
.
.
.
.
.
.
.
.
.

and on the blank lines write the titles of


accounts that might properly appear such
in

Add Profit for period.


.
.
.
.
.
.
.
.
.

balance sheet. Do not supply the amounts.


a

.
.
.
.
.
.
.
.
.
.
.
.
.
.
.

Current Assets.

Ques. 10:—In the general ledger


S.
E.
of

Beake Co., the Accounts Receivable ac


&

debit footing $16,987.34


of

count shows
a

credit footing $9,671.52; the Ac


of

and
a

counts Payable account shows debit foot


a

Note:—The acceptance $7,


written across
$ii. $3,672.91 and footing
of

credit
*

is

the face of the draft.


The Sales Ledger shows the following SOLUTION:
accounts with debit balances: A. H. Coates,
$429.61; C. E. Burton, $1,738.42;
(NOTE:—As the method of averaging
M. V. accounts is not generally understood,
French, $987.64; J. E. Hudson, $1,576.91; a
word of explanation is perhaps in order.)
M. A. Thurston, $1,238.73; F. M. Thompson,
$1,344.51. (To find the average date of the three
The Purchase Ledger shows the follow amounts in this problem, we will assume
ing accounts with credit balances: that each item was due at a certain date,
W. K.
Thornton, $367.89; E. M. Goff, $728.62; H. say March 1st; in other words, we
will
regard them like cash transactions for the
B. Cowles, $1,438.96; A. R. Knapp, $1,075.
Prepare abstracts of the purchase and purpose of establishing a basis of operation.
sales ledgers, showing their agreement with On this assumption, it is evident that the
their respective controlling accounts in the debtor would owe the creditor, not only the
general ledger. - amount of each bill, but also the interest.)
The transactions were not for cash, how
ANSWER: ever, but on time. The debtor, therefore,
Accounts Receivable Trial Balance. is entitled to the $309.17 interest, i. e., he
A. H. Coates 429.61 can hold the $42,000 for such time after
C. E. Burton 1,738.42 As per Con March 1st as it would take that sum to
M. V. French 987.64 trolling aſc yield $309.17 interest at 6 per cent per an
J. E. Hudson 1,576.91 Dr. 16,987.34 num. The proposition then resolves itself
M. A. Thurston 1,238.73 Cr. 9,671.52 to this: How long will it take $42,000 to
F. M. Thompson 1,344.51 accumulate $309.17 interest? This can be
done by proportion, knowing that it will
Total as per Con take 12 months for $100 to accumulate $6
trolling a/c 7,315.82 7,315.82 interest.

Accounts Payable Trial Balance. $42,000 : 100 :: (12) 2 mos.


W. K. Thornton 367.89 As per Con- ($6) : $309.17
E. M. Goff 728.62 trolling aſc - -
H. B. Cowles 1,438.96 Cr. 7,284.35 (The figures in parentheses are cancelled.)
A. B. Knapp 1,075.00 Dr. 3,672.91 By cancellation 6 disappears and 12 becomes

-
2. Then we have $309.17 multiplied by 2,
3,610.47 or $618.34; then multiplied by 100, or $61,


Difference with --- 834. Dividing by $42,000 we get 1.4 months.
Controlling aſc .97+
1.4 months equals 42 days, the average
term of credit; and the equated time of pay
3,611.44 3,611.44 ment, April 12th.

Principal Interest
Amount of bill, $14,000; interest March 1–March 21, 20 days at 6%.......... $46.67
Amount of bill, 21,000; interest March 1–April 17, 47 days at 6% . . . . . . . . . . 164.50
Amount of bill, 7,000; interest March 1–May 23, 84 days at 6% . . . . . . . . . . 98.00

42,000
$309.17

* The difference probably is due either to Ques. 2 and 3:—New York, July 12, 1916.
a bill of 97 cents not having been credited On demand I promise to pay to the order
to someone's account, or to a payment of of Albert Higgins. . . . . . . . . . . . . . . . . . $475.00,
the same amount not having been entered Four hundred seventy-five dollars. Value
in the Controlling Account column in the received. At National City Bank, with in
Cash Book. terest at 6 per cent.

ARITH METIC–Weight JEROME DEXTER.


2.
(Give all the figuring on the ruled sheets.) Endorsed as follows: Aug. 12, 1916, Paid
$15.00; Sept. 17, 1916, Paid $25.00;
(To be finished at 2.30 P. M.) Dec. 22,
1916, Paid $100. What amount must be paid
Ques. 1:—Payments are due to be made to settle this note on June 22, 1917?
as follows, interest to be allowed on any
payments not made when due and discount SOLUTION:
on payments made before maturity:
(NOTE:—In the matter of partial pay
March 21 . . . . . . . . . . . . . . . . . . $14,000 ments the U. S. courts have decided that
April 17 . . . . . . . . . . . . . . . . . . . 21,000 interest to date of partial payment must be
May 7,000 computed at the time of such payment.)
23 ... . ... . .. .... . .. ..
The difference between interest and par
Upon what date may the whole amount tial payment is deducted from principal to
of $42,000 be paid without loss to debtor or get new principal for next interest calcula
creditor? tion. In this particular problem each of
the three payments exceeds the accrued in Consumer paid $3.22, which is 161% of Munici
terest. cost of manufacture.
1.61)3.22
Principal, July 12, 1916. . . . . . . . . . . . $475.00 Cost of manufacture $2 (Ans.).
Int. from July 12, 1916, to Aug. 12, 2.
1916 (1 mo.) 6%. . . . . . . . . . . . . . 2.37
Ques. 5:—Three partners, A, B and C, had
Amount . . . . . . . . . . . . . . .. . . $477.37 shares in a business proportional to the
First payment, Aug. 12, 1916. . . .. .. 15.00 numbers 4, 5 and 6, respectively. C retired
and received as his share of the business
- New Principal... . . . . . . . . . . $462.37 $15,000. How much of this money should
Int. from first payment to Sept. 17, be paid by A and B respectively in order
1916 (1 1/6 mos.) 6% . . . . . . . . . 2.70 that after C's retirement their shares might
be equal?
$465.07 SOLUTION:
Second payment, Sept. 17, 1916.... 25.00
A X 4

* New principal . . . . .. ... ... . $440.07


B X 5

payment
C $15,000 6
Int. from second to Dec.
22, 1916 (3 1/6 mos.) 6%. . . . . . 13.93
15
6/15=15,000.
$454.00 1/15=1/(6)}{(15,000) 2,500–2,500. (Fig
Third payment, Dec. 22, 1916. . . . . . 100.00°
ures in parentheses are canceled.)
15×2,500−$37,500, worth of A, B and C.
New principal . . . . . . . . . . .. $354.00
A 4X2,500−$10,000."
Int. to settlement date, June 22,
B 5X2,500- 12,500.
1917 (6 mos.) 6%. . . . . . . . . . . . . 10.62 *
C 6X2,500- 15,000.
% of $37,500−$18,750.
Amount due June 22, 1917 (Ans.) .. $364.62
$18,750–$10,000-$8,750 A's payment (Ans.)
18,750— 12,500= 6,250 B's payment (Ans.)
It should be noted that the law does not
permit the excess of interest over partial $15,000 Received by C.
payment to be added to principal. Where

-
the interest exceeds the installment paid, Ques. 6:—On $2,000 worth of goods (list
calculation of interest continues on the for price), a merchant allows a discount of 15
mer principal until the period when the pay per cent, 9 per cent and 5 per cent for cash,
ments taken together exceed the interest then 9% per cent to clinch the bargain; how
due. This excess is then deducted from much cash did he receive and how much
principal, as in the above example. profit did he make, his rate of profit being
-
8 per cent of cost?

-- --
Ques. 4:—A manufacturer sold at a profit SOLUTION:
of 25 per cent to a wholesale dealer, who
sold at a profit of 12 per cent to a retail $2,000 1467.81=108% of cost

-
dealer, and the retail dealer sold for $3.22, .85
1359.08 cost
and made a profit of 15 per cent; what was
the cost of manufacture? 10000
16000

- -
1.08)1467.81
108
SOLUTION: 1700.00
Cost of manufacture 100% .91 387
Manufacturer's profit 25% 324
1700 a
Wholesale dealer's cost 125% 15300 638

-
Wholesale dealer's profit .12
540
1547.00
250
.95 981
125
972
7735
Wholesale dealer's profit 15.00
13923 900
Wholesale dealer's cost 125.00
864
1469.65
Retail dealer's cost 140.
Retail dealer's profit .15 1469.65–5–100-(14.69) 1.84X1/(8)=1.84.
1.84
700
140 1467.81 Selling price (Ans.).
(Figures in parentheses cancelled.)
Retail dealer's profit 21.00 Selling Price $1467.81
Retail dealer's cost 140.00 Cost 1359.08

Selling price to consumer 161%. Profit $108.73 (Ans.)


10
Iunicipal Civil Commission, N. Y. the subsidiary books. The totals

of
Service these
columns, the postings are correct, will
BOOKKEEPER,

of if
3rd GRADE. be totals the posting mediums for the
(Salary, period under consideration; the totals do

if
$1,200 to $1,800.)
not agree their sources error will have

of
Date: July been located orig

in
9, 1914. the different books

of
inal entry and detailed checking may be
DUTIES-Weight done

in
5. the books which show errors.
HANDWRITING AND NEATNESS—
Ques. 2:—(a) Name six accounts that are
Weight 2—will closed into Loss and Gain Account.
be rated on this paper.
(b) Name three accounts, other than mer
(To be finished by 2.30 P. M.) chandise account, which you would have

in
to
take an inventory

to
in

at
order arrive
ANSWERS BY JOHN J. MULDOON, the true condition of each account men
No. 1 on the List, who made 95.90 per cent. tioned. Explain your answers by examples.
(c) Name five requisites

as
(NOTE:-The form which

to
answers are as given at note, draft

or
other negotiable paper

or
the examination.)

a
instrument should have.
Ques. 1:—(a) When is the Loss and Gain (d) What meant by the following

is
Account debited, and when credited? terms: (1) Capital Stock; (2) Common
...

liabil
.
.

(b) State the difference between Stock; (3) Preferred Stock; (4) Cumulative
a

ity and loss; resource and gain. Preferred Stock?


a
a

a
do

(c) What you understand by the goºd ANS.:—(a) Merchandise, expense, wages,
.
of

will” business? How may “good will stationery, telephone, rent.


a
be

regarded making
in

financial statement (b) Insurance: Prepaid insurance, for


a

of business?
a

which the total period prepayment has

of
In

(d) assuming charge


of

not elapsed, the inventory value being the


of

set books,
a

on
of

state the method procedure the part amount insurance applicable


of

period

to
in of

the bookkeeper when the ledger not unexpired. Stationery and Supplies:
in is

This
balance. Give mode procedure full
of

given credit for the quan


be

account should
under the varying conditions that may arise. tity remaining on hand. Coal and Engine
ANS.:—(a) The Loss and Gain. Account Room Supplies: This account should be
in
an

debited when expense outlay given credit for the amount unused
or
is

is

the

at
curred which has not resulted the ac
of

the period.
in

end
quisition asset. Similarly, the account
an
of

(c) Must be
an

promise

to
unconditional
credited when assets are acquired which
is

pay
or

fixed determinable sum money


orof
a
do

an

not necessitate parting with


of

certain party
to

asset his order,


or

bearer,
a

equal greater value nor the setting up


or

writing and signed by the


be

and must
in
an
of

equal greater liability. The ac


or

maker.
.

count also debited when the value as


is

of

(d) Capital Stock the ownership


1.

is

sets parted with greater than those ag


of

the equity
of

corporation, usu
is

and
is
a

quired. These entries are usually made ally stated par value.
of in

or
at

nominal
a

the Loss and Gain Account the close Stock


at

Common
2.

the evidence of
is

the accounting period. ownership


of

- corporation, the holder be


toa

Liability
-

(b) ing entitled


an

obligation
A

which
is

receive dividends on his


will ultimately have
be

paid.
to

Loss stock when they are earned and declared,


A

is

the proprietor
in

the value
of

decrease being
a

and participate pro rata


to

entitled
ship which has already been sustained.
the cor
in

the distribution
of

the assets
of

(c) “Good Will” poration when


an

intangible
is

asset dissolved.
is
it
up

made
of

the location the business, Preferred Stock


of

3.

stock to which
is

the reputation the concern and the prob preference


of

given over other stock


is

in

ability that the same customers will con: the


payment dividends,
of

to

certain extent.
to a

tinue trade with the concern.


to

“Good may also


be
It

entitled preference
in

Will,” where has been actually purchased, the distribution assets, and may
of

may
it

of or

may
be

as

an

shown asset not have


in

financial voice
in

the conduct the


a

statement. Conservative management, how corporation. These things are all subject
ever, would suggest that portion
of

this the agreement, made


to

of
at
a

the time the


“Good Will” be written off periodically. issue the stock, which define its rights
of

(d) The first step would


be

get the and limitations.


to

trial balance for the period when the ledger Cumulative preferred pre
4.

stock
a is

was last actually balance. Then anal


an

ferred stock on which dividends


in

at

fixed
ysis the ledger from the time
of

paid before any dividends are


be
ofof

the last rate must


correct trial balance
to

the time assum declared on the common stock. In the


ing charge
of

a be

the books should


of

made. event the dividend not being declared


in

This analysis made by listing on multi any one year the amount fixed as dividend
is

columned sheet the names captions


in or

of

charge against prof


on

the stock remains


all

the ledger accounts, and the columns


of

its the corporation, and these charges


headed with the names the different post
of

are accumulated from year year, and all


to

ing mediums should


be

shown the amount must be paid before any dividend can be


that ledger account from each paid on the common stock.
to

posted
of

11
Ques. 3:—(a) Why do bookkeepers use a June Rent, $250.

2.
Suspense Account? State fully how the To Cash 250.00

.
.
.
.
.
.
.
.
.
.
.
.
.
.
.
.
.
.
.
.
.
items recorded under this account are treat Paid rent for store.
ed, and explain in detail the final disposition Proprietor—drawing
June ac

3.
made of these items.

.
count, $100.
(b) Show the advantages to a business in To Cash 100.00

.
.
.
.
.
.
.
.
.
.
.
.
.
.
.
.
.
.
.
.
.
.
keeping “Depreciation.” Accounts and give Drawn for private use.
a simple method used in calculating “De
June Loss and gain, $47.50.

4.
preciation.”
To Cash 47.50
ANS.:—(a) The Suspense Account is

.
.
.
.
.
.
.
.
.
.
.
.
.
.
.
.
.
.
.
.
.
Cash lost from drawer.
used when transactions occur which are
not clearly assignable to any other account. June Cash, $800.

5.
Whenever it is finally determined to what To prop.–Cap. Acct.......... 800.00
accounts the items contained in this ac Additional investment.
count belong they are transferred to these June Cash, $47.50.

6.
accounts. Its most general use is with To Loss and Gain............. 47.50
items involving a loss, the amount of which Recovery

of

to
cash said have been
cannot be accurately determined. These lost.
items are carried in the Suspense Account June Insurance, $37.65.

8.
until the determination of the amounts;
To Cash 37.65
the items are then transferred to the Loss

.
.
.
.
.
.
.
.
.
.
.
.
.
.
.
.
.
.
.
.
.
Insurance on merchandise.
and Gain and other accounts.
(b) Assets, such as might be called Cap June Notes payable, $500.

9.
To Cash 496.00
ital Assets, those which are for permanent

.
.
.
.
.
.
.
.
.
.
.
.
.
.
.
.
.
.
.
.
.
Discount 4.00
use in the business and not for sale, are

.
.
.
.
.
.
.
.
.
.
.
.
.
.
.
.
.
.
.
.
.
$500; discount, $4.

of
subject to depreciation in value through Paid note
use, wear and tear, and obsolescence. The (b) B's farm, $6,000.
amount of this depreciation should be writ To house and lot.............. $3,400.00
ten off periodically so that the true value Mortgage B's farm............ 1,500.00
of the assets may be shown in the balance Accrued interest on mortgage... 164.17
sheet, and that the true costs of operating Cash 835.83

.
.
.
.
.
.
.
.
.
.
.
.
.
.
.
.
.
.
.
.
.
.
.
.
.
for the period be determined. Some such mortgage
Purchased B's farm subject

in to
assets are real estate, machinery, tools, fur interest; gave payment
niture and fixtures. One method of calcu and accrued
house and lot valued Cash for

at
$3,400.
lating depreciation would be to determine
the life of the asset to be depreciated ap
balance.
is,

Dioximately; that how long will be


it

(EDITOR'S NOTE:—Cash paid should


before the asset will have be replaced,
to

of
be $935.83 instead $835.83.)
and whether will have any residual value.
it

If, say, $1,100 will last


a at

machine valued Ques. and 6:—Make out balance sheet


a

a
ten years and then have scrap value from the following items:
of

$100, depreciation might be fixed


at

the Dr. Cr.


$100 per annum. Or again,
of

rate fixed Proprietor $3,200


a

percentage per annum might be used.


.
.
.
.
.
.
.
.
.
.
.
.
.
.
.
.

Cash $2,800 2,265


.
.
.
.
.
.
.
.
.
.
.
.
.
.
.
.
.
.
.
.
.

following B., Receivable 1,700 1,300


Ques. 4:—(a) Journalize the
.
.
.
.
.
.
.
.
.
.
.
.

transactions: B., Payable 450 2,000


.
.
.
.
.
.
.
.
.
.
.
.
.
.
.

Black 325 795


J.

June The proprietor began business


.
.
.
.
.
.
.
.
.
.
.
.
.
.
.
.
.
.
1.

R. Gilbert 730 150


w.th cash on hand, $7,850.45.
.
.
.
.
.
.
.
.
.
.
.
.
.
.
.
.

R. M. Brown. 880 250


Paid rent for store, $250.
.
.
.
.
.
.
.
.
.
.
.
.
.

June
3. 2.

Mdse. 5,600 3,800


June The proprietor draws cash from
.
.
.
.
.
.
.
.
.
.
.
.
.
.
.
.
.
.
.
.

business for private use, $100.


Reading Railroad Stock.... 4,000 2,500
Expense 375
June By counting cash discovered
.
.
.
.
.
.
.
.
.
.
.
.
.
.
.
.
.
.
4.

is
it

Commission 600
.
.
.
.
.
.
.
.
.
.
.
.
.
.

that $47.50 has been lost.


June ad
The proprietor has made an There merchandise on hand, $1,500;
5.

is

cash, $800.
of

ditional investment Reading Railroad Stock (unsold), $2,000.


June The cash, supposed
to

$47.50,
6.

have been lost on 4th, has been found. ANSWER:


June Paid insurance on goods, $37,65. Cash $535
9. 8.

.
.
.
.
.
.
.
.
.
.
.
.
.
.
.
.
.
.
.
.
.
.
.
.
.
.
.
.
.

June John Smith owes note, $500. Reading Railroad Stock invy... 2,000
a

.
.
.
.

He pays the same, less the discount, $4. Mdse. inventory 1,500
.
.
.
.
.
.
.
.
.
.
.
.
.
.
.
.
.
.

(b) exchanged his house and lot, valued Bills Receivable 400
A

.
.
.
.
.
.
.
.
.
.
.
.
.
.
.
.
.
.

$3,400, with for his farm, valued R. Gilbert 580


$6 at

at
B

.
.
.
.
.
.
.
.
.
.
.
.
.
.
.
.
.
.
.
.
.
.
.
.
.

assumes the mortgage on B's R. M. Brown. 630


A

000.
.
.
.
.
.
.
.
.
.
.
.
.
.
.
.
.
.
.
.
.
.

farm for $1,500, with interest due date,


to

$154.17, and pays the balance cash. Give $5,645


in

jcarnal entry A's books.


in

Bills Payable $1,550


.
.
.
.
.
.
.
.
.
.
.
.
.
.
.
.
.
.
.
.
.

ANSWER (a): Black 470


J.

.
.
.
.
.
.
.
.
.
.
.
.
.
.
.
.
.
.
.
.
.
.
.
.
.
.

June Cash, $7,850.45. Proprietor 3,625


1.

.
.
.
.
.
.
.
.
.
.
.
..
.
.
.
.
.
.
.
.
.
.
.

To prop., Cap. Acct. $7,850.45


.
.
.
.
.
.
.
.
.
.

To charge cash with investment. $5,645

12
Ques. 7 and 8:—A and B are equal part ARITH METIC–Weight 3.
ners. B retires from the firm, and an ad
justment is made. The partners' accounts (To be finished at 4 P. M.)
stand as follows: A debtor $6,100, credit
$8,100; B debtor $5,100, credit $7,800. They (Give all the figuring on the ruled sheets.)
divide their resources and liabilities as fol. prob
lows: Cash, which B takes to his account,
(NOTE:-In calculating interest
lems, allow 360 days to the year.)
$4,400; Bills Receivable, which B also takes
to his account, $3,700; Mdse. on hand, which Ques. 1:—For what sum must A draw up
A takes to his account, $5,600; they owe on a note payable in 3 months to enable him
account Notes, which A assumes, $2,200; to take up the note for $800 at 6 months,
Personal Accounts Receivable, on which A with interest at 4 per cent in each case?
is allowed 10 per cent discount for bad
debts, $6,300; Personal Accounts Payable, ANSWER:
which A assumes, $1,100. The total net Note, $800.
credit of Loss and Gain Account, after all
$32 interest 1 year at 4%.
loss and gain accounts are closed, is $12,000.
$16 interest 6 months at 4%.
Tell which partner is indebted to the other, $800+$16=$816, amount now due.
and how much.
4% per annum=1% for 3 months.
ANSWER: 100%—1%=99%=$816.
Cash .. . . . ... . . ... . . .. .. . . . . .. . . .. $4,400
Bills Receivable . .. ... . . . . . . . .. ... 3,700 99)816(824.24 $824.24=face of note.
Merchandise ............ . . . . . . .. .. 5,600 792
Accts. Receivable...... . . . . . . $6,300
Less Bad Debts. . . . . . . . . . . . . 630 5,670 240 Proof $824.24
198 Dis. 1% 8.24
$19,370
420 $816.00 now due
Accounts Payable . . . . . . . . . . . .. . .. $1,100 396
Notes Payable . . . . . . . . . . . . . .. . .. . 2,200
A (capital account). . . . . . . . . $2,000 240
(Half profits) . . . . . . . . . . . . . . 5,685 7,685 198

B (capital account). . . . . . . . . . $2,700 420 (Ans.) $824.24


(Half profits) . . . . . . . . . . . . . . 5,685 8,385 396
-
$19,370.2. 24

Net Profit . . . . . . . . . . . . . . . . . . . .. . .. $12,000 Ques. 2:—A bought a horse for $250, and
Less Bad Debts allowed. . . . . . ..... 630 sold the horse for $262.50. Later on he
bought the horse back for $240, and then
$11,370 sold him for $255. What did he gain and
what was the gain per cent?
Half to A. . ... .... .. .. .. .. .. .. .. .. $5,685
Half to B. . . ... ... .. .. .. . .. . .. .. .. 5,685
ANSWER:
Invested $250; sold for $262.50; profit
A's Capital Account: $12.50.
Merchandise .................. $5,600 Bought for $240; sold for $255; profit $15.
Accts. Receivable. . . . . . . . $6,300 Total profit, $27.50.
Bad Debts . . . . . . . . . . . . . 630 5,670
250) 27.50 (11
250
$11,270

250
Net Capital......... . . . . . . . . . . . $7,685 -
Notes Payable assumed. . . .. .. . 2,200
250 Profit on $250 is 11%.
Accts. Payable . . . . . . . . . . .. .. . 1,100
Ques. 3:-On May a contractor
1, 1914,
Balance . . . . . . . . . . . . . . . . . . .. . . . 285 bought 750 barrels of flour at $5.75 a barrel,
and on the same day sold it at $6.80 a barrel,
$11,270 receiving in payment a note due July 3,
1914, which he discounted in bank at 8 per
B’s Capital Account: cent on the day of the sale. What was his
Cash . . . . . . . . . . . . . . . . . . . . . . . . . $4,400 profit?
Bills Receivable . . . . . . . . . . . . . . 3,700
Balance . . . . . . . . . . . . . . . . . . . . . . 285 ANSWER:
750 750
$8,385 575 680

Net Capital .. . .. ... . .. . .. .... $8,385 3750 6000


5250 4500
A has debit balance of . . . . . . . . . . . . $285 3750
B has credit balance of............ 285 $5100 selling price total.
A therefore owes B. . . . . . . . . . . . . . . 285 $4312.50 purchase price total.
May 1 to July 3=2 months 2 days. Municipal Civil Service Commission, N. Y.
$51.00-6% for 2 mos.
1.70=6% for 2 days. 3rd GRADE BOOKKEEPER.
(Salary $1,200 to $1,800.)
$52.70=6% for 2 mos., 2 days.
17.57=2% for 2 mos., 2 days. Date: July 9, 1913.

$70.27=8% for 2 mos., 2 days. TECHNICAL–Weight 5.


$5,100.00 face of note
70.27 discount HAN DWRITING AND NEATNESS—
Weight 2—will be rated on this paper.
$5,029.73 net proceeds
4,312.50 purchase price (To be finished by 2 P. M.)

$717.23 profit (Ans.). (NOTE:—This examination was cancelled


on the ground that the questions were too
difficult for the grade.)

Ques. 4:-A note of $1,624, dated Novem Ques. 1:—(a) The following balances,
ber 16, 1912, due June 6, 1916, and bearing among others, appear in a ledger on June
interest at 4% per cent, was sold for its 30, 1913. State what you understand to be
present worth at 5 per cent, January 12, the exact meaning of each: Cash Dr. $250,
1914. What was received for the note when Machinery Dr. $1,000, Wages Dr. $5,000.
it was sold, and what was the discount? Withdrawals Dr. $1,700, Capital Cr. $25,000,
Discounts Cr. $500, Bills Payable Cr. $6,000.
ANSWER: (b) How should the items given below be
posted from the cash book? Give reasons:
November 16, 1912, to June 6, 1916=3 Cash Dr. $500. -
years, 6 months, 20 days. Balance on hand at commencement of
January 12, 1914, to June 6, 1916=2 years, period $1,000.
4 months, 24 days. Cash Cr. $600.
30 days to month.
Cash paid for merchandise $7,500.
ANSWER (a):
3 years, 6 months, 20 days=3 200/360= Cash Dr. $250. Balance of cash on hand.
3 5/9 years. Machinery Dr. $1,000. Book value.
2 years, 4 months, 24 days=2 144/360= Wages Dr. $5,000. Disbursements there
2 2/5 years. f Or.
Withdrawals Dr. $1,700. Amount with
$1624 drawn from business.
4%% Capital Cr. $25,000. Amount invested.
6496
Discounts Cr. $500. Gained by the busi
neSS.
812
Bills Payable Cr. $6,000. Owed by the
business.
$73.08 interest 1 year
(b) These items should be posted to the
3 5/9 1/9–8.12 ledger as follows:
219.24 Cash Dr. $500. To the credit side of the
40.60 account, representing the source of receipt,
e.g., John Brown (a customer).
259.84 interest 3 5/9 years at 4%% Cash Cr. $600. To the debit side of ac
1,624.00 principal count, representing recipient or object of
payment, e. g., Wm. Smith (a creditor).
$1,883.84 principal and int. due June 6, 1916. Cash paid for merchandise $7,500. To the
debit side of Midse. Purchases aſc, this be
2 2/5 years at 5% =12%. ing for a cash purchase.
1.12) 1883.84 (1682. present value, Jan. 12, 1914 It is unnecessary to make posting to the
112 complementary account, Cash for the indi
.vidual transactions. Although the cash
763 book is really a detailed Cash Account, a
672 summary Cash Account is frequently found
in the ledger. In such cases, postings are
918 made to the Cash Account periodically,
896 debiting it with the total cash received dur
ing the period and crediting it with the
224 total payments made.
224 Balance on hand at commencement of pe
riod $1,000. No posting is necessary here,
$1,883.84 due June 6, 1916. as this item represents merely the differ
1,682.00 value Jan. 12, 1914. ence between receipts and disbursements,
provision for posting which has been indi
$201.84 discount. cated in the foregoing.

14
Ques. 2 and 3:—Find the errors in the Bills Payable . . . . . . . . . . . 102,000
trial balance of the firm of U. & Tools & Patterns. . . . . . . . 6,000
§llowing Mortgage . . . . . . . . . . . . . . . 47,000
Customers' Controlling
Dr. Cr. a/c . . . . . . . . . . . . . . . . . 31,000
.
Capital . . . . . . . . . . . . . . . .
Rent . . . . . . . . . . . . . . . . . . . $10,000
$103,000 Freight aſc . . . . . . . . . . . . .
Mdse. . . . . . . . . . . . . . . . . . . .
39,000
45,000
Cartage a/c . . . . . . . . . . . . . 57,000 Cash . . . . . . . . . . . . . . . . . . . . 19,000
Creditors' Controlling aſc 38,000 Horses & Wagons. . . . . . . 11,000
P. & L. a/c. . . . . . . . . . . . . . » Real Estate . . . . . . . . . . . . . 60,000
Bills Payable. . . . . . . . . . . . 102,000 Light & Power. . . . . . . . . . 18,000
Tools & Patterns. . . . . . . . 6,000
*
Mortgage . . . . . . . . . . . . . . . 47,000 $300,000 $300,000
Customers' Controlling Ques. 4 and 5:—On January 7, 1911, D
a/c . . . . . . . . . . . . . . . . . 31,000
sells to the following: 724 pieces linings,
E
Freight aſc. . . . . . . . . . . . . . 39,000
40 yards to the piece, at 12%.c. per yard;
Mdse. . . . . . . . . . . . . . . . . . . . 45,000
terms, six off, thirty, 30 days' extra dating.
Cash . . . . . . . . . . . . . . . . . . . . 19,000
The buyer makes a shortage claim of 80
Horses & Wagons. . . . . . . 11,000
yards, which is allowed, and then sells the
Real Estate . . . . . . . . . . . . . 60,000
goods to G for $1,000 cash. When the bill
Light & Power. . . . . . . . . . 18,000
becomes due, E sends check in settlement,
which check is returned unpaid, with pro
test charges of $2.50. On March 15 the mat
The following were Expense Accounts: ter is adjusted by E giving D a note for
Cartage, Freight, Light & Power. The rent $2,000 payable in three months, endorsed
is overdue and payable by the firm to the by G, balance in cash, all accrued interest
landlord. The Profit & Loss account rep charges being calculated at the rate of 6%.
resents bad debts. The Tools, Patterns, D at once gets the note discounted at the
Horses and Wagons have been bought dur H National Bank. By the time the note
ing the past year. The Mortgage is a lien has fallen due, E has failed, and G com
against the Real Estate, which Real Estate promises by paying $1,000, the creditors
belongs to the firm. agreeing to look to the assets of the bank
rupt firm for the balance. The bank there
ANSWER: upon deducts the amount due it from D's
Dr. Cr. balance (deposit account). The bankrupt
Capital $103,000 firm finally pays 40 cents on the dollar.
Rent . . .
. .. .. ... . .. .. . ...
.... ... .. .. ... .. 10,000 Show by ledger accounts a record of all
Cartage a/c . . . . . . . . . . . . . $57,000 these transactions, (a) on the books of D;
Creditors' Controlling aſc 38,000 (b) on the books of G.
P. & L. a/c. . . . . . . . . . . . . . 14,000 ANSWER:

r- Books of D.

E's Account.
1911
1/7 Mdse. sales . . . . . . . . . . . . . $3,620.00",
f 1911
1/7 Allowance . .. .. .. . ... ... $10.00.
V
3/7 CK. ret'd . . . . . . . . . . . . . . . 3,393.40/ 3/7
--
Cash . . . . . . . . . . . . . . . . . . . 3,393.40

-
Disc. sales . . . . . . . . . . . . . 216.60 V Disc: sales . . . . . . . . . . . . . 216.60
Fees . . . . . . . . . . . . . . . . . . . 2.50 3/15 Notes rec. . . . . . . . . . . . . . . 2,000.00 tº
3/15 Int. . .. ... .. . ... .... . .. . 4.52 Disc. sales 216.60
6/15 Notes rec. . . . . . . . . . . . . . . 1,000.00 Cash . . . . . . . . . 1,400.42
6/15 Cash . . . . . . . . . . . . . . . . . . . 800.00
P. & L. . . . . . . . . . . . . . . . . . 200.00

-
$8,237.02 $8,237.02


Moise. Sales

- -
1911
1/7 E . .. .. . ... ... ... . ... .. .

-
Allowance on 'Sales
1911
1/7 E ... . .. . . .... ... ... ... . $10.00."

15
Discount on Sales
1911 1911
3/7 E .. .. .. .. ... . . .. . .. .. . . $216.60% 3/7 E (contra). . . .. ... . .. ... $218.80/
3/15 E. . .. .. . .. . .. . .. .. .. . . .. 216.60

Interest

-
1911
3/15 E. .. ... ... .. .. .. .. . ... .. $4.52

Discount
-
1911
3/15 Notes Rec. disc. . . . . . . . . $30.00

Cash
1911 y
3/7 E . . . . . . . . . . . . . . . . . . . . . . $3,393.40
1911
3/7 E (ck. ret'd). . . . . . . . . . . . $3,393.40d ,
3/15 E. . . . . . . . . . . . . . . . . . . . . . . 1,400.42 E Fees . . . . . . . . . . . . . . . . . 2.50".
3/15 Notes Rec. disc. . . . . . . . . 1,970.00 6/15 Notes Rec. disc. . . . . . . . . 1,000.00
6/15 E. . . . . . . . . . . . . . . . . . . . . . . 800.00

Notes Receivable

1911 / 1911
-

3/15 E. . . . . . . . . . . . . . . . . . . . . . . $2,000.00 V 6/15 Notes Rec. disc. (G). . . . $1,000.00


6/15 E. Notes . . . . . . . . . . . . . . . 1,000.00

Notes Receivable Discounted

1911 1911
6/15 Notes Rec. . . . . . . . . . . . . . $1,000.00 Mch. 15 Cash and disc. . . . . . . . $2,000.00
44
Cash . . . . . . . . . . . . . . . . . . . 1,000.00 *-

Profit & Loss


1911
6/15 E. . .. .. .. . .. . ... ... . .. .. $200.00

Books of G

Mdse. Purchases

1911
1/7 Cash . . . . . . . . . . . . . . . . . . . $1,000.00

-
Cash

1911
1/7 Mdse. purchases. . . . . . . . . $1,000.00
6/15 E. . . . . . . . . . . . . . . . . . . . . . . 1,000.00

- E’s Account
1911 1911
6/15 Cash . . . . . . . . . . . . . . . . . . . $1,000.00 6/15 P. & L. . . . . . . . . . . . . . . . . . $1,000.00

Profit & Loss


1911
6/15 E. . . . . . . . . . . . . . . . . . . . . . . $1,000.00

16
NOTE:—It will be seen from the fore 1. Take a trial balance of account bal
going that the author bases his solution an Ces.
on certain assumptions. This is necessary 2. Take a trial balance of account totals.
on account of the lack of detail in the 3. The use of a reverse posting book.
statement of the problem. Self-balancing ledgers save time for
In the first place, it is assumed that E “credit-man” in determining daily balances
did not suffer the loss of 6% trade discount and provides a daily check on accuracy of
on sales, even though he made settlement posting; also facilitates drawing off trial
in 68,instead of conforming to the terms, balance.
60 days. Although from a legal point of
view he was not entitled to deduct the Ques. 7 and 8:—(a) Journalize the follow
$216.60 discount, actual business practice in ing for posting in a double entry ledger:
such cases would seem to overlook the Jan. 1, 1896. John Smith commenced
strictly legal aspect. Further, the instruc business with Cash on hand $3,000, Mdse.
tions “all accrued interest charges being $2,565, Store Furniture and Fixtures worth
calculated at the rate of 6%,” are assumed $600, Acct. against James Munson $57.80.
to refer to 6% interest from the due date, He owes William Cook on account $48.75.
March 7, 1911, to date of settlement, March Jan. 2. Bought of James Sharp Möse.
15, 1911, or interest on $3,393.40 for 8 days. amounting to $250, paid one-half cash, bal
It is assumed that the 3 months note for ance on account. Sold James Smith Mdse.
$2,000 was non-interest bearing. (See en amounting to $150, and took in payment
tries of March 15.) On June 15th, the 40% his note at 30 days, without interest.
settlement, $800, is given to D and not
divided between D and G, the assumption Jan. 3. Sold Peter Snell a bill of goods
amounting to $38; took in exchange 5 bar
being that G is a personal creditor of E
and is not entitled to share in the distribu rels of flour at $6 per barrel, balance on
tion of assets to business creditors. account.
A loss of $200 is shown on D's books, Jan. 4. Paid William Cook's sight draft
even though ordinarily he could hold G for in favor of Thomas Munson by check on
it as endorser of E's note. Whether G the First National Bank.
should be charged with this $200 depends Jan. 5. Charles Smith paid his note of
on whether the “compromise” mentioned the 2nd inst., having been allowed 1% dis
covered this point and was accepted by D. count on the same.

Ques. 6:—(a) How many different classes Jan. 6. Received of James Munson $57.30
of accounts are there? Give an example of in full of account, and gave him receipt for
the same.
each class. Show the use of each of your
examples. What would a Dr. balance upon (b) Post the Mdse. account of the above
each of your examples signify? A Cr. bal
it,
transactions, and close assuming that"
ance? Mdse. now hand worth $3,000.
in

is

(b) Mention three tests that may be ap


plied to test the correctness of a double ANSWER (a):
entry ledger. State the advantages of self January
1,

1916.
balancing ledgers. Cash $3,000.00
.
.
.
.
.
.
.
.
.
.
.
.
.
.
.
.
.
.
.

ANSWER:— (a) There are two main Mdse. Inventory. 2,565.00


classes of accounts, Real and Nominal.
&.
..
.
.
.
.
. .

Store Furniture Fixt. 600.00


In turn, real accounts are divided into Jas. Munson 57.30
sub-classes, assets and liabilities (Balance
.
.
.
.
.
.
.
.
.
.
.
.

- To John Smith $6,173.55


.
.
.
.
.
.
.
.
.
.
.
.
.

Sheet items). Wm. Cook 48.75


.
.
.
.
.
.
.
.
.
.
.
.
.
.

Similarly, nominal accounts are classified —2—


as expense or revenue (Profit and Loss Mclse. Purchases. $250.00
.
..
.
.
.
.
.

items). To James Sharp $125.00


Examples:
.
.
.
.
.
.
.
.
.
.
.
.

Cash 125.00
.
.
.
.
.
.
.
.
.
.
.
.
.
.
.
.
.
.
.
.
.
.

Real Accounts. Notes Rec. $150.00


.
.
.
.
.
.
.
.
.
.
.
.
.

Cash is an asset account; Accounts Pay To Moise. Sales. $150.00


.
.
.
.
.
.
.
.
.
.
.
.

~able, a liability.
A debit balance in “Cash” shows amount Mdse. Purchases. $30.00
.
.
.
.
.
..
.

on hand. Peter Snell 8.00


.
.
.
.
.
.
.
.
.
.
.
.
.

A credit balance in “Accounts Payable” To Moise. Sales. $38.00


.
.
.
.
.
.
.
.
.
.
.
.

shows amount which is owed to creditors.


Wm. Cook $48.75
.
.
.
.
.
.
.
.
.
.
.
.
.

Nominal Accounts. To Cash $48.75


.
.
.
.
.
.
.
.
.
.
.
.
.
.
.
.
.
.
.
.

Salaries is an expense account; Interest


(received) is a revenue account. Cash $148.50
.
.
.
.
.
.
.
.
.
.
.
.
.
.
.
.
.
.
.

A debit balance to “Salaries” denotes Disct. on Sales. 1.50


.
.
.
.
.
.
.
.
.

amount paid out for services. To Notes Rec. $150.00


.
.
.
.
.
.
.
.
.
.
.
.
.
.

A credit balance to “Interest” indicates —6—


amount of money received as consideration Cash $57.30
.
.
.
.
.
.
.
.
.
.
.
.
.
.
.
.
.
.
.

for loans. To Jas. Munson. $57.30


.
.
.
.
.
.
.
.
.
.
.
.

(b) Three tests that may be applied to Mdse. Inventory


test the correctness of a double entry (new aſc) $3,000.00
.
.
.
.
.
.
.
.
.
.
.
.

ledger are as follows: To Mdse. Inventory (old a/c) $3,000.00

17
(b) Mdse. Inventory
1896 1896
Jan. 1 J. Smith . . . . . . . . . . . . . . $2,565.00 Jan. 6 Inventory . . . . . . . . . . . . $3,000.00
6 Trading (bal.)......... 435.00

$3,000.00 $3,000.00

Jan. 6 Inventory . . . . . . . . . . . . $3,000.00

Moise. Purchases

1896 1896
Jan. 2 Sharp . . . . . . . . . . . . . . . . $125.00 Jan. 6 Trading (bal.)......... $280.00
2 Cash . . . . . . . . . . . . . . . . . 125.00
3 Snell . . . . . . . . . . . . . . . . . 30.00

$280.00 $280.00

Moise. Sales

1896 1896 .
Jan. 6 Trading (bal.)......... $188.00 Jan. 2 Notes Rec. . . . . . . . . . . . $150.00
3 Moise. Purchases. ... . . . 30.00
3 Snell . . . . . . . . . . . . . . . . . 8.00

$188.00 $188.00

Discount on Sales
1896 -
1896
Jan. 6 Notes Rec. . . . . . . . . . . . $1.50 Jan. 6 Trading (bal.)..... . . .. $1.50

Mdse. Trading Account


1896 1896
Jan. 6 Mdse. Purchases....... $280.00 Jan. 6 Mdse. Sales . . . . . . . . . . $188.00
6 Disc. on Sales. . . . . . . . 1.50 6 Mdse. Inv. (dif.). . . . . . 435.00
6 Bal. (Profit) . . . . . . . . . 341.50

$623.00 $623.00

NOTE:—Theproblem calls for a “merchandise account.” The above solution is


in conformity with up-to-date practice, as very few businesses keep a single merchan
dise account. The contents of the above five accounts can very easily be embodied
in one merchandise account if the student so desires.

Ques. 9 and 10:—

H. T. Herrick Samuel Mabon


$7,381.00 | $13,246.00 $5,369.00 $12,863.00

Assets Liabilities.
Cash . . . . . . . . . . . . . . . . . . . . . . . . . $3,125.25 Bills Payable . . . . . . . . . . . . . . . . . $2,575.00
Mdse. . . . . . . . . . . . . . . . . . . . . . . . . 4,222.84 Int. Payable . . . . . . . . . . . . . . . . . . 172.49
Bills Receivable . . . . . . . . . . . . . . 375.00 Accts. Payable . . . . . . . . . . . . . . . 310,71
Accts. Receivable . . . . . . . . . . . . . 1,784.67 Mortgage Payable . . . . . . . . . . . . 2,000.00
Furniture & Fixtures. . . . . . . . . . 1,596.00 Int. Payable . . . . . . . . . . . . . . . . . . 65.18
Real Estate . . . . . . . . . . . . . . . . . . 4,500.00
Horse & Wagon. . . . . . . . . . . . . . 520.00
Merchandise Expense
Bought $6,128.07 | Sales $62,324.33 $2,148.94 |

18
You are given the above statement of ac SOLUTION:

-
counts in order to find the net gain or net Paid July 30. $1.00
loss, each partner's net capital after adjust

.
.
.
.
.
.
.
.
.
.
.
.
.
.
.
.
.
.
.
Due mos. on June 24. .06

3
Inent, and the correct amount of sales. The

.
.
.
.
.
.
.
.
.
.
losses and gains are shared equally by each Int. mo., days. 12) .0600

6
partner.

.
.
.
.
.
.
.
.
.
.
.
Give all the work.
.005
ANSWER: 1.2
Liabilities . . . . . . . . . . . . . . . . . . . . . . $5,123.38v
Capital— 2300 10
Herrick . . . . . . . . . . . . $5,865.00

5
Mabon . . . . . . . . . . . . . 7,494.00 13,359.00,
1.006)2313.800
2012xxx .0060
Total Liabilities and Capital. . . . . $18,482.38
Total Assets . . . . . . . . . . . . . . . . . . . 16,123.76

of
3018 $2,313.80 Amount check
3018 2,300.00 Amount of bill
Net Loss . . . . . . . . . . . . . . . . . . . . . . $2,358.62
Less Expense . . . . . . . . . . . . . . . . . . 2,148.94 00 $13.80 Interest (Ans.)
Ques. 2:—A drover borrowed $4,800

at at
Selling Loss .. . ... .. ... ... . $209.68 6%, and invested the sum live stock

in
Moise. Bought . . . . . . . . . . $6,128.07
an average price $50 per head. Fifteen

at of
Mdse. Inventory . . . . . . . 4,222.84 days later he sold an average price

of
$55
per head, and loaned the proceeds for 90
Cost of Goods Sold. . . . .. . .. . ... 1,905.23
days Ten days after the loan was

at
7%%.
paid he returned the sum borrowed with

- -
Net Sales . .. .. . .. . .. . .. . .. . $1,695.55
interest and deposited the balance. How

-
much was the deposit?

--
Proof. SOLUTION:

- -
Trial Balance. $4800 96 96
Account Dr. Cr. .06 55
H. T. Herrick, Capital...

-
$7,381.00 $13,246.00 50)4800
Samuel Mabon, Capital... 5,369.00 12,863.00 12)28800 450 480

-
Cash . . . . . . . . . . . . . . . . . . . 3,125.25 480
Bills Receivable . . . . . . . . 375.00 24. 300
Accounts Receivable..... 1,784.67 3.8 1/3 300 5280
Furniture & Fixtures. ... 1,596.00 .07%
Real Estate . . . . . . . . . . . . 4,500.00
8

Horse & Wagon. . . . . . . . 520.00 192 2640


Bills Payable . . . . . . . . . . . 2,575.00 72 36960
Int. Payable . . . . . . . . . . . . 172.49
Accounts Payable . . . . . . 310.71 92.0 12)396.00
Mortgage Payable. . . . . . . 2,000.00 4800. $5379
Int. Payable . . . . . . . . . . . . 65.18 48.92 33
Expense . . . . . . . . . . . . . . . . 2,148.94 $4892.
3

Mdse. (Bought) . . . . . . . . 6,128.07 $487 (Ans.)


Deposited. 99
$31,232.38 5280
Difference (Mdse. Sold). 1,695.55
$5379
Total . . . . . . . . . . . . $32,927.93 $32,927.93 3:—Find the average date
of

Ques. the
H. T. Herrick Sam'1 Mabon
following accounts, and prove the work:
Investment . . . . . $13,246.00 $12,863.00 1886—May 10, $450
Withdrawals .... 7,381.00 5,369.00 May 31, 375
July
3,

700
Sept. 20, 938
Net Investment... $5,865.00 $7,494.00
Less % Net Loss 1,179.31 1,179.31 SOLUTION:
Interest
0 at

6%
Net Capital...... $4,685.69 $6,314.69 May 10, $450X 0=
0

May 31, 375).{ 21= 7,875 1.31


July 700X 54= 37,800 6.30
3,

ARITH METIC–Weight 3. Sept. 20, 938X133=124,754 20.79

Give all the figuring on the ruled sheets. $2463 170,429 $28.40
days from May 10th,
or

2,463)170,429
+

(To be finished by 4 P. M.)


147,78
(69
July 18th (Ans.).
Ques. 1:—March 24, 1887, a wholesale
merchant sold goods on three months' cred 22,649
The bill was paid July
30

with interest 22,167


at it.

by

6%, check for $2,313.80, How much


was the interest? 482

19
Proof. necessary entries involving returns, dis
counts, notes, transfers or other closings
Interest on $1.00 for 170,429 days of accounts.
Interest on $170,429 for 1 day =$28.40 His duties also include the proper post
Interest on $2,463 for 69-H days ing of transactions from books of original
A slightly different method of working entry to the ledger or ledgers, the abstrac
cut a similar problem, with explanation of tion of trial balances and the preparation
the theory of averaging accounts, is given of statements of accounts.
elsewhere in this book. At the end of the fiscal period, he may
also be called upon to express on the books
Ques. 4:—Borrowed a sum of money De the result of operations, by closing the
cember 15, 1886, at 4%% and on the same nominal accounts and bringing down bal
date loaned it at 6%. June 3, 1887, I made ances with which to start the new period.
a settlement with all the parties concerned
The preparation of statements of profit
and found I had gained $48.50. What and loss and of financial condition are fre
amount did I receive from the person to quently within the scope of the duties of
whom I loaned the money?
a bookkeeper, particularly where no public
SOLUTION: accountant is employed for that purpose.
December 15, 1886, to June 3, 1887-170 days. Ques. 2:—Explain how bad and doubtful
.06 .015 debts are treated in a set of books.
.045 170 ANS.:–Bad and doubtful debts are treat
ed in a set of books in the following man
.015 1050 ner: -
15 Usually, based on past experience, an
estimate is made of the amount of accounts
365)2.550(.007 receivable outstanding which may in the end
2555 prove either partially or totally uncollect
ible. Such a sum is reserved from distribu
.007) $48.50 tion as profits or surplus, by a charge to
the latter account and the creation of a
$6928.57)&(1.06X170/365) =$7122.19 credit account entitled “Reserve for Bad
Ans.) and Doubtful Debts.” The accounts receiv
able are allowed to remain on the books at
full original value, but this reserve account
Municipal Civil Service Commission, is treated as a deduction from the outstand
N. Y. ing receivables on the Balance Sheet. When
BOOKKEEPER, 3rd GRADE. a customer's account proves worthless,
either in part or in whole, it is credited
(Salary, $1,200 to $1,800.) for the loss sustained and the reserve ac
count debited with a like amount. Thus,
Date: March 16, 1909.
the net credit balance in the reserve ac
TECHNICAL. count will represent the amount withheld
from distribution, in anticipation of future
(Handwriting and Neatness will be rated due to bad and doubtful accounts.
on Technical paper.)
Ques. 1:—What are the duties of a book
keeper on taking charge of a set of books
V. Ques. 3:—Explain the method and show
by example how you would reconcile a
in an ordinary mercantile business? check book balance with a balance in bank
ANS.:—On taking charge of a set of pass book. -
books in an ordinary mercantile establish ANS.:—A check book balance is recon
ment, it is a bookkeeper's first duty to ciled with balance in bank pass book in
ascertain that the books have been kept ac the following manner:
cording to modern methods. If not, he The canceled checks returned by bank
should call attention to the proprietor of are compared with the stubs in check book
any deficiencies in methods of the past and to ascertain what checks drawn during the
suggest that steps be taken to institute im period have not as yet been presented to
provements. the bank for payment. These are listed
For his own protection, the new book and totaled. The sum of outstanding
keeper should immediately make count of checks is added to check book balance and
cash on hand and compare with balance to this is then added any interest which
shown in books. Having ascertained, fur the bank allows on average balance, and
ther, that the books are in balance, if a deduction made for exchange or other
double entry, he should proceed to make charges. The result should then agree with
necessary entries in the Sales Book of all the bank balance, if the bank pass book or
goods shipped to customers, in the Pur statement and the check book entries and
chase Book (if such a record is kept) of footings have been made accurately. If an
all purchases from creditors, all cash re error exists, it can be located by checking
ceived and paid out, as well as any other entries and footings. '

20
Example of Reconciliation. (b) Makejournal entry for following:
Balance as per Check Book. . . . . . . . $1,000.00 T. J. Allen failed, and assigned for benefit
Add checks outstanding: of creditors. We have agreed to settle our
Amount claim of $3,284.24 at 25 cents on the dollar:
No.
123. . . . .. .. . .. . . . . . . .. . $100.00
one-half in cash, and balance by six months’
156. . . ... . . . .. . . . . . . ... 65.00 note bearing interest at 5 per cent.
157. . . . . .. . .. . . . . . . ... 35.UU ANSWER: .
158. . . .. ... . .. . . . . . . .. . 100.00
(a) Real Estate.............. $5,670
160. . . ... .. . .. . . . . . . . .. 250.00 550.00
Mtge. on Real Estate. . . . . . . . . $1,500

$1,550.00
Accrued Mtge. Interest........ 70
John Williams, Builder........ 600
Add interest credited by bank..... 5.00
Cash . . .. .. . . . .. . .. . .. . . .. . .. . 3,500

$1,555.00
exchange and other charges 3.00
Purchase of real estate from H. Jones
Deduct subject to liens amounting to $2,170, as
above.
Balance as per Bank Book. . . . . . . . $1,552.00 (b) Cash . . . . . . . . . . . . . . . . . $410.53
Ques. 4:—Where there are several ledg Notes Receivable . . . . . . . . . 410.53
ers in a business, how would you arrange so Profit & Loss (or Reserve
that an error could be easily located in a for Bad Debts). . . . . . . . . 2,463.18 -
particular ledger? Explain fully. To T. J. Allen................. $3,284.24
ANS.:—Where several ledgers are kept Settlement of claim at 25 cents on the
in a business, an error in any particular dollar, one-half in cash and balance by six
ledger may be easily located by means of months' note bearing interest at 5 per cent.
controlling accounts.
A controlling account is one which ex Ques. 6:—J. Pratt and W. Barnes have
hibits a summary of all of the accounts in been in partnership for a year on equal foot
a ledger or of all accounts of the same ing. Total capital at beginning was $1,185.
class. The two principal controlling ac The books have been kept by single entry.
counts used in a trading concern are called The assets and liabilities at end of year
“Sales (or Customers') Controlling Ac were:
count” and “Purchase (or Creditors') Con Mdse. . . . . . . . . . .................... $1,000
trolling Account.” ................
Cash . . . . . . . . . . . . . . 285
An illustration of one of these, Sales, Building and Fixtures.............. 1,200
will suffice for purposes of explanation. Customers' Accts. ................. 1,100
In the first place, all goods sold to cus Creditors' Accts. .................. 1,700
tomers are entered in a book used exclu J. Pratt Capital.................... 585
sively for such transactions—a Sales Book. W. Barnes Capital. . . . . . . . . . . . . . . . . 600
These sales are posted to the debit side of . Find the Profit and Loss and show the
the individual customers' accounts in a journal entry required to open a new set
ledger containing customers' accounts sole of double entry books.
ly. At the end of the month the total sales
are posted to the debit side of the Sales ANSWER: -
Controlling Account, so that the sum of Statement of Profit.
the debits in the individual's ledger will
Assets.
agree with the one debit item in the Con
trolling Account of the general ledger. The Mdse. Inventory ............. $1,000

credits are handled similarly.


.
Cash . . . . . . . . . . . . . . . . . . . . . . 285
Building and Fixtures........ 1,200
All payments by customers are entered ............
Customers' Acct.
in a special column provided for that pur
1,100
pose in the Cash Book, whence postings
are made to the accounts in the customers' Total Assets ................. $3,585
ledger. - Liabilities, etc.
The total cash received during the month Creditors' Accts. .................. 1,700
is posted to the credit side of the con
trolling account. Whatever journal entries Capital - - - - - - - - - - - - - - - - - - - - - - - - - - - $1,885
may be necessary are made in special col Less Investment .................. 1,185

umns provided for the separate ledgers, so year................


that a total posting can be made at the end Profit for $700
of the month to the controlling accounts. Journal Entry.
An abstract of balances from the custom Mdse. Inv. . . . . . . . . . . . . . . . . . . $1,000
ers' ledger when summed up should agree .
Cash . . . . . . . . . . . . . . . . . . . . . . 285
with the net debit balance shown in the Buildings and Fixtures....... 1,200
Controlling Account. - Customers' Accts. Rec....... 1,100

Ques. 5:—(a) You bought a house and lot - $3,585


from H. Jones, paid cash $3,500 and assumed To Creditors' Accts. Payable. . . . . $1,700
a mortgage for $1,500, with $70 accrued J. Pratt, Capital............... 935
interest, and a claim of $600 held by John W. Barnes Capital............ 950
Williams, the builder. Make the journal
entries on Jones' books. $3,585

21
To express on books, in double entry Division of Net Profit:
form, statement of condition of Pratt &
James Allen 3/5. . . . . . . . . . . . . . . $1,478.70
Barnes as at January 1, 1909, books here Wm. Butler 2/5. . . .. . .. . ... ... 985.80
after to be kept by double entry method.

Ques. 7 and 8:—James Allen and William


Butler entered into partnership on January Partners' Capital Accounts.
1, 1908, Allen investing $3,000 and Butler James Allen Wm. Butler
$2,000. Profits to be divided in proportion Investment Jan. 1,
to capital brought in. Interest to be charged 1908 . . . . . . . . . . . . $3,000.00 $2,000.00
at 6 per cent on amounts withdrawn by Less:
each. The Trial Balance on December 31,
1908, was as follows:
Withdrawn . . $600 $500.00
Int, on....... 27 627.00 17.50 517.50
Machinery .................. $1,600
Furniture . . . . . . . . . . . . . . . . . . . 200 $2,373.00 $1,482.50
Horse & Wagon. . . . . . . . . . . . . 300 Add share of profit 1,478.70 985.80
Goods purchased............. 6,000
Sales .. . .. .. . .. .. . . . .. . .. .. . $8,500 Capital Dec. 31,
Rent . . . . . . . . . . . . . . . . . . . . . . . . 250 1908 . . . . . . . . . . . . $3,851.70 $2,468.30
Expense . . . . . . . . . . . . . . . . . . . . 130
Freight ... . . . . . . . . . . . . . . . . . . 120
Discounts . . . . . . . . . . . . . . . . . . . 40 Balance Sheet
Interest and Com. . . . . . . . . . . . 30 Allen & Butler
Printing and Stationery...... 25
Traveling Expense. . . . . . . . . . . 200 Jan. 1, 1909.
Salaries .. . . .... ... .. .. . .... . 800 Assets.
Discounts Received.......... 80
James Allen Capital. . . . . . . . . . 3,000 Customers' Accts. Rec. . . . . . . $3,685
Wm. Butler Capital. . . . . . . . . . 2,000 Mdse. Inventory . . . . . . . . . . . . 1,435
Jas. Allen drew out April 1... 600 Machinery . . . . . . . . . . . . . . . . . . 1,600
Wm. Butler drew out May 31. 500 Furniture . . . . . . . . . . . . . . . . . . . 200
Customers' Accts. . . . . . . . . . . . 3,685 Horse & Wagon. . . . . . . . . . . . . 300
Creditors’ Accts. . . . . . . . . . . . . 900
Total Assets . . . . . . . . . . . . . . . . . $7,220.00
$14,480 $14,480

-Inventory Liabilities.
of stock December 31, 1908,
was $1,435. Prepare a Profit and Loss ac Creditors' Accts. Payable. . ... .. .. 900.00
count and a Balance Sheet.
$6,320.00
ANSWER:
Statement of Profit and Loss.
Allen & Butler Capital.
James Allen . . . . . . . . . . . . . . . . . . . . . . $3,851.70
Jan. 1 to Dec. 31, 1908. Wm. Butler . . . . . . . . . . . . . . . . . . . . . . 2,468.30
Sales . . . . . . . . . . . . . . . . . . . . . $8,500.00
Cost of Sales:
Goods Purchased. . $6,000
Less Inv. Dec. 31
1908 . . . . . . . . . . . . 1,435 4,565.00

Gross Profit . . . . . . . . . . . . . . . . . . . . . $3,935.00 ARITH METIC.


Salaries . . . . . . . . . . . . . . . . . . $800,00
Rent .. ... ... ... .. . ... .... 250.00
(Give all the figuring on the ruled sheets.)
Expense . . . . . . . . .. . . . . . . . . 130.00
Ques. 1:—A note dated May 3, 1908, pay
Freight . . . . . . . . . . . . . . . . . . . 120.00 able in 8 months and drawing 5 per cent
Interest and Commission. . 30.00 interest, was discounted in bank September
Printing and Stationery... 25.00 27, 1908, at 4% per cent, the proceeds being
Traveling Expenses. . . . . . . 200.00 $2,040.39. What was the face of the note?
Discount paid . . . . . . . . . . . . 40.00
SOLUTION:–8 mos. after May 3, 1908–
Jan. 3, 1909, due date of note with 5% in
$1,595.00 terest.
Less Discount Received... 80.00 1,515.00
Sept. 27, 1908, to Jan. 3, 1909–98 days,
Business Profit . . . . . . . . . . . . . . . . . . . $2,420.00 period of discount.
Add Interest on partners' with Face of note-H8 mos. interest at 5%—dis
drawals: count for 98 days at 4%% per annum=
James Allen . . .. .... . . $27.00 $2,040.39(proceeds).
Wm. Butler .. ... . .. . . 17.50 44.50 Effective rate at 4%% per annum is
1.22%% for 98 days (360 days in year).
Net Profit........ $2,464.50 100%—1.22%%=98.775% (ratio of proceeds
to amount discounted).
22
$2,040.39–5–98.775X100-face of note-H5% Interest at 6% for 10 days. Interest for
interest=$2,065.70. 60 days $118.46.
Effective rate at 5% per annum is 3 1/3% 1/6 of $118.46=$19,74, interest for 10 days.
for 8 months. $11,846+$19.74=$11,865.74, proceeds-Hin
terest.
100
$2,065.70X
3.6%)($11,865.74=$44.50, amount of dis
= $1,999.10, face of note COunt.
103.5 (Ans.) $11,865.74+$44.50=$11,910.24, amount of
Formula: 10-day draft (Ans.).
100 100 NOTE:—Business men and bankers usu
$2,040.39× X = $1,999.10 (Ans.) ally calculate the discount on drafts and
98.775 103.5 subtract or add to the value as the case
requires. In the above example the 3%%
Ques. 2:—A bought a bill of goods for
$800 and marked them so as to gain 40 per
was figured on the amount of net proceeds
and interest, $11,865.74, to conform with
cent. After selling three-fifths of them at
this price, 50 per cent of the remainder was
business practice. While the above is suffi
ciently accurate for ordinary uses, to be
sold at 25 per cent discount from the strictly accurate it would be necessary to
marked price and the balance was lost by
fire. What amount and what per cent did divide by the rate of exchange .995%. The
necessary correction can be made by adding
he gain or lose?
to the answer given above, 3%% of $44.50
SOLUTION: amount of discount, or about 17 cents, mak
Por- ing the result $11,910.41.
40% Marked Dis- Selling
tion Cost Added Price count Price
3/5 $480 $192 $672 None $672
1/5 160 64 224 25% 168
1/5 160 64 224 Fire 0
Municipal Civil Service Commission, N. Y.
All $800 $320 $1120 $840 BOOKKEEPER, 3rd GRADE.
Selling Price. . .
Cost . . . . . . . . . . ... ...
.. ..
$840
800
(Salary, $1,200 to $1,800.)

Date: March 16, 1909.


Amount gained...... $40
TECHNICAL.
$40+$800-5% gained (Ans.)
HANDWRITING AND NEATNESS will
Ques. 3:—18 months ago a man invested
at 4% per cent compound interest (com be considered on this paper.
pounded semi-annually), a certain sum Ques. 1:—Define a Balance Sheet. Ex
which now amounts to $4,185. What was plain how and why it is prepared and the
the amount invested?
source of the information thereon.
SOLUTION: ANS.:—A Balance
Sheet is a statement
showing the various debit and credit bal
4%% for 1 year=2%% for 94 year.
ances brought down after closing all ledger
2%%=.02125.
Principal (1.00) + Interest (.02125)=1.02125, accounts. It is prepared by merely abstract
ing these balances and classifying them as
R at 10.
assets, liabilities, capital, etc. The object
Period Multiplication A mount
of preparing a Balance Sheet is to show the
6 mos. 1.00 X 1.02125 = 1.02125 financial condition of the business. The
12 mos. 1.02125 X 1.02125 = 1.04295 source of information contained in the Bal
18 mos. 1.04295 X 1.02125 = 1,06511
ance Sheet is the general ledger of the busi
$4,185––1.06511=$3,929.17, amount invested neSS.
18 months ago (Ans.).

Ques. 4:—A commission merchant sold a Ques. 2:—Explain the meaning of the fol
consignment of olive oil at 4 per cent com lowing terms: (a) Scrip; (b) Accommoda
mission for $12,600. His charges other than tion Note; (c) Mortgage; (d) Annuity.
commission amounted to $250. How large ANS.:—(a) Scrip is a term applied to a
a 10-day draft can he buy with the proceeds receipt given for installment on stock sub
to remit to the consignor if exchange is at scription. The Scrip is exchanged for a
% per cent discount and interest 6 per cent? formal certificate when all installments have
been paid.
SOLUTION: (b) Accommodation Note—A note which
proceeds . . . . . . . . . . . . . . . . . . .
Gross $12,600 is given without consideration, to enable
Commission at 4%. . . . . . . . . . . . $504 the payee to raise money.
Charges . . . . . . . . . . . . . . . . . . . . . . 250 (c) Mortgage—A temporary transfer of
title to land, goods or chattels to secure
Total Deductions.................. 754 payment of a debt.
(d) Annuity—An amount payable to or
Net Proceeds. . . . . . . . . . . . . . . . . $11,846 received from another each year for a term
of years or for life.

23
Ques. 3:—Explain the modern method of Ques. 5:—Make entries in journal form
separating merchandise account into parts, to represent the following: (1) A. H. Jones
showing what each subdivision covers and began business investing $3,500; merchan
the advantage of the whole plan. dise, stocks and bonds $3,200, and a note
ANS.:—The merchandise account as kept of Wm. Avery for $500 on which is due $25
nowadays is usually divided into four parts: interest; and Jones owes on his note held
Inventory, Purchases, Sales and Trading ac by Wm. Smith $1,000 plus accrued interest,
COunts. $87. (2) He discounts Avery's note, which
Merchandise Inventory account shows the

:*
netted him $505. (3) Wm. Avery's note was
on hand at the beginning of the pe dishonored at maturity; interest, $50, pro
rio Gl. test fee 75 cents.
Merchandise Purchases account shows on
ANSWER (1):
the debit side the gross purchases; on the . Dr. Cr.
credit side, purchases returned, trade dis
Mdse. Inventory. . . . . . . . . . $3,500.00
counts and allowances. The net debit rep
resents net purchases. Investments (stocks
Merchandise Sales account shows on the bonds)
Notes Receivable. . . . . . . . . .
credit side the gross sales; on the debit
side, sales returned, trade discounts and Accrued Interest Rec. . . . . .
allowances. The net credit represents net Notes Payable
sales. Accrued Interest Payable. . . . . .
Trading account is the account to which A. H. Jones—Capital Acct..... 6,138.00
Began business with assets and liabilities
the balances of the first three accounts are -
as above.
transferred. To this account then is cred
ited the amount of Merchandise Inventory
at the close of the period. The difference (2)
Cash . . . . . . . . . . . . . . . . . . . . . $505.00
between the debit and credit sides then rep
resents gross profit or loss on merchandise. Notes Receivable Discounted. . $500.00
Interest .. .... ... . ... . . .. . .. .. 5.00
The advantages of this plan are: It fur
nishes an analysis of the old style merchan Discounted Wm. Avery's note.
dise accounts; it prevents the confounding
of the different elements; it shows clearly (3)
the amount of sales, purchases, etc.; it en Notes Receivable Disc. . . . $500.00
ables one to readily calculate percentage Wm. Avery . .. .. .... . .. .. 50.75
of profit on cost and sales. Cash (75c Protest Fees)....... $500.75
Ques. 4:—It is sometimes desirable to find Accrued Interest Receivable... 25.00
the approximate value of a stock of goods Interest Account . . . . . . . . . . . . . 25.00
without actually taking an inventory. Do . Note of Wm. Avery returned unpaid with
this from such of the following facts as are interest of $50, plus 75 cents protest charges.
necessary: Goods on hand January 1, $24,
000; Purchases, $8,000; Sales, $17,500; Re Ques. 6:—Small and King, who are part
turn sales, $800; Cash discount on goods ners on equal terms, agree to dissolve part
sold, $72; Cash discount on goods pur
nership. The following is the trial balance
chased, $160. of their books:
The firm estimates that it
Customers' Accounts . . . . . $7,000
makes a profit of 16 per cent on the gross Creditors' Accounts . . . . . . . $8,000
amount of each bill and goods sold. Mach. and Plant. . . . . . . . . . 3,000
ANSWER: Merch. on hand. . . . . . . . . . . 4,000.
Sales . . . . . . . . . . . . . . . . . . $17,500.00
Bills Payable . . . . . . . . . . . . . 1,000
Returns ... . . . . . . $800 Profit & Loss Acct........ 1,500
Sales disc. . . . . . . . 72 872.00 $16,628.00
Small Capital Acct. . . . . . . . 2,000
King Capital Acct. . . . . . . . . 1,500
Cost of goods sold—
Inventory, Jan. 1. . . . . $24,000.00 $14,000 $14,000
Purchases 8,000.00
King is to take over the business at above
$32,000.00 figures less 10 per cent off Customers’ Ac
Purch. disc. ... . . . .. . 160.00 counts for bad debts. Show how this ar
rangement will be carried out and how
$31,840.00 much King will have to pay Smith.
Deduct Inventory at ANSWER:
end of period...... 17,872.48 Journal entry creating reserve of 10 per
cent on Accounts Receivable:
Cost of goods sold (84% of Profit & Loss. . . . . . . . . . . . . $700
sales) . . . . . . . . . . . . . . . . . . . . . . 13,967.52 To Reserve for Bad Debts. . . . $700
being 10% on $7,000, total of Customers'
Gross profit, 16%. . . . . . . . . . . . . $2,660.48 Accounts, reserved for bad debts by agree
ment of Small and King.
Inventory plus Net Purchases. . . $31,840.00
Less materials consumed (84% Journal entry transferring net profit to
on Sales) -- ------ ---------- 13,967.52 partners' capital accounts:
Profit & Loss. . . . . . . . . . . . . $800
Inventory at end of period. . . . . . $17,872.48 To Small's Capital Acct....... $400
King's Capital Acct........ 400

24
Balance Sheet.
Assets. Liabilities.
Customers’ Accounts.... $7,000 Creditors' Accounts. . ... $8,000
Less 10% reserve. . . . . . 700 $6,300 Bills Payable . . . . . . . . . .. 1,000 $9,000

Machinery and Plant. . . .. . ... ... 3,000 Capital:


Merchandise on hand. . . . . . . . . . . 4,000 Small—Investment ... $2,000
Profit ...... .. . 400 2,400
-
King—Investment .. .. $1,500
Profit . . . . . . .. . 400 1,900

$13,300 $13,300

King would have to pay Small $2,400 for his interest in the business.

Ques. 7 and 8:—The following is the trial


balance of Jones & Cannon, who are equally
interested in the business, as of December
31, 1908:

Stock, Jan. 1, 1908. . . . . . . . . $8,400 Balance Sheet, Jan. 1, 1909.


Goods purchased . . . . . .‘. . . 40,400
Assets.
Goods sold . . . . . . . . . . . . . . . $51,600
Discount . . . . . . . . . . . . . . . . . 700 Cash .. .. ... .. . ... ... .. .. . $1,400
Rent and Taxes. . . . . . . . . . . 1,900 Accounts Receivable . . . .. . 5,400
Wages . . . . . . . . . . . . . . . . . . . 2,400 Bills Receivable . . . . . . . . . . 2,400
Salaries . . . . . . . . . . . . . . . . . . 900 Inv. of Mdse. . . . . . . . . . . . . 8,500
Expenses . . . . . . . . . . . . . . . . . 650
Stationery and Printing.... 500 $17,700
Bills Payable . . . . .. . .. . . .. 3,600
Bills Receivable . .. . ... .. . 2,400
Creditors’ Accts. .. . ... ... 4,600 Liabilities.
Customers’ Accts. . . .. . ... 5,400
Accounts Payable . . . . . . . . $4,600
Cash . . . . . . . . . . . . . . . .. . .. . 1,400
Bills Payable . . . . . . . . . . . . 3,600
Jones' Capital . . .. . .. ... . . 1,900
Cannon's Capital ... . .. ... 1,950 Total Liabilities ... .. . . .. .. . .. . .. . $8,200

$64,350 $64,350 Capital.


-
Stocks on hand December 31, 1908, was Jones'— -
$8,500. Prepare a Profit and Loss Account Balance Jan. 1, 1908. . . . . $1,900
and a balance sheet. Add 2 P. & L. Acct. . . . 2,825 4,725

ANSWER: Cannon's—
Income and Profit and Loss Statement Balance Jan. 1, 1908. . .. . $1,950

Jan. 1, 1908, to Dec. 31, 1908. Add W. P. & L. Acct. .. . 2,825 4,775

Sales . . .. .. ... . .. ... . .... ... . .. . . $51,600 $17,700


Cost of Goods Sold:
Inv. of Mdse. Jan. 1, 1908 $8,400
Purchases ... ... ... . . . .. 40,400 ARITH METIC.
a on
all

$48,800 (Give the figuring the ruled sheets.)


Deduct Purchase Discts. What sum would man receive who
1.

700
discounted on July 16, 1908, per cent,
at
8

$48,100 note for $5,000, dated June 16, 1908, pay


a

Deduct Inv. of M dse. able 90 days, with per cent interest?


A in

8,500 dealer bought 4,000 bushels


% of

Dec. 31, 1908. . . . . . . . . . wheat.


2.

-
He sold 1/5 per cent gain;
at

at

10
11 at % 5

Cost of Goods Sold . . 39,600 per cent gain; 12 per cent gain, and
16 at

.. . . .. . .. . .. .
the remainder per cent gain. he
If

Gross Profit . . . . . . . . . . . . . . . . . . . . . $12,000 had sold all per cent profit he would
at

Expenses: have made $35 more. What did the wheat


Wages . . . . . . . . . . . . . . . . . $2,400 cost him per bushel?
Salaries . . . . . . . . . . . . . . . . provide for the pay
to

900 man wishes


3.

Rent and Taxes. . . . . . . . . debt of $715 fall due


to
of

ment two
in

1,900
a

Stationery and Printing. 500 years from now. What amount should be
Miscellaneous put 4% per cent, same
to

.. . .. ... . . 650 interest


at

at

be
compounded semi-annually?

25
4. An agent sold 40 casks of wine, each
indebtedness, $100,000 (of which there are
containing 36 gallons, at $1.50 a gallon, and outstanding $97,000); sinking fund account,
$2,624; U. bonds (held as

S.
after deducting his commission of 3 per Government
reserve account), $10,000;

of
cent remitted the proceeds by a sight draft investment
purchased at 9% per cent premium. merchandise, $68,492; dividends accrue
What
was the face of draft? (payable), $1,000; undivided profits, $2,573;
real estate, $194,000; N. City bonds (pur

Y.
chased for sinking fund account), $2,500;
taxes arrears, $760.

in
Municipal Civil Service Commission, N. Y. Roe, partners, who have
and Doe

of &
6.
5
BOOKKEEPER, 3rd GRADE.
kept the books their firm by single en
try, wish admit Jones third partner

to

as
a
(Salary, have their books thereafter kept by

to
$1,200 to $1,800.) and
double entry. From their accounts they
Date: July 11, 1913. find the following resources and liabilities:
TECHNICAL–Weight 5. Mdse. per inventory, $9,241; cash, $850; real
estate, $3,000; bills payable, $975; Doe's
HANDWRITING AND NEATNESS— credit, $5,390; Roe's credit, $6,400. They
owe $4,175 (personal accounts); persons
Weight 2. owe them $6,941; store fixtures are worth
What was the gain Find

or
$571. loss?
(To be finished by 2 P. M.) each partner's net capital, assuming that
they share losses and gains equally.
(NOTE:—This examination was cancelled
Jones admitted partnership and in

to
is
on the ground that some of the questions
vests $3,000 cash, moise. $2,000, bills receiv
were too difficult for the grade.)
able $1,500.
1. (a) Why is it expedient to keep the
Make the entry open the books the

to

of
accounts for cash in a separate book from
new firm by double entry, and admit the
the other ledger accounts?
new partner by using one entry.
(b) How is net loss or profit arrived at
from a set of books? From the following debit and credit
7.

(c) How would you deal with an over items given below make out loss and gain

a
charge where you had entered amount of account and balance account: Debit items:
a
Cash, $1,334; mase., $6,230; bills receivable,
bill rendered your firm before overcharge Osborn, $130;
$426; bills payable, $440;

J.
was discovered?
expense, $14. Credit items: West,

B.

L.
(d) How would you deal with such an
overcharge where the same was discovered $4,000; cash, $494; mase. sales, $1,624; bills
receivable, $216; bills payable, $940; Os

J.
before the entry was made?
born, $50; H. Clum, $1,250. There

is
R.

(e) State what you know regarding mod


mdse. on hand, per inventory,
as

$4,700.
crn mechanical methods of charging a bill
of goods on the books while at the same West the proprietor.
is

time making out the statement. and are partners business. At in


A
8.

2 and 3. Journalize the following: the close the statement of their accounts
Sold md se. to J. P. King amounting to follows: Resources, mdse., $13,800;
as

are
$369.84. Received in payment my own note cash, $4,200; First National Bank stock,
in favor of F. May for $179.20, interest ac $2,175; liabilities, Brown Co., $8,432; Hub
&

crued thereon $4.09, and a certificate of de bard Bro., $4,435; mortgage payable, $8,
&

posit to balance account. 050; bills payable, $3,500; net gain, $11,758,
Bought of J. N. Howard md se. as per shared equally. A's proportion in
of

the
solvency
58

invoice, $1,050. Gave in payment J. Smith's and B's 36. What was each
is

note for $450 at 2% per cent discount draft partner's insolvency


at

the commencement
at 10 days sight on H. J. Harris for $200, of business?
and my note at 60 days to balance. Paid and 10. Show the ledger accounts and
9

freight, $8.90. T, that


of

entries on the books and


D

Bought of X. & Co. mcise. for $970.69; should result from the following transac
paid by check after deducting 2 per cent tion: On February purchases
1,

1912,
T

for cash and making claim for $68.40 for from mase. amounting
H to

$7,642, terms
D

shortage, which claim was allowed. per cent off days, guaranteeing
60
in
5

Exchanged 10 $1,000 railroad bonds at the account up By mistake the


to

$5,000.
96% and interest for 100 shares of Z Mfg. goods are charged H, and the error
10 to

is

Co. stock at 103 flat. Semi-annual interest not discovered till fails, when cor
T

is
it

on bonds due in 2 months. Balance settled per cent


of

rected. claim made for


A

is

by cash payment. imperfections goods, which claim ad


in

is

In the following list of assets and justed after some time by


of
an

4. allowance
liabilities there is an error in the cash item; half the amount demanded on the claim.
Bills payable, $7,684; reserve ac
it:

find further purchase on March


4 in 28

made
A

T 10 is

count, $9,324; cash, $5,675; accounts receiv by $5,476, cash days.


of

from
is D
T

able, $588.48; capital stock, $276,000; notes Neither bill paid. On April trans
receivable, $71,422; surplus, $97,749; plant security accounts due him (T)
to

as

fers
D

and machinery, $89,000; interest due and from other parties amounting
of
to

$10,000,
ayable, $1,940; accounts payable, $3,346; collects half, crediting
T.

which week
A
D
1

dividends accrued (receivable), $66; bonded iater fails. The following adjustment
T

is

26
made: H gives his note with interest at each contributing $30,000 in cash. Each
6 per cent to D for the amount of his guar was to have a drawing account of $200 a
antee. As to the balance of T's indebted month. P has availed himself of this, but
ness D is obliged to look to the assets of Q has left his drawings untouched, on the
the bankrupt concern, being allowed to ben understanding that he was to receive no in
efit by the transferred account only to the terest thereon. An automobile was bought
extent of the dividend declared by the re for $1,000; it is now worth $500. The rent
ceiver of the bankrupt concern, which has been $100 a month, payable in advance.
finally pays 25 cents on the dollar. When Wages have amounted to $7,000. The con
H's note falls due he pays $3,000 and gives cern owes for moise. $10,000; for royalties
a note for the balance dated July 1, due in $1,000. There is due from customers $53,000.
3 months, which D discounts at the L. Na Breakage, repairs, etc., have amounted to
tional Bank. When the note falls due the $1,750. The firm made a profit of $2,000 on
bank charges it against H's account. a stock deal. The bank balance amounts
to $23,990. There is a three-year fire insur
ARITH METIC–Weight 3. ance policy, expiring Dec. 31, 1914, which
1. January 15, 1887, I purchased 2,000 cost $120. The year's raw material cost
bushels of potatoes at 37% cents. February
$87,000. There is one bad debt (entirely
20 I sold half of the purchase at 45 cents, uncollectible) of $10,000. This is not in
giving the purchaser an allowance of 2 per cluded in the customers' controlling account
cent for decay. March 25 I sold 475 bush above. The finished output cost $97,000.
April 1 I sold 500 more This includes items underscored. Other
els at 50 cents.
shop expenses totaled $250. The sales were
at 48 cents, and was obliged to throw away
the remainder as worthless. If money was $115,000. Discounts came to $1,500. There
worth 6 per cent, how much did I gain? is moise. on hand inventoried at $6,550. The
furniture, etc., cost $790, and may be taken
2. H. E. Moore's draft on J. F. Jones
for $1,200 favor of W. M. Swallow, dated as now worth $700. We consider all costs,
July 10, 1887, payable 90 days after date, expenses, etc., except those underlined to
be chargeable to the selling end of the
was accepted July 13 and discounted July
business.
31 at 7 per cent. Find the proceeds.
3. A debt of $7,500 is canceled by two From the above letter show the follow
ing: Ledger account balances, profit and
notes. One is a note for $6,000, maturing
in one month. The other is to be drawn loss account, manufacturing account, trad
ing account, final capital account of each
at 30 days for such a sum that when dis (profits and losses to be shared
partner
counted on its date the proceeds of both equally).
notes will pay the debt. What shall be the
face of the second note, the rate of dis 3 and 4. From the following letter pre
pare a statement of (a) the affairs of L &
count in both cases being 5 per cent?
Co.; (b) a statement of their affairs as per
4. I bought a farm for $5,600 and ex
pended 8 per cent as much for improve reduction agreement with the S Mfg. Co.;
ments. The cost of stock for the farm (c) a statement of the affairs of the S Mfg.
was 225 per cent of the sum expended for Co. before amalgamation; (d) a statement
improvements. showing assets and liabilities after amalga
These sums were drawn
from my bank and constituted 25 per cent
mation; (e) a statement showing disposi
of my original deposit. The net profit from tion of half the actual profits of the deal
through the distribution of a stock dividend.
the farm for two years was $500, which sum
I deposited. I then sold out at 10 per cent Show also the book value of the purchas
ing company's stock before the union, after
advance on total cost and deposited the
proceeds. How much was my bank account the same and after the payment of the
before the purchase? After the sale? What
stock dividend. All the above are to be
was the per cent of increase? made in proper bookkeeping form, and on
the assumption that the plant and machin
ery are finally scrapped and bring nothing;
Municipal Civil Service Commission, N. Y. that only 5 per cent of the accounts of L
BOOKKEEPER, 3rd GRADE. & Co. prove uncollectible; that there is an
unexpected loss on their merchandise of
(Salary, $1,200 to $1,800.) $5,000, and the R. E. is sold for $107,500,
July 14, 1913.
subject to mortgages herein referred to:
Date:
TECHNICAL–Weight 5. New York, June 1, 1913.
HANDWRITING AND NEATNESS— Messrs. L & Co.
Weight 2.
Gentlemen:—Your books show that while
(NOTE:—This examination was cancelled you started the year with a capital of $200,
on the ground that some of the questions 000, you have to-day a bank balance of
were too difficult for the grade.) $2,400, and owe for moise. . $34,000 and for
1 and 2. machinery, etc., $6,000. Your real estate
is valued by you at $76,000, and against it
New York, Jan. 1, 1913.
the M National Bank holds a mortgage of
Gentlemen:—We beg to report as follows: $40,000. It also holds $23,000 of your paper
Messrs. P & Q began business Jan. 1, 1912, unsecured. Your plant, machinery, etc., are
27
figured at $67,000, and there is due you soap being shipped directly from the fac
$60,000, of which $26,000 is in notes, all but tory. They average about 3,000 open ac
$6,000 of which have already been dis counts on their sales ledger. They wish
counted. Your inventories show mase. their books to show in detail the losses
worth $63,000. We propose the following: and gains arising from the business. State
We will take over your concern, assuming what books should be used, and mention in
all liabilities on these terms: The plant detail what accounts showing losses and
and machinery is to be put in at a value of gains should be kept. Give reasons.
$60,000; the $6,000 worth of undiscounted 7. Show how the items given below
notes to be thrown out as practically worth would appear on the appropriate account
less; the unsecured accounts due to be in the ledger:
taken at their book valuation less 10 per Aug. 1–Received cash for the use of
cent; the R. E. to be valued at $66,000; money loaned to A. B. Thurber, $25.
the mase. to be taken at a discount of 33 1-3 Aug. 5–Paid cash for use of money re
per cent. Payment for net assets over lia ceived from John Ames, $13.
bilities is to be made as follows: $4,000 in Aug. 10–Prepaid our note $800. Dis
cash, a second mortgage of $6,000 to be count given to maturity, $45.12.
placed against the R. E. balance in our Aug. 15–Paid cash for interest, $15.
stock at 110. Our indebtedness amounts to Aug. 20—Received cash for interest, $85.
$48,000; we have $33,000 on deposit with Aug. 30—Paid cash for interest, $50.
the V. State Bank; our equipment is con 8. (a) A joint stock company has been
servatively estimated at $75,000; we have formed for the purpose of conducting a
outstanding accounts (all first class) of manufacturing business. The capital stock
$260,000, and $200,000 worth of silver ore. is $150,000, divided into 3,000 shares of $50
Our capital stock consists of 4,500 shares, each; $100,000 of this stock is preferred
par value $100, of which 4,000 have been stock, and the remaining $50,000 is common
issued. Sufficient additional stock will be stock; 75 per cent of the preferred stock
created to complete the deal and to pay a has been subscribed at par and paid for in
stock dividend of 5 per cent. We have also cash; 40 per cent of the common stock has
a bonded debt of $100,000, which is to be been subscribed at $40 per share, on which
retired out of the first profits made on the a 20 per cent installment has been paid;
sale of your R. E. - $10,000 of the common stock have been is
sued to a firm of bankers in payment for
Yours truly, services rendered in organizing the com
pany; $5,000 of the preferred stock have
S MFG. CO.
been issued in payment for the use of a pat
5. In the trial balance given below there ent. Require the entries to open the books.
are two errors, one on the debit side and (b) At the end of the year the net profits
one on the credit side. Assume that the of the above company are $28,000. A divi
footings of the trial balance should be $30, dend of 7 per cent on the preferred stock
016.55. Ascertain the exact amount that and 5 per cent on the common stock is
each side is wrong, and show clearly and declared. It is also ordered that the re
fully your method of finding the errors, maining profits be transferred to a sinking
and state how you would verify the fact fund account. What are the proper en
that you had found the errors: tries?
9 and 10. A and B are equal partners.
Trial Balance— They wish to dissolve partnership and di
Dr. Cr. vide their resources and liabilities between
Stock account. .. .. .. . . .. $5,000.00 them. The ledger accounts stand as fol
Cash . . . . . . . . . . . . . . . . . . . $9,832.00 7,234.21 lows: A is debtor $1,300, credit $4,500; B
Mdse. . . . . . . . . . . . . . . . . . . 15,671.00 13,327.00 is debtor $4,500; expense, debtor, $1,350;
Bills payable............ 250.00 credit, $1,617; merchandise, debtor, $23,400;
Banks & Co............. 1,725.50 credit, $22,950. Merchandise per inventory
.
Expense . . . . . . . . . . . . . . 50.70 which B takes to his account, $4,650; cash
J. D. Brown............ 1,534.94 230.50 on hand, which A takes to his account,
H. G. Fuller............ 672.41 1,519.24 $1,395; bills receivable, which A takes, $5,
S. W. Burrows. . . . . . . . . . 150.00 75.00 051; accounts receivable, which B takes to
A. E. Schmidt. . . . . . . . . . . .80 733.10 his account, being allowed a discount of 16
E. F. Mully. . . . . . . . . . . . . .300.00 1,575.50 per cent for bad debts, $2,347; accounts pay
able, which B assumes, $1,985; store and lot,
6. and B are general jobbing and com
A which A takes to his account, $2,450; bills
m ssion merchants, dealing in grains, pro payable, outstanding, which A assumes,
visions and dairy products, buying and sell $1,133. Commission account is credited
ing on their own account as well as sell
ing on commission. They are also general
$2,033. Find the amount one must pay to
the other in cash and the entry for the
ag cnts for the Pearl Soap Co., marketing same on the firm's book.
the entire output on a net commission of
8 per cent, all discounts allowed on prepay ARITH METIC–Weight 3.
ment of bills being charged to the soap com
pany. They are to credit the soap com (To be finished by 4 P. M.)
pany with all sales, having a uniform credit 1. With what sum would a person be
of 90 days, with monthly settlements, the credited at his bank if he should deposit a

28
draft for $9,000, accepted June 23, and nishes $4,000 in cash and gives his note at
dated June 20, the draft being payable at 90 days, bearing 6 per cent interest, for
90 days' sight? $500. Q furnished $4,800 in cash. Make the
proper journal entries for books of the co
2. A broker charged 34 per cent for buy
ing goods, and his brokerage was $94.80. partnership, journalizing “cash.”
He afterward sold the goods at 9% per cent 6. At the end of one year the above-men
brokerage, and it was found that the net tioned firm, L, M and Q, finds itself in
profit from the transaction was $946.45. possession of merchandise to the amount
VWhat did the goods sell for? of $10,000; it has bills receivable amounting
to $2,000. Book accounts of $3,000, as fol
3. Borrowed $550 at 5 per cent for 4 lows:
In on ths. When the loan was due I returned
the sum borrowed, and sold the lender an Due from L. . . . . . . . . . . . . . . . . . . . . . . $1,000
invoice of goods amounting to $3,800. How Due from M. . . . . . . . . . . . . . . . . . . . . . . 700
long a term of credit at 6 per cent per
Due from Q. . . . . . . . . . . . . . . . . . . . . . . 1,300
annum should I allow the purchaser to can
cel the interest on the above loan 2
and $1,500 cash in hand. It goes on firm
Received $153.66 for the use of a sum
4. notes discounted at the Tenth National
of money for 3 months, 18 days, at 9 per Bank, $1,800, and the sundry creditors for
cent. What would be the amount of the merchandise, $890. Make up for these items
sum from December 27, 1886, to October 16, a balance sheet, and profit and loss state
1887, at 6 per cent? ment, and frame the following accounts:
L's capital account; M's capital account;
Q's account, showing the loss or gain of
each partner.
7. The above-mentioned firm have re
Municipal Civil Service Commission, N. Y. solved to dissolve; has converted all its
assets, except debts due from the partners,
BOOKKEEPER.
into cash, without loss. Prepare the neces
Date: September 16, 1904. sary accounts to close the business, exhibit
ing balance sheet, and show the following
1. (a) Cash Book; (b) Journal;
Define: ledger accounts: Cash account; bills pay
(c) Ledger; (d) Controlling Account; (e) able account; accounts payable; L's account;
Cost Account; (f) Stock Book; (g) Pur M’s account; Q's account.
chase Book: (h) Sinking Fund; (i) Reserve
Fund; (j) Deficiency Account. 8. The balance sheet of a corporation
is summarized for you as follows:
2. What is the difference (a) between
capital and working capital; (b) between
discount and interest; (c) between common Assets.
stock, watered stock and preferred stock;
(d) between bonds and debentures; (e) be Cash, Stock & Accounts Receivable. $67,500
tween rent and royalty? Manufacturing Plant . . . . . . . . . . . . . . . 15,000
3. A and B are partners, sharing profits
equally. Their books have been kept “sin
gle entry,” but they wish to have them
kept by “double entry.” Determine the loss Liabilities.
and gain of each partner and formulate the Notes & Accounts Payable. . . . . . . . . . $49,500
Journal entries required in order to make Capital Stock . . . . . . . . . . . . . . . . . . . . . . 37,500
the desired change in bookkeeping system
from the following abstract of condition of Would you consider the business insolv
the business: Original investment of each ent? Is anything lacking in this statement
partner is $12,500; withdrawals of A $2,500, of affairs? If so, what? Give full reasons
of B $2,000; accounts receivable, $8,500; ac for all your answers.
counts payable, $6,000. Merchandise (as 9. Show what ledger entries result on
per inventory), $18,000; cash in bank, $5, the books of Z and C from the following
000; bills receivable, $2,300; bills payable, transactions:
$2,000; real estate, $5,000.
The firm of Z & Co. purchased from Hen
4. X and Y form a partnership on Janu derson Bros. office furniture for 72 pur
ary 2, each investing $7,500, the profits to chasers for cash; $798 worth of stationery
be decided in proportion to the amount of purchased from Barnes & Barnes; 5,000
capital invested and the time it is employed. sacks of flour at $1.50 per sack; sells to
On March 1, X invests $2,400 additional Rowland, Reed & Co. 1,500 bushels of oats
and Y withdraws $1,200. On July 1, Y in at 35 cents per bushel; pays freightage on
vests $2,500 additional and X withdraws same (this freight is to be charged to pur
$2,500. On December 31 the profits are chase), amounting to $27.50; pays cartage
found to be $4,800. Show each partner's on same amounting to $5.50; gives 30-day
average investment and share of the profits. 5 per cent note to Barnes & Barnes in set
5. L, M and Q form a partnership, agree tlement of flour bill, deducting 2 per cent
ing to share profits and losses in proportion discount and also $3 for damages to goods
to the capital invested. L furnishes $3.000 in transit. Pays note 15 days before it falls
in cash and $2,000 in merchandise. M fur due.

29
ARITH METIC. your name, but with your examination num
ber.
1. Add:
256,678,905,489.14
245,867,986,697.08 Municipal Civil Service Commission, N. Y.
378,897,897,582.98
BOOKKEEPER.
789,293,827.02
1 Date: Many years ago.
289,026,394,792.90
968,896,589,190.76 ARITH METIC.
444,444,444,444.44 1. Add:
592,295,454,856.77 1896857.4635.2678934
745,956,298,867.62 23345.567789.9234567
256,652,796,978.99 2637,485927899.37889
269,978,932,685.21 2436475869.28374665
120,078,870,329.82 243546.57899.2773879
4.99 17.961594.8235426997
176,786,258,935.57 654333552300654270
2839.48.2785.47732656
2. A lodge room is 48 feet 9 inches long, 189078546789.253928
37 feet 6 inches wide, and 18 feet high. It 65.784398979657.2332
is plastered at $6.25 a square yard, and the 2637.485928.76543.245
floor is to be covered with Brussels carpet 24354657.682.4678.659
(34 yard wide) at $1.50 a yard. What is the 98.7654329876543.298
entire bill? 8675.645324256.72843
3. A commission merchant sold a cargo 2567893.67823567234
of sugar for $39,585, on commission of 2% 24354758926783.2566
per cent. He bought coffee with the net 243.2553.27899765486
proceeds, after having retained his commis
1594.13894.132553378
sion of 1% per cent for buying. How much 298.5436.7865.4787.579
did he receive for buying the coffee? 75846329865.4321811
4. Bought goods at $201, 12% and 10
2. Add: wº
per cent off, and the discounts amounted
2738492784367234
to $555. What was the retail price? 210.0034278927853
5. A planter sold 120 barrels of sugar, 2789.3267834267.43
taking in payment a note of 105 days, which
96.857.4635.2678.999
he discounted at the bank the same day at 2673482775483.265
7 per cent. The proceeds of the note was 796857.4635278975
$1,468.50. At what price per barrel did the 18796.57893427899
sugar sell?
176895.4789.243647
6. If Mr. Brown's property is valued at 27893487654321.87
$1,050, and has tax of $12.60, what is the
769584.32678.43211
entire amount of tax if the valuation is
27384957283.64728
$78,975?
8667558339223426
2131415161718198
LETTER. 26378.49.276543.232
2637.485928.765432
Write a letter of not less than 250 words 1896857.432893876
to some friend properly dated, addressed, 543.2893467823428
etc., on the recent strike. 18976544.31243674
N. B. No. 1–In rating this letter, con 27384928.76345299
sideration will be given not to the merit 99.887766.5544:3322
of the opinion expressed, but to the expres
sion of these opinions, grammar, composi 3. Find the cash balance, September 30,
tion, etc. 1886, on the following account, including
N. B. No. 2—Sign your letter, not with interest at 6 per cent:

Dr. Wm. Callen, in a/c current with H. A. Rice Cr.

1886 1886
Feb. 5 To
**
M dse. 30 da . . . . . . $284.00 Apr. 30 By Cash . . . . . . . . . . . . . $200.00
Mar. 7 “ “ “ ...... 370.00 May 18 “ " . . . . . . .. . . . . . 300.00
Apr. 12 “ “ “ “...... 590.00 Aug. 28 " " . . . . . ... . . . . . 120.00
May 24 “ Cash. . . . . . . . . . . . 840.00 Sept. 15 “ " . . . . . . . . . .. . . 124.00

4. On January 12, 1884, J. Fagan & Son Co. $2,500. What should be the date of an
sold Ferguson Bros. & Co. a bill of mer interest-bearing note to be given to J. Fa
chandise amounting to $1,825, on 3 months' gan & Son in settlement of the balance due
credit; and on February 11, 1884, J. Fagan Ferguson Bros. & Co.?
& Son borrowed from Ferguson Bros. & 5. A ship is insured for enough more
30
than its value to include the cost of insur cent off for 60 days, giving note due March
ance at 1% per cent, and $6 for the policy 3, which note at maturity is liquidated by
and survey. If the value of the ship is the payment of $10,000, together with a
$51,350, what should be the sum insured? new note at three months bearing interest
6. Harvey and Roberts engage in busi at the rate of 6 per cent per annum. On
ness January 1, 1885, Harvey investing $5, April 1 this note is discounted for B by
000 and Roberts $8,000. On May 1, Harvey the X State Bank, and on April 15 is taken
invested $2,000 additional; and on June 1 up by A with check on the Y National Bank,
Roberts withdrew $1,000. On August 1 A obtaining the necessary funds for settle
they took in Williams as a third partner, ment by borrowing $2,000 from C, and ne
who invested $7,000. On January 1, 1886, gotiating a call loan at 2 per cent from the
their net gain was $5,000. What was the Y National Bank with which he pledges
share of each partner? 100 shares of R S T stock, par value $100
per share, market value 105. On January
10, 1906, A sells to D 1,000 pieces flannelette,
SPECIAL PAPER. 45 yards to the piece, at 8 cents a yard,
terms 2 per cent off 4 months. Settlement
1. A startsin business with $1,500 in
bank, goods which cost $8,000, and furniture for this purchase is made May 10, an allow
ance having been claimed of $15 and admit
and fixtures which cost $1,250. He owes ted, by check which is deposited in Y Na
$2,000 on open account for goods and $3,000
tional Bank by A, May 11. The following
borrowed money for which he has given day A takes up his call note by paying to
his notes. Journalize and post these items.
the bank the proceeds of this check, less
2. Show how you would close merchan $2,000 which he repays to C, giving the bank
dise account at the end of the year after 1, at 5 per cent,
stock-taking. a new note due November
endorsed for accommodation by C. When
3. When do you credit bills payable ac
count? this note falls due it is not paid and is
accordingly protested (protest charges
4. What does a debit balance in profit $2.63), and is finally taken up by C on De
and loss accountindicate?
ledger account in which cember 2. On December 10, A makes set
5. Mention a
tlement with C by giving him a mortgage
all the entries are usually debit. How do on his house for $10,000. The balance due
you close such an account?
in settlement of the transactions between A
6. John Smith gives Thomas Brown a
quarter interest in his business for $20,000 and C is settled by check, A allowing C $100
cash. Show exactly how the transaction for expenses of searching, recording, etc.
would appear on the ledger. 4 to 6. Show all the other postings that
7. How would you transfer on your should result from the above-mentioned
books $5,000 from contingent account to transactions on the books of A.
salaries account? 7. The firm of A, Z & Co. is about to be
8. . If in drawing off a trial balance you turned into a stock company with 2,000
found the totals differed by 9 cents or some shares of $100 (par value) each, one-half 6
multiple of that amount, what would you per cent preferred, one-half common. It
infer caused the error? owes for merchandise $50,000 and has doubt
9. How do you determine from a bal ful accounts receivable of $40,356, estimated
ance sheet the net profits? to be actually worth $30,000 net. In addi
tion expert appraisers' schedules show the
LETTER. following: Merchandise completely manu
Write a letter of not less than 300 words, factured, $100,000; Bills Receivable, $11,500;
properly dated, addressed, and signed, to Accounts Receivable Guaranteed (in addi
some friend, on the recent disaster in Mar tion to those mentioned above), $10,000;
tinique. Furniture and Fixtures, $2,400; Moise. part
ly finished, $40,000; Cash, $1,000; Industrial
stock (par value $7,000) estimated at $5,500;
Trade-marks, Copyrights, etc., $19,000; Real
Municipal Civil Service Commission, N. Y. Estate (mortgaged for $10,000), $20,000;
BOOKKEEPER, Tools, Lathes, etc., $2,500. At what sum
4th GRADE.
must the good-will be estimated in order
(Salary, $1,800 to $2,400.) that the total assets may appear equal to
the total liabilities, on the books of the
SPECIAL. new company?
Date: January 15, 1907. 8. The PQ R Co. has the following re
sources and liabilities on January 1, 1906:
HANDWRITING AND NEATNESS to be Resources: Fixed assets, $100,000; Receiv
rated on 1 to 3. ables, $550,000; Cash, $150,000. Liabilities:
Capital Stock (par value), $1,000,000; Ac
1 to 3. Show in proper form the cash counts, etc., Payable, $200,000. Is the com
entries that should result on the books of pany solvent? Would it be solvent if the
A, B, C and D, from the following set of fixed assets proved to be worthless and
transactions: . On January 2, 1906, A buys the receivables realized only $100,000?
3f B, 7,000 pieces flannelette, 45 yards to Would your answers be different if instead
the piece, at 6% cents per yard, terms 5 per of the capital stock being $1,000,000, the
31
capital stock were $500,000 and the bonded N. B.-Brokerage in stock transactions is
debt $500,000 (6 per cent mortgage bonds) calculated from par value.
due 1925? Give full reasons for all your
an Swers.
9. On January 7, 1906, W, a fruit grower Municipal Civil Service Commission,
N. Y.
and dealer, consigns to Y, a New York
commission merchant, 100 crates of berries, PROMOTION GRADE GEN
TO 5th
40 boxes to the crate, at the same time ERAL BOOKKEEPER AND-
drawing on consignees at sight for $100. AUDITOR,
The freight charges amount to $17.28. One
fourth of the goods is spoiled so as to be Department of Correction.
unsalable through the negligence of the
railroad company, which pays $50 damages (Salary, $2,400 and up.)
to Y in settlement of claim for reimburse
ment, and of the remaining boxes of fruit, Date: July 14, 1911.
one-third is sold at 5 cents a box, and the BOOKKEEPING—Weight
balance at 10 cents a box, cash, on July
5.
10, the auction and commission charges (To be finished by 1 o'clock.)
being 5 per cent. W’s draft is paid by Y
on July 8. Prepare a statement to be sent 1. (a) Explain the difference between
on July 20 from Y to W with check in single entry and double entry bookkeeping.
settlement of account to that date, interest (b) How would you change a single en
being charged and allowed at the legal rate. try set of books to double entry?
10. The X Company starts business Janu 2. Define clearly the following terms:
ary 1, 1905, with a capital stock of $100,000 (a) General Ledger; (b) Stock Ledger; (c)
and a bonded debt of $100,000. The bonds Balance Sheet.
are 5 per cent 50-year bonds. On Decem 3. Explain fully the process of closing
ber 31, 1905, its resources and liabilities ex a set of double entry books.
clusive of stocks and bonds are as follows: 4. (a) Make out a trial balance from the
Real Estate, $500,000; Cash, $20,000; Re following items taken from a set of books:
ceivables, $100,000; Bills and Accounts Pay Accounts Receivable, $16,119.10; Mase.
able, $14,000. There must be a sinking fund (cost), $16,205.46; Accounts Payable, $10,
contribution of 2 per cent and a 10 per 302.22; Bills Receivable, $4,102.50; Interest
cent reserve for depreciation. After these paid, $15.62; Cash, $4,834.14; Discount re
deductions what rate of dividend can be ceived, $62.50; Bills Payable, $4,260; Office
paid? Under the circumstances outlined Fixtures, $1,750; Real Estate, $10,500; Insur
above, can it be paid in cash? If so, how? ance (cost), $150; A. J. Morley capital ac
If not, how? Give reasons for your an count, $38,750; Interest earned, $353; Ex
Swers. pense, $50.60.
(b) . If the value of the merchandise was
$6,840 at the end of the period, what would
ARITH METIC. .
be the profit or loss of above business?

(Give all the figuring on the ruled sheets.) 5 and 6. John Wright and William Leach
start in business on January 1st. Wright
1. A plot of ground 200 feet square is brought in $12,000, and Leach $8,000. Prof
valued at $60,000. It is divided into lots its to be divided in proportion to capital
of the following sizes: 25x200, 125x80, 75x60, brought in. On December 31 following,
20x100, 20x75, 60x100, 35x200 and 40x100. stock was taken and amounted to $12,600,
What is the valuation of each lot? and the trial balance of books at that date
was as follows:
2. A firm failed with liabilities amounting
to $52,250 and with assets, exclusive of real Purchases . . . . . . . . . . . . . . . . $28,750
estate, amounting to $2,042.50. The assignee Sales . . . . . . . . . . . . . . . . . . . . . $20,400
obtained for the real estate $31,350 and de Freight . . . . . . . . . . . .. . . . . . . 120
ducted for settling the bankruptcy $475. A's Salaries . . . . . . . . . . . . . . . . . . 570
claim against the firm was $7,284, B's $6,382, Taxes . . . . . . . . . . . . . . . . . . . . 220
C's $5,794, D's $4,767 and E’s $3,764. How Traveling .. . .. .. ... . . .. .. 190
much did each of the creditors receive? Discount . . . .. . .. . . .. .. . .. 70
3. What sum will be due on January 18, Gas and Fuel. . . . . . . . . . . . . 40
1905, on a note of $3,500, dated May 8, 1901, Petty Expenses . . . . . . . . . . 120
and drawing interest at 5 per cent per an Accounts Receivable . . . . . . 4,320
num, payable semi-annually, if the first four Cash in Bank. . . . . . . . . . . . . 6,750
interest payments were paid when due and Accounts Payable . . . . . . . . 2,750
no subsequent payments made? John Wright Capital Acct. 12,000
4. On June 1st, A purchased through a Wm. Leach Capital Acct . . . 8,000
broker 100 shares of railroad stock at 837%, John Wright Drawing Acct 1,200
par value 100, and deposited $1,000 as mar
gin.
Wm. Leach Drawing Acct. 800
On June 19th the stock was sold at
863%. What was the profit, allowing Wé per $43,150 $43,150
cent each way for brokerage and 6 per cent
interest on margin and on money borrowed Make out a profit and loss account and
to complete purchase? a balance sheet.

32
7 and 8. Explain fully the differences in 4. X, Y and Z are in business together,
books and system and methods of doing X and Y being partners sharing according
business between your department and an to capital invested, and Z having an “in
ordinary commercial business. terest” of 10 per cent of the net profits.
X has put in $50,000 and Y has put in
$100,000. At the end of a year their assets
and liabilities are as follows:
Municipal Civil Service Commission, N. Y.
PROM. 8th TO 10th GRADE BOOK Cash . . . .. . .. . . . .. . . .. . .. . .. . .. . . . $6,000
KEEPER. Accounts Receivable .............. 40,000
Accounts Payable . . . . . . . . . . . . . . . . 30,000
Department of Bridges. Bills Payable ..................... 7,000
Bills Receivable . . . . . . . . . . . . . . . . . . 2,000
Date: June 6, 1907.
Merchandise (as per inventory)... 55,000
(To be finished by 1 o'clock.) Real Estate ...................... 15,000
(NOTE:—The salary classification has Buildings and Machinery.......... 50,000
since been changed. The present corre Furniture and Fixtures............ 26,000
sponding grades are 4th and 5th.)
1. A joint stock company is formed with Z receives as his share $600. He claims
a capital of $200,000, half common and half
that he ought to receive $1,000 instead, on
6 per cent preferred (par value of every
the grounds (1) that the Real Estate has
share $100). It also issues 100 5 per cent been undervalued, and (2) that even if it
bonds, par value $1,000 each. Half the com
has not been undervalued, he is entitled to
mon stock is sold at 50, the other half is
given as a bonus with the preferred stock,
more than $600. If he is right in his con
tentions, how much is the Real Estate ac
one share of common with each two shares tually worth, and what is the share of each
of preferred. All the preferred is disposed of the two partners in the profits?
of at an average of 95 and all the bonds are
placed at 102 net. Show journal entries 5. State fully the jurisdiction of the Com
resulting from the above transactions. missioner of Bridges.
2. A, B and C form a partnership, A 6. What report of moneys received is to
contributing $50,000 in cash, B contributing be made by the Commissioner to the Comp
plant, machinery, etc., valued at $60,000, troller? -
and C contributing merchandise worth $70,
7. (a) A has $15,000 invested at 4% per
000. The partners are to share profits and cent; $12,500 at 5 per cent, and $9,000 at
losses equally, but interest at the rate of 3% per cent. What is his income for 2
5 per cent per annum on each man's capital years, 4 months and 15 days from the three
prior to division of profits. At the end of investments?
the year the books show a net gain, before
making interest allowances of $5,000. Make (b) The exact calculated tax on a house
journal entries to open firm's books and also and lot together is $335,364.75, the rate being
1.49051 per cent. The house alone is valued
show ledger accounts of each partner after
apportionment at $7,500. What is the valuation of the lot
of interest and profits or
losses. per square foot, the dimensions being 20x
100 feet?
3. In the following items, taken from a
bank statement, there is an error in the 8 to 10. Write a letter of not less than
“cash” figures; find it: 250 words, giving an account of various
methods suggested for relieving the crush
Capital Stock . . . . . . . . . . . . . . . . . . $4,375,000 at the entrance to the Brooklyn Bridge.
Mortgages . . . . . . . . . . . . . . . . . . . . . . 10,435,000
Stocks and Bonds. . . . . . . . . . . . . . . 4,805,000
N. B. 1.-In rating this letter, consider
ation will be given, not to the merit of the
Surplus . . . . . . . . . . . . . . . . . . . . . . . . . 6,000,000
opinions expressed, but to the expression
Undivided Profits, etc. . . . . . . . . . . 564,000
Bills Purchased .. . ... .. .. ... . .. 1,681,000
of these opinions, grammar, composition,
etc.
Loans on Collateral. . . . . . . . . . . . . 13,700,000
Overdrafts (of depositors). . . . . . 1,000 N. B. 2.-Sign this letter with your ex
Deposits (subject to check)..... 24,000,000 amination number and not your name.
Cash on hand. . . . . . . . . . . . . . . . . . . 5,254,000 N. B. 3.-Handwriting will be rated on
Due other banks................ 1,135,000 this letter.

33
BOOKKEEPER, NEW YORK STATE SERVICE.

Answers to Examination Questions and Specimen Ques


tions for 4th, 5th and 6th Grades.

State of New York—The Civil Service SOLUTION:


Commission. A mount of note, day. . . . . . $100.00

BOOKKEEPER, 6% interest for 90 days


4th GRADE. .
(1%%) . . . . . . . . . . . . . . . 1.50
(Salary, $721 or $600 and Maintenance.)
Value of note at maturity. ... . . $101.50
Date: January 26, 1916.
Note due April 2, 1916 (leap year)
COMMERCIAL ARITH METIC. Discounted Feb. 26, 1916.
Feb. 26 to April 2=36 days.
Directions: 30 days interest=%%. $.50
1. Allcomputations must be given in full:
2. In computing interest use only the 60 0 ... . .. . .. . .. .. . .10 .60
day method, or the 6 per cent method, but
no other. Consider 360 days as one year. Proceeds (Ans.) $100.90
3. Concise, business-like methods will be
given preference. Ques. 4:—Make out, in correct form, the
following bill:
Henry Wilson bought of the Camden Sup
Ques. 1:—A commission agent bought for ply Company on February 23, 1916, 6 dozen
a merchant $5,482.50 worth of goods and Jack-planes at $11.50 dozen; 4 dozen socket
sold same for $6,963.50. The rate of com chisels at $6.50 dozen; 3 dozen gimlets at
mission both for buying and selling was 75 cents dozen; 9 dozen Claw-hammers at
2 per cent. Did the merchant gain or lose $6.50 dozen. On the planes he was allowed
on the transaction, and how much? a discount of 20 per cent; on the chisels,
15 per cent, and on the hammers, 2% per
SOLUTION: cent. The terms of sale on the whole bill
Cost of goods. . . . . . . . . . . . . . . . . . . . $5,482.50 were 5 per cent cash, 2% per cent 10 days,
Add 2% for buying. . . . . . . . . . . . . . . . 109.65 net 30 days. Wilson paid by check on
Add 2% for selling at $6,963.50. . . . . 139.27 March 4, 1916. The Supply Company re
ceipted same through its bookkeeper.
Total . . . . . . . . . . . . . . . . . . . . . . . . $5,731.42
SOLUTION:
Selling price ............. $6,963.50 February 23, 1916.
Less total, as above....... 5,731.42
CAMDEN SUPPLY COMPANY.
Gain (Ans.) . . . . . . . . . . . . . . . . . . $1,232.08 Sold to Henry Wilson, Dr.
New York City.
Ques. 2:—Determine the simple interest Terms—5/Cash 2%%/10 N/30
on $25,000 from June 16, 1914, to December
31, 1915, at 6 per cent. Compute interest 6 doz. Jack Planes $11.50 $69.00
on the basis of 360 days per year. Less 20% 13.80 $55.20
SOLUTION: “ Socket Chisels

-
4 6.50 26.00
Principal .. . .. . .. . .. . .. . .. . .. . .. . . $25,000 Less 15% 3.90 22.10

Interest: 9 “ Claw Hammers 6.50 58.50


June 16, 1914, to June 16, 1915, Less 2%% 1.46 57.04
1 year at 6% . . . . . . . . . . . . . . . . . $1,500
June 16, 1915, to Dec. 31, 1915= 3 “ Gimlets .75 2.25
198 days.
180 days (% year) at 3%. . . . . . . . 750 $136.59
18 days (1/10 of $750). . . . . . . . . . 75 Less 2%% Cash Discount 3.41

Total interest (Ans.).......... $2,325 $133.18


Paid, March 4, 1916,

Ques. 3:—Compute the proceeds on a note CAMDEN SUPPLY CO.


made January 3, 1916, 90 days, 6 per cent, By John Doe,
discounted February 26, 1916. Cashier.

34
Ques. 5:—A street 3892.3 feet long, 60 feet Curbing.
wide (curb to curb), is paved with brick.
A traction company operates cars on two 3892.3 ft.=1297.4 yds. of street length
tracks, and is responsible for the paving of 2
the street between two parallels each 7 feet
from the center line of the street. The city 2594.8 Total yols. of curb
is responsible for the remainder of the pav .75 Cost per ya.
ing and the curbstones. Compute the share
1297.45
of expense to (1) the traction company, (2)
181643
the city, paving costing $3 a square yard,
and curbing 75 cents a yard.
Paving.
$1946.175 Cost of curbing borne en
tirely by City.
3892.3 ft.= 1297.4 yds.-length of street
60 ft.= 20 yds. =width of street

25,948 sq. yols.=area of street


3 cost per sy. yd.

$77,844 Total cost of paving. Summary.


pays for 14
“ ft.

Company (2X7)
“ of

-- st. width
--
City 46 (balance)

-- -- … Cost Cost
-i.

Total 60 to Co. City Total

to
Company share 14/60X$77,844= $18,163.60 Paving $18,163.60 $59,680.40 $77,844.00
City's share 46/60X 77,844= 59,680.40 Curbing .
1,946.17 1,946.17
.
.
.
.
.
.
.
Total $77,844.00 Total $18,163.60 $61,626.57 $79,790.17

BOOKKEEPING.
Directions:
All computations must given full, on the white paper by the
in be

furnished
in
1.

examiner, and must be handed with the other papers.


computing interest use only the 60-day method, or the per cent, but no
In
2.

other.
h

Concise, business-like methods will be given preference.


4. 3.

entering the following least three books are necessary: (a) Journal, (b)
In

at

Cash Book, (c) Ledger.


Using the white paper furnished by the examiner, rule up each of the books
5.

you use. The ruling highly important. The form your


of

of

of

books account
is

its accuracy and the methods by which you obtain results, will re
as

as

work well
ceive consideration rating.
in

Use only the double entry system


of

accounts.
6.

Ques. 1-5:-Herbert Saunders, Utica, N. Y., sells his retail grocery business
a to

Samuel Richards, the buyer take possession January


to

Saunders prepares
3,

1916.
balance sheet for Richards as follows:
Resources.
Cash on hand and bank...............................................
in

$1,873.62
Accounts Receivable ..................................................... 4,928.73
Notes Receivable ......................................................... 2,179.31
Real Estate and Building................................................. 25,500.00
Office Furniture and Fixtures.

.
2,628.75
.
.
.
.
.
.
.
.
.
.
.
.
.
.
.
.
.
.
.
.
.
.
.
.
.
.
.
.
.
.
.
.
.
.
.
.
.
.
.
.
.
.
.
.

Store Fixtures 4,382.65


.
.
.
.
.
.
.
.
.
.
.
.
.
.
.
.
.
.
.
.
.
.
.
.
.
.
.
.
.
.
.
.
.
.
.
.
.
.
.
.
.
.
.
.
.
.
.
.
.
.
.
.
.
.
.
.
.
.
.

Inventories 6,781.41
.
.
.
.
.
.
.
.
.
.
.

.
.
.
.
.
.
.
.
.
.
.
.
.
.
.
.
.
.
.
.
.
.
.
.
.
.
.
.
.
.
.
.
.
.
.
.
.
.
.
.
.
.
.
.
.
.
.
.
.

Merchandise $6,483.26
.
.
.
.
.
.
.
.
.
.
.
.
.
.
.
.
.
.
.
.
.
.
.
.
.
.
.
.
.
.
.
.
.
.
.
.
.
.
.
.
.
.
.
.
.
.
.
.

Sundries 298.15
.
.
.
.
.
.
.
.
.
.
.
.
.
.
.
.
.
.
.
.
.
.
.
.
.
.
.
.
.
.
.
.
.
.
.
.
.
.
.
.
.
.
.
.
.
.
.
.
.
.
.

$48,274.47

Liabilities and Present Worth.


Mortgage Payable per cent.... $10,000.00
........................................................
5

.
.
.
.
.
.
.
.
.
.
.
.
.
.
.
.
.
.
.
.
.
.
.
.
.
.
.
.
.
.
.
.
.
.
.
.
.
.
.
.
.
.

Accounts Payable 5,202.48


Notes Payable ........................................................... 3,675.56
Herbert Saunders ........................................................ 29,396.43

$48,274.47
Richards' cash balance in the Merchants’ & Traders' Bank, Utica, January 3, 1916,
is $36,893.46. He buys the above business, giving Saunders his check for $27,000, the
price finally agreed upon. He also invests, in his new business, his final balance of
cash in bank. (N. B.-Richards does not take over Saunders' cash.) Richards buys
for cash, new account books, $75. (a) Open Richards' new books, showing all details

º:
of the above transactions. (Note:—Make Journal Dr. and Cr. entries and then post
to ledger.) (b) Continue the business as follows:
January 3. Richards continues the as before. Monthly payroll includes
1 bookkeeper, $95; 1 cashier, $85; 1 stenographer, $80; 4 clerks, 2 at $75 each and 2 at
$70 each; 1 office and store helper, $25; 2 delivery boys, $40 each.
Saunders has rented two small delivery wagons and ponies. Richards discontinues
this charge and buys one delivery auto, $645, of the Utica Garage, paying cash. He
hires a chauffeur at $90 per month, discharging one of the two delivery boys. He buys
1 barrel of gasoline (50 gallons) at 25 cents a gallon.
Saunders was enlarging his office at considerable expense, having paid for only a
part of the work. Richards agreed to pay for the remainder when completed accord
ing to his own specifications; Contractors, Henderson and Robinson.
Richards bought the following for cash: 1 small safe, $126; 1 typewriter, $95; 1
12x16 rug, $40; desk sundries, $32.16; stationery and stamps, $85.

January 8. Sold to Herman Ridder, on his 30-day 6 per cent note, 2 barrels flour
at $6.90; 1 barrel sugar (320 lbs.) at 4% cents per lb.; 1 chest tea (150 lbs.) at 35
cents per lb.; 25 lbs. coffee, unground, at 32 cents per lb.
Cash sales, $346.78.
Bought sundry store supplies, $25.48.
January 17. Bought 20 barrels flour at $6.10; 10 barrels sugar (320 lbs. each) at
4 cents per 1b.; 10 chests tea (150 lbs. each) at 31% cents per lb.: 200 lbs. coffee at 28%
cents per lb.; 4 barrels rolled oats at $6; 10 dozen brooms at $3.15.
Sold James Lawson 25 lbs. coffee at 31% cents; 5% barrel rolled oats at $6.35;
1 barrel flour at $7; % chest tea at 38 cents per lb.; 6 brooms at $3.40 per dozen. Re
ceived cash $15; balance on account.
Cash sales, $492.83.
Discounted Ridder's note, given January 8th, depositing proceeds.
Henderson and Robinson presented their bill for work on office, $195, this being
the portion of the work done according to specifications of Richards. Paid in full by
check. (Do not charge this to “Expense.”). ,

January 25. Received check from Lawson in full of account (sale January 17th).
Cash sales, $526.38.

January 31. Paid all employees.

ANSWER:

JOURNAL.
January 3, 1916.

L.F. Dr. Cr.


2 Cash .. . .. . .. . .. . .. . .. . .. . . . .
. .. . . . . .. .. . . .. . . .. .. . . .. . . $9,893.46
3 Accounts Receivable . . . . . . . . .
.. .. . . . . . .. . . . . . . .. . . . . . . . . 4,928.73
4 Notes Receivable . . . . . . . . . . . .
.. .. . . . . . . . . . . . . . .. .. .. . . . . 2,179.31
5 Real Estate and Buildings. . . .
. ... . . . .. .. . . .. . . . . .. . . . . . . 25,500.00
6 Office Furniture and Fixtures. . . . . . . . .. .. . . . .. . . . .. .. . . . . 2,628.75
7 Store Fixtures . . . . . . . . . . . . . . . . . . . . . .. . . . . .. . . .. .. . . .. . . 4,382.65
8 Merchandise Inventory ................................. 6,483.26
9 Sundries Inventory . . . . . . .. . . . . . . . . . . . . . . . . . . . . . . . . . . . . . 298.15
11 Mortgage Payable . . . . . . . . . . . . . . . . . . . . . . . . . . . . . . $10,000.00
12 Accounts Payable . . . . . . . . . . . . . . . . . . . . . . . . . . . . . . 5,202.48
13 Notes Payable . . . . . . . . . . . . . . . . . . . . . . . . . . . . . . . . . . 3,675.56
1 Samuel Richards, capital........................ 37,416.27

To record the above assets and liabilities, all of which


except cash were taken over from Herbent Saunders,
and to open a double entry system of bookkeeping.

36
3 Herman Ridder (Accounts Receivable). . . . . . . . . . . . . . . . . . 88.70
16 Merchandise Sales . . . . . . . . . . . . . . . . . . . . . . . . . . . . . . 88.70
2 bbls. Flour... . . . . . . . . . . . . . . . . . $6.90 $13.80
1 bbl. Sugar (320 lbs.)........... .045 lb. 14.40
1 chest Tea (150 lbs.)............ .35 52.50
25 lbs. Unground Coffee......... .32 8.00
44
4 Notes Receivable .. .... . .. . . .. . .. ... .. . .. . .. .. . .. .. ... .. 89.15
3 Herman Ridder . . . . . . . . . . . . . . . . . . . . . . . . . . . . . . . . 88.70
17 Interest and Discount... . . . . . . . . . . . . . . . . . . . . . . . . .45
In payment of a/c—30-day note with 6% interest.
17—
15 Merchandise Purchases ................................. 835.00
12 X. Y. Z. (Accounts Payable). . . . . . . . . . . . . . . . . . . . 835.00

20 bbls. Flour................... 6.10 $122.00


10 bbls. Sugar (3200 lbs.)........ .04 128.00
10 chests Tea (1500 lbs.)........ .315 472.50
200 lbs. Coffee. . . . . . . . . . . . . . . . . . .285 57.00
4 bbls. Rolled Oats.............. 6.00 24.00
10 doz. Brooms................. 3.15 31.50

-
3 James Lawson (Accounts Receivable). . . .. . ... .. ... . .. .. 119.22
Merchandise Sale . . . . . . . . . . . . . . . . . . . . . . . . . . . . . . . 119.22

25 lbs. Coffee................... .315 $78.75


% bbls. Rolled Oats. . . .. . .. . .. . 6.35 3.17
1 bbl. Flour. . . . . . . . . . .. .. . .. . .. . 7.00
% chest Tea (75 lbs.) .. . .. ... ... .38 28.50
6 brooms . . ... . . . . .. . . . per doz. 3.40 1.70

57,426.38 57,426.38

CASH BOOK
(2)

Receipts (Left-hand page)

a/c Net Dis- Gen- Cash aſc Re


to

be
Date Credited Explanation L.F. Cash count eral Sales ceivable

1916
Jan. Balance 9,893.46 9,893.46
V
17 8 3

Moise. Sales Cash sales 346.78 346.78


James Lawson On a/c 15.00 15.00
Mdse. Sales Cash sales 492.83 492.83
Notes Rec. Disc. Ridder's note 8th 14 88.83 .32 89.15
a/c
25

Lawson 104.22 104.22


of

James Settlement
Moise. Sales Cash sales 526.38 526.38
Total Cash $11,467.50
Balance Jan. 9,893.46
3

Received $1,574.04 11,467.50 .32 9,982.61 1,365.99 119.22

- (2) (17) (V) (16) (3)


Feb. Balance 8,596.36 8,596.36
1

(Figures parentheses under columns are ledger folios. Check mark indicates no
in

posting required.)

37
(Right-hand page) CASH BOOK Disbursements (3)

a/c to be Net Dis- Gen- Ex- a/c


Date Debited Explanation L.F. Cash count eral pense Payable
1916
Jan. 3 Auto truck
From Utica Garage 10 645.00 645.00
Auto expense
50 gal. gas at 25c. 20 12.50 12.50
Office furn. & fix.
Safe $126, rug $40, type
writer $95 6 261.00 261.00
Postage & stationery
Desk sundries $32.16,
sta. & stamps $85,
books $75 21 192.16 192.16
8 Misc. expense
Sundry store supplies 22 25.48 25.48
17 Office furn. & fix.
Henderson & Robinson
balance due for en
larging office 6 195.00 195.00
X. Y. Z.
Cash purchase 835.00 835.00
31 Delivery salaries
Chauffeur and boy 19 130.00 130.00
Office salaries
Clerks, stenographers,
bookkeeper 18 575.00 575.00
2 (2,871.14) (1,101.00) (935.14) (835.00)
Balance 8,596.36 8,596.36

11,467.50 9,697.36 935.14 835.00

(2) (V) (V) (12)

Ques. 6:—Post the above items to the Ledger.

ANSWER:
Samuel Richards, Capital. (1)

Jan. 3 Balance J1 37,416.27

(2) Cash

1916 1916
Jan. 3 Balance J1 9,893.46 Jan. 31 Total disbursements C3 2,871.14
31 Total receipts C2 1,574.04
(8,596.36) (11,467.50)

Accounts Receivable (3)


1916 1916
Jan. 3 Balance J1 4,928.73 Jan. 3 Notes Rec. H. Rid
8 Herman Ridder J1 88.70 der J1 88.70
17 James Lawson J1 119.22 31 Total receipts C2 119.22
(4,928.73) (5,136.65)
(207.92)

(4) Notes Receivable


1916
Jan. 3 Balance J1 2,179.31
8 H. Ridder's note
and int. J1 89.15
(2,268.46)

38
(5)
Real Estate and Buildings

1916
J1
-
25,500.00
Jan. 3 Balance

(6) Office Furniture and Fixtures

1916
Jan. 3 Balance J1 2,628.75
“ Safe $126, rug $40,
& typewriter $95 C3 261.00
17 Henderson & Rob .
inson C3 195.00
(3,084.75)

Store Fixtures (7)

1916
Jan. 3 Balance J1 4,382.65

(8) Mdse. Inventory

1916
Jan. 3 Balance J1 6,483.26

Inventory (9)
Sundries

1916
Jan. 3 Balance J1 298.15

Auto Truck
(10)
1916
Utica Garage C3 645.00
Jan. 3.
*
Mortgages Payable (11)

1916
Jan. 3 Balance J1 10,000.00

(12) Accounts Payable


1916
1916 5,202.48
Jan. 3 Balance
Jan. 31 Total payments 17 X. Y. Z. J1 835.00
(X. Y. Z.) C3 835.00
(5,202.48) (6,037.48)

(13)
Notes Payable
1916 -
Jan. 3 Balance J1 3,675.56

Notes Receivable Discounted


(14)
1916
note 1/8/16 C2 89.15
Jan. 17 Ridder's

Mdse. Purchases (15)

1916
Jan. 17 X. Y. Z. J1 835.00

Mdse. Sales
(16)
1916
Jan. 8 H. Ridder J1 88.70
17 J. Lawson J1 119.22
31 Cash sales C2 1,365.99
(1,573.91)

39
Interest and Discount (17)

1916 1916
Jan. 31 H. Ridder's note C2 .32 Jan. 8 H. Ridder's
*: .13 )
J1 .45

(18) Office Salaries

1916
Jan. 31 Bookkeeper, clerks -
& stenographers C3 575.00

Delivery Salaries (19)

1916
Jan. 31 Chauffeur & boy C3 130.00

(20) Auto Expense


916
Jan. 3 Gasoline C3 12.50

Postage & Stationery (21)

1916
Jan. 3 Sundries C3 192.16

(22) Miscellaneous Expense

1916 -
Jan. 8 Sundries C3 25.48

Ques. 7:—Take off a trial balance as of January 31, 1916.

ANSWER:
Samuel Richards
Trial Balance, January 31, 1916.

L.F. - Dr. Cr.


1 Samuel Richards, Capital. . . . . . . . . . . . . . . . . ............... 37,416.27
2 Cash . . . . . . . . . . . . . . . . . . . . . . . . . . . . . . . . . . . . . . . . . . . . . . . . . . . 8,596.36
3 Accounts Receivable . . . . . . . . . . . . . . . . . . . . . . . . . . . . . . . . . . . 4,928.73
4 2,268.46
Notes Receivable.......: . . . . . . . . . . . . . . . . . . . . . . . . . . . . . . . . . .
5 Real Estate and Buildings. . . . . . . . . . . . . . . . . . . . . . . . . . . . . . . 25,500.00
6 Office Furniture and Fixtures. . . . . . . . . . . . . . . . . . . . . . . . . . . 3,084.75
7 Store Fixtures . . . . . . . . . . . . . . . . . . . . . . . . . . . . . . . . . . . . . . . . . . 4,382.65
8 Merchandise Inventory . . . . . . . . . . . . . . . . . . . . . . . . . . . . . . . . . 6,483.26
9 Sundries Inventory . . . . . . . . . . . . . . . . . . . . . . . . . . . . . . . . . . . . . 298.15
10 Auto Truck . . . . . . . . . . . . . . . . . . . . . . . . . . . . . . . . . . . . . . . . . . . . 645.00
11 Mortgages Payable
.. ... . ... .... . . ... . ... . .. .... ... . . ... 10,000.00
12 Accounts Payable .. .. . ..... . .. .. . .. .. . .. . .. . .. .. .. .. ... 5,202.48
13 Notes Payable .. .. . .. . .. . .. . .... .. . .. .. . ... .. .. ... .. . .. 3,675.56
14 Notes Receivable Discounted . . . . . . . . . . . . . . . . . . . . . . . . . . . . 89.15
15 Merchandise Purchases . . . . . . . . . . . . . . . . . . . . . . . . . . . . . . . .. 835.00
16 Merchandise Sales . . . . . . . . . . . . . . . . . . . . . . . . . . . . . . . . . . . . .. 1,573.91
17 Interest and Discount. . . . . . . . . . . . . . . . . . . . . . . . . . . . . . . . . .. .13
18 Office Salaries . . . . . . . . . . . . . . . . . . . . . . . . . . . . . . . . . . . . . . ... 575.00
19 Delivery Salaries . . . . . . . . . . . . . . . . . . . . . . . . . . . . . . . . . . . . . . . 130.00
20 Auto Expense . . . . . . . . . . . . . . . . . - - - - - - - - - - - - - - - - - - - - - - - - - 12.50
21 Postage and Stationery. . . . . . . . . . . . . . . . . . . . . . . . . . . . . . . . . 192.16
22 Miscellaneous Expense . . .. . .. . .. .. ... .... . .. ... ... .. . .. 25.48

57,957.50 57,957.50
Ques. 8:—Define: Account; Liability; 12. Contract, an agreement between two or
Present Worth; Surplus; Inventory. Give more persons; contract.
with figures an example of each. 13. Premium, consideration paid for con
insurance; premium.

of
tract
ANSWER: Fiscal, referring finances; fiscal.

to
14.
An Account Valuation, determination

of

of
is a record of transactions 15. worth
with a person, property or thing, having assets; valuation.
two sides, opposite in tendency and so ar Liquidation, settlement claims; liqui

of
16.
ranged as to show a definite result. All dation.
of the items in the foregoing trial balance striking out rejection;

or
17. Cancellation,
are accounts, for example, Cash, $8,596.36; cancellation.
Accounts Receivable, $4,928.73, etc. authorized proof ac

as
Certification,

to
18.
A Liability is an obligation or debt of a curacy; certification.
business or individual. Examples found in 19. Equipment, machinery and tools essen
the foregoing trial balance are: Mortgages factory; equipment.

to
tial

a
Payable, Accounts Payable, $5, Fluctuation, variations

to
$10,000; value due

in
20.
202.48, etc. extraneous causes; fluctuation.
Present Worth is the net capital of an
individual at a given time. The present LETTER-WRITING.
worth of Samuel Richards on January 3,
1916, was $37,416.27, as per the foregoing Directions: Write letter of not less

a
trial balance. than 120 words, but not more than 150
Surplus represents the excess of assets of words, to the State Civil Service Commis
a corporation over liabilities and capital, sion, Albany, Y.,

on
N.
the follow

of
one
which is reserved from, but subject to, dis 111g topics:
tribution. Advantages and disadvantages

of
the
1.

Example: loose-leaf ledger system;


The advantages the private busi

of
2.

Total Assets . . . . . . . . . . . . . $115,000 compared with the free pub

as
ness school
Total Liabilities . . . .. ... .. 50,000 lic schools of the same kind.
Sign your examination number, not your
Net Assets . . .. . . .... .. ... $65,000
You are liberty make prelim
at

to
name.
Capital Stock .. ... .. . . .. . 50,000 inary drafts on other paper before putting
the final work on the white paper furnished.
Surplus . . .. . .. . ---------- $15,000
In marking the letter, form and address,
paragraphing, spelling, punctuating, use

of
Inventory is a schedule of property, or capitals, syntax, style and treatment sub

of
goods belonging to a business. The Mdse. ject will be considered.
Inventory on January 3, 1916, in the busi
ness of Samuel Richards was $6,483.26.

SPELLING. State of New York—The Civil Service


Commission.
The examiner will read aloud to the class
the following instructions: BOOKKEEPER, 4th GRADE.
The word will be first pronounced, then (Salary,
or

720, $600 and Maintenance.)


defined or exemplified, then pronounced
again, after which the candidate is required Date: October 14, 1911.
to write Begin all words with capital
it.

letters. SPELLING.
The examiner will read aloud
to

Currency,lawful money; currency. the class


U.
S.
. .

Administrative, referring the following instructions:


to

the conduct This exercise


of business; administrative. Spelling.
20

The word
of

consists words
in
a

Accumulation, additions; will be first pronounced, then defined or


of

sum-total -
.

accumulation. exemplified, then pronounced again, after


Apportionment, disbursement according required
to

it.

which the candidate write


is

previous estimate; apportionment. Begin all words, with capitals: Notary,


to

Disbursements, payments cash; dis Genuine, Visual, Maturity, Mucilage, Com


of

bursements. mercial, Verify, Acknowledge, Schedule,


Replenish, replace withdrawals; re Marginal, Depression, Encircle, Transac
to
.

plenish. tion, Government, Accepted, Statute, Filing,


Deficiency, margin loss between as Obvious, Assessment, Development.
of
.

sets and liabilities; deficiency.


Systematic, orderly and methodical TEST IN RAPID ADDITION.
in
.

manner; systematic.
Verification, test accuracy rec Instructions Do not turn
to

candidates:
of

of
a
.

ord; verification.
so

this sheet over until the signal


to

do
is

10. Periodical, occurring regular inter given by examiner. The exercise consists
at

vals; periodical.
as

figures. Add rapidly


as
of

of

columns
11. Negotiable, capable transfer by en you can, commencing the right. You
of

at

dorsement; negotiable. will be allowed exactly seven minutes


to

41
complete this exercise. At the signal to LETTER-WRITING.
stop you must turn this sheet over at once.
N. B.-Examiners shall call candidates' Write in proper form a good business let
attention to the above instructions and al ter, addressed to The State Civil Service
low them time to read the same and fill in Commission, applying for the position for
the blanks at the top before giving the which you are being examined and setting
signal to commence. forth your training, experience and other
qualifications for the position.
Test in Addition. Caution:—The competitor should date
the letter at the city of the examination,
497,864 35 84,596 49 984,576 65,984 36 and should address it as follows: State
578,797 49 9,864 57 49,873 97,797 56 Civil Service Commission, Albany, N. Y.
986,874 98 36,699 85 536,574 9,647 79 He should not refer to his political or re
695,375 47 49,589 36 825,639 58,768 98 ligious affiliations or opinions, and should
854,954 39 47,764 59 87,426 93,642 37 use his examination number and not his
845,969 98 66,498 87 594,327 78,598 54 name as a signature. Use pen and ink. The
574,887 93 77,364.65 36,298 9,716 main purpose of this exercise is to test skill
43
931,546 75 36,952 17 84,963 8,489 97 in simple English composition. In rating
497,398 66 79,378 57 57,748 36,576 48 the letter consideration will be given to
848,925 95 58,785 89 376,694 49,376 59 errors in form and address, in spelling, cap
674,297 48 96,374 67 896,577 48,697 16 italization, punctuation, syntax and style,
and its adherence to the subject.
648,596 47 948,897 789,566 87
974,685 86 375,864 496,385
95,246 78 59,897 883,465
65 COMMERCIAL PRACTICE AND
84
378,645 35 46,375 997,936 49
ARITH METIC.
67,683 97 93,846 475,743 69

all
(Show steps employed obtaining

in
595,483 64 978,624 964,736 57 answers.) -
374,759 57 87,926 726,437 94

on
Find the simple interest

at
$1,366
1.

694,877 69 384,629 642,756 58


4% per cent from January 1910, until

2,
89,974 86 765,782 749,837 48 to-day.
37,597 48 997,695 876,544 57
has lots worth $750 each; has
2.

8 B
4
476,874 35 762,498 644,675 38
lots worth $1,050 each, and has lots

C
6

worth $1,400 each. required

to
divide

ofIt
is
PENMANSHIP. among them tax $194.40 for street im
a

provement. Find the amount each should


N. B.-The rating on Penmanship will be pay.
determined by legibility, rapidity, neatness, An automobile requiring gallons

of
3.

2
.

and general appearance, and by correctness gasolene for miles, starts with
35
of

run
20 40 a

and uniformity in the formation of words, supply gallons. additional sup


15,of

If
3
a

letters, and punctuation marks.


25

plies gallons respectively


of

and
gallons are still
of 12

are taken on and

in
if

the tank the end the trip, how long


at

COPYING FROM PLAIN COPY. was the trip?


N. B.-Paragraph, house insured for $6,528; the pre
4.

is

spell, capitalize, and


A

mium paid Find the rate of in


is

$48.96.
punctuate precisely as in copy. All omis Surance.
sions and mistakes will be taken into con
sideration in rating this subject. Use pen
5-6, ...Compute and make out the follow
.

ing bill proper form for presentation:


in

and ink. Penmanship will be rated on this


Frisbee bought Leon
D.

John
of

Stevens
&

exercise.
ard, Albany, Y., September 30, 1911,
N.

Make an exact written copy of the fol


3 1

doz. hhds. hind.;


of

at

lowing: molasses $15


a

lb; herring
of
2c
a of

cwt. lime boxes


at

Open competitive examinations were re


9
a

at

dozen boxes; gross


$9

of

candles
5c at

quired by law, intending that “appointment


7

dozen; butter, each contain


19

of

or promotion shall be given to the man tubs


a

ing 28% lbs.,


at

25c 1b.; 200 Oranges at


who is best fitted to discharge the duties of
a

special discount per cent


10
of

doz.
A

the position, and that such fitness shall be 10c


a

allowed, and per cent off for cash.


is

ascertained by open, fair, honest, impartial,


5

competitive examination. The impartiality Define fully the following: Accept


7.

ançe, consignment, insolvency endorsement,


of these examinations is to be secured by lading.
bill
of

every possible safeguard; they will be the


Explain the meaning the following
of

test of the applicant for the particular place


8.

signs and abbreviations: C.O.D., F.O.B.,


to which he aspires.” It was the inten Ltd., disct., prox.
tion of Congress to place all citizens of the
United States on an equal footing and to BOOKKEEPING.
give all an opportunity to demonstrate their
fitness to serve the Government. This was Post the following transactions
to

1-4.
a

brought out as a prominent feature in the double entry ledger: John Budd
2,

October
debate upon the civil-service law.—(Four invested business, cash $2,000; October
in

teenth Annual Report of the United States boºght Henry Kent on note
at
3,

of

10
Civil Service Commission, p. 15.)
4,

days, 500 bu. 85c.; October


of

wheat
at

42
º: º
bought of P. Dunn for cash, 500 bu. of corn

;
Ques. 1:—A man bought

50
sha
at 60c.; October 5, sold David Dix on ac

ſº
120%, and six months later

at
stock
count 200 bu.. wheat at $1.15; October 6, Peſ, cent dividend was declared. He then
sold Adams & Co. on their note at 10 days brokerage
§:each case

in
bu.. wheat at $1.14; October 7, sold

i.
200 per cent. Find his net gain
gai

or
loss, money

8
David Dix for cash 100 bu. wheat at $1.14; being worth per cent.

5
October 9, paid rent of store in cash, $30;

Ҽ,
October 10, bought of Henry Kent on ac SOLUTION:
count 600 bu. wheat at 90c.; October 11,

. . at
sold David Dix on account 200 bu.. wheat Shares—50
at $1.14; October 12, sold Adams & Co. 120% 6,012. 50

$
.

.
of at..
3 .
.
.
.
.
.
.
. .
. .
. .
. .
400 bu. of corn at 85c., receiving $200 cash,
Dividend 2/3% $183.33
the rest on account; October 13, paid Henry Proceeds Sale. 900.00
v vv.

5
.
.
.
.
.
.
.

;
Kent note of October 3 in cash; October Brokerage 12.50

.
.
.
.
.
.
.
.
.
.
.
.
.
.
14, paid rent of store in cash, $30; paid Interest 15031

.
.
.
.
.
.
.
.
.
.
.
.
.
.
.
.

-
wages, $35; October 16, received of Adams Loss 91.98

.
.
.
.
.
.
.
.
.
.
.
.
.
.
.
.
.
.
.
& Co. amount of note of October 6 in cash.
5–7. Make out a complete final balance $6,175.31 $6,175.31
sheet or statement for the above, showing
resources and liabilities, losses and gains,
present worth of business and net loss or 91%
amount,
2:-At..what rate will $856.50

in
to
$748.41 years

at
gain. months
Moise. inventory is $450.

6
simple interest?
8. Write the notes mentioned in the
transactions of October 3 and 6. SOLUTION:
9. Make journal entries in the books of 656.50)748.41(1.14
each party to the following draft: The 656 50.

*-
drawer is George Thomas, the drawee is
Brown Bros., the payee is Dundy & Co. It 91 910
is drawn at sight October 5 for $1,265. 65 650
10. Describe three ways of endorsing a
check and explain the force of each. 26 26 00
26 26 00
State of New York—The Civil Service
Commission. 114%–100%=14% interest
(Ans.)
BOOKKEEPER.
Fifth and Sixth Grades.
(Salary, $721 to $1,200.)
- Ques:- 8:-A, having lost per cen of
as 25
y

Date: October 14, 1911.


h:
t
his capital, now has who
º
as

much
PART I–SPELLING.
on

Just gained per cent


15

his capital. B's


original capital was $5,000. what was A’s?
The examiner will read aloud to the class
the following instructions: SOLUTION:
This exercise consists of 20 words in 100%= $5,000
Spelling. The word will be first pronounced,
=

15% 750
then defined or exemplified, then pro
and B's present
A

nounced again, after which the candidate is capital $5,750


required Begin all words with
it.

to write
capital letters. $5,750=3; of
A*

Original
igi
pi

Ca ital.
z

Collector, Acceptable, Co
S

Commercial,
s l

4/3X$5,750=$7,666.66 A’
Ju Original
A

cap ital
*

operation, Salaries, Eatables, Monetary,


t
)

dicious, Mileage, Consensus, Collateral,


Unanimous, Commodities, Accountancy, Al

...
lotment, Receipted, Acknowledgment, Su
persede, Recommend, Insolvency. Ques. 4:—A certain -
iece of pro
s”;

assessed $18,785, Whi.


at

is
3

PART II—TEST IN RAPID ADDITION.


its

less than assessment the preceding year


when the tax paid upon was $58.50. What
it

This exercise was the same that given


as

of

was the rate taxation that year?


for Bookkeeper, 4th Grade, October 14,
1911, which will be found on the preceding SOLUTION:
page. $18,785–96 1/3% last year.
of
by

to

..Multiplying clear fractions we


3

PART III—COMMERCIAL PRACTICE


AND ARITH METIC. 289)56,355
-
all

(Show steps and necessary computa 19,500


.

tion employed obtaining answers.)


in

on

$58.50=3% $19,500.

43
Ques. 5:—An agent sold goods at a com terprise for a period of years over and
mission of 5 per cent through a broker who above the normal rate of return in that
charged him 2 per cent, and the agent's com particular line of business. It may be due
mission after paying the brokerage was $315. to various causes, such as superior service,
How much did the agent remit to his em quasi-public monopoly, advertising, location,
ployer? etc.
SOLUTION: An Injunction is a decree issued by a
court of equity upon application by one
$315=3%. 1%=$105. party, ordering another party to do or to
Agent remitted 95% =$9,975. refrain from doing a certain act.
Ques. 6:-After getting a note, without Ques. 8:—In about 150 words discuss the
interest, discounted at a bank for 3 months present tendency in the business world to
at 6 per cent, I had $354.60. What was the ward combination, its advantages and dan
face of the note? gers, and the present attitude of the Gov
SOLUTION: ernment toward combinations, as deter
mined by the Federal Supreme Court.
$354.60=98%%. 1%=$36.
Amount of note=100%=$360 (Ans.). Ques. 9 and 10:—Write the following
Ques. 7:—Explain clearly and fully the draft and acceptance and make journal en
following: Liquidation, corporation, wa tries in the books of each of the parties
tered stock, syndicate, assignment, fran concerned: David Rose received from Sid
chise, debenture, assessment, good-will, in ney Deane on account a draft for $750 at
Junction. 30 days' sight, drawn October 3rd in Sidney
Deane's favor by Brown Brothers and ac
ANSWER: cepted by Cole & Wood.
Liquidation—In general the closing trans
actions of a commercial house incident to ANSWER:
its retiring from business; the payments of New York, Oct. 3, 1911.
debts or liabilities.
Thirty days after sight pay to the order
A Corporation is a business entity created of Sidney Deane the sum of Seven Hundred
by the State and existing by grace of the
State, enjoying such privileges and subject and Fifty Dollars ($750.00), and charge to
account of BROWN BROS.
to such liabilities as prescribed by statute.
Usually the most important privilege is the To Cole & Wood,
immunity of the stockholders from person New York.
al liability beyond the payment for the stock Accepted: Cole & Wood, Oct. 4, 1911.
issued to them. The State exacts a tax
from every corporation and limits their ac Books of Sidney Deane.
tivities to the business specifically provided David Rose
for in the charter or certificate of incorpora To Bills Receivable.
tion.
Watered Stock—Stock issued in excess For 30-day sight draft for $750, accepted
by Cole & Wood, due Nov. 4, 1911.
of the actual value of property. Given by us to David Rose on ac
Syndicate—A combination of independent Count.
operators for the purpose of promoting an
enterprise for profit. The syndicate usually Books of David Rose.
guarantees the success of the project, in
Bills Receivable
return for which it receives a fixed percen
tage of the amount involved. To Sidney Deane.
Assignment—The transfer of a chose in For draft of $750 accepted by Cole &
Wood, in favor of Sidney Deane, due
action by the holder to another as his at
torney. An assignment does not give rise Nov. 4, 1911.

to any new rights in the assignee. He takes


it subject to all defenses which could be PART IV—BOOKKEEPING.
made against the assignor.
Franchise is a right granted by the Gov Ques. 1:—Explain fully the workings of
ernment to operate a public utility. the voucher system of accounting.
Debenture—A form of unsecured bond ANS.:—The voucher system of account
ing is one which provides for the enter
issued by a corporation on which the inter
ing of invoices and records of payment
est does not accrue unless earned by the
obligor. on the same page. It eliminates the use
of an Accounts Payable ledger. Invoices
Assessment—A levy made against stock payable are entered in a voucher register.
holders of a corporation above the par As these are paid the payment is recorded
value of the stock, after the stock had been
in a column on a line with the original
paid for in full. It is usually made only entry. Sometimes the date of payment is
where the corporation has encountered shown instead of the amount due. Accounts
financial difficulties. payable can be ascertained by adding the
Good-will—An intangible asset, usually amounts credited, and deducting the sum
measured by the earnings of a business en of payments.

44
Ques. 2 and 3:—A merchant began the SOLUTION:
year with $2,400 liabilities. Since that time
A's Account.
he has gained $1,150. At present he has
$1,534.72 in cash, $850 worth of merchan Mar. $800.00 Jan. $15,000.00

1
dise, and he owes on notes $670. What June 15 500.00 Dec. 31 Int. 697.72
were his resources at the beginning of the Oct. 350.00

1
year? Show how you reached your result. Dec. 31 Bal. 14,047.72
SOLUTION: $15,697.72 $15,697.72
Assets–Cash . . . . . . . . . . . . . . . . . . . . . $1,534.72
Mdse. . . . . . . . . . . . . . . . . . . . . 850.00 Jan. Bal. $14,047.72

1
Total . . . . . . . . . . . . . . . . . . . . . . . . $2,384.72 B’s Account.
Liabilities—Notes . . . . ... . . . . . . . . . . 670.00 May $1,200.00 Jan. $7,500.00

1 1 1

1 1
Sept. 500.00 July 7,500.00
Net Worth .......................#1,714.72 Oct. 500.00 Dec. 31 Int. 507.92
Gain in Year. . . . . . . . . . . . . . . . . . . . . . 1,150.00 Dec. 31 Bal. 13,307.92

Net Worth at beginning. . . . . . . . . . . $564.72 $15,507.92 $15,507.92


Liabilities at beginning............ 2,400.00
Jan. Bal. $13,307.92

1
Assets to balance Liab. & Capital.. $2,964.72
C’s Account.
Ques. 4:—(a) What precautions and pre Mar. 15 $250.00 Jan. $5,000.00

1
liminary steps should a bookkeeper take on July 350.00 Dec. 31 Int. 230.90

1 1
first assuming charge of a set of books? Dec. 200.00
(b) Name five kinds of ledgers and state Dec. 31 Bal. 4,430.90
briefly the use of each.
ANS.:—(a) The bookkeeper should see $5,230.90 $5,230.90
that the general ledger was in balance at
the time of the taking of the last trial bal Jan. Bal. $4,430.90

1
ance. Where there were ledgers controlled
by accounts in the general ledger, see that Interest Statement—A.
such ledgers agreed with the general ledger 5% on $15,000 capital for year. $750.00

1
accounts; that the bank reconciliation

.
.
.
was Less:
correctly made, and that the check book 5% on $800 withdrawal for
agreed with, or was reconciled with, the 10 mos. $33.33
.
.
.
.
.
.
.
.
.
.
.
.
.
.
.
cash book. .
5% on $500 withdrawal for
(b) 1. Customers' Ledgers—For keeping 6% mos. 14.58
with customers.
.
.
.
.
.
.
.
.
.
.
.
.
.
.
.

accounts
5% on $350 withdrawal for
2. Creditors' Ledger—For keeping ac mos. 4.37 52.28
3

.
.
.
.
.
.
.
.
.
.
.
.
.
.
.
.
.

counts with creditors.


3. Bills Receivable Ledger—For keeping $697.72
accounts of notes and bills receivable (rare).
4. Bills Payable Ledger—For keeping Interest Statement—B.
accounts of notes and bills payable (rare).
5. Private or General Ledger—Ledger 5% on $7,500 capital for 12
in which are kept all asset, liability and
In OS. $375.00
.
.
.
.
.
.
.
.
..
.
.
.
.
.
.
.
.
.

capital accounts which form the basis of 5% on $7,500 capital for


. 6

the balance sheet and income statement. In OS. 187.50 $562.50


.
.
.
.
.
.
.
.
.
.
.
.
.
.
.
.
.
.

There are various forms and rulings in


ledgers, such as: Less:
1. Standard debit and credit columns. 5% on withdrawal
$1,200
2. Boston debit, credit and balance col
formos.. 0.00
8

.
.
.
.
.
.
.
.
.
.
.
.

umns. 5% on $500 withdrawal for


system. mos. 8.33
4

3. Card
.
.
.
.
.
.
.
.
.
.
.
.
.
.
.
.
.

4. Voucher Register-Ledger. 5% on $500 withdrawal for


mos. 6.25 54.58
3

.
.
.
.
.
.
.
.
.
.
.
.
.
.
.
.
.

Ques. 5 and 6:—A, B and C are partners.


A invested January 1, $15,000; withdrew $507.92
March 1, $800, June 15, $500, and October
January 1, $7,500, July
Interest Statement—C.
1, $350. B invested
l, $7,500; withdrew May 1, $1,200, Septem 5% on $5,000 capital
12

for mos... $250.00


ber 1, $500, and October 1, $500. C invested Less:
January 1, $5,000; withdrew March 15, $250, 5% on $250 withdrawal for
July 1, $350, December 1, $200. Each was 9% mos. $9.50
.
.
.
.
.
.
.
.
.
.
.
.
.
.
.

to receive interest at 5 per cent on capital 5% on $350 withdrawal for


invested. Make the necessary entries in mos. 8.75
6

.
.
.
.
.
.
.
.
.
.
.
.
.
.
.
.
.
the

partners' accounts, including interest 5% on $200 withdrawal for


year. Prepare an interest mo.
at

ue,
of

close .85 19.10


1

.
.
.
.
.
.
.
.
.
.
.
.
.
.
.
.
.
.

statement showing how you arrived


at

the
interest due each partner.
of

amount $230.90

45
Ques. 7-10:—Three partners, A, B and C, Partners' Capital Accounts.
begin the year with a capital of $72,000, of
which $40,000 is invested by A, $20,000 by Original Share of With- Net
B and $12,000 by C. They take 5 per cent Investment Profits drawals Balance
per annum interest on capital, and they di $40,000.00 $10,734.68 $1,520.00 $49,214.68
vide profits or losses according to their re 20,000.00 5,367.34 1,200.00 24,167.34
spective proportions of the capital. At the 12,000.00 3,220.41 1,175.00 14,045.41
end of the year their trial balance is as fol
lows: Debits: Land, $5,000; buildings, $20, $72,000.00 $19,322.43 $3,895.00 $87,427.43
000; machinery, $6,600; investments, $10,500;
good-will, $20,000; drawing account A, $1,
520; drawing account B, $1,200; drawing A, B and C
account C, $1,175; cash in bank, $3,500; cash
in safe, $377.47; bills receivable, $1,790.50; Income, Profit & Loss Statement
accounts receivable, $7,260.22; purchases,
$37,070; salaries and wages, $12,500; dis For the year ended December 31, 1910.

counts, $125; bad debts, $1,125; sundry ex Income from Sales. . . . . . . . . . . . . . $59,750.00
penses, $13,012.57; total, $142,755.76. Cred Cost of Sales:
its: Partners' capital, $72,000; bills payable,
$2,600; accounts payable, $8,405.76; sales, Purchases . . . . . . . . . . $37,070.00
$59,750; total, $142,755.76. The value of the Salaries & Wages... 12,500.00
goods on hand at the end of the year is
$24,000. Prepare their balance sheet or $49,570.00
statement, their profit and loss account and Less Inventory, Dec.
capital accounts. Charge no interest on 31, 1916 . .. ... ... . 24,000.00
drawings; allow 2 per cent per annum de
preciation on buildings, 7% per cent on ma Total Prime Cost... $25,570.00
chinery, and 6 per cent appreciation on land. Dep. of Mach . . . . . . . 495.00

A, B and C Total Cost of Sales. . . .. ... ... 26,065.00


Balance Sheet as at January 1, 1911.
Assets. Gross Profit on Sales. . . . . . . . . . . . $33,685.00
Sundry Expenses . . . . . . . . . . . . . . . 13,012.57
Fixed Assets: -
Land . . . . . . . . . ... .. . .. $5,300.00
Buildings . . . 20,000.00 Net Income from Operations. . . . $20,672.43
. . . .. . .. .
Machinery . . 6,600.00
Deduction from Income—Discts. 125.00
. . . . . . . . .
Investments .. . . . . . .. . 10,500.00
$20,547.43
Good-will . . . . . . .. . .. . 20,000.00
Profit & Loss Charges:
Bad Debts. . . . . . . . . . . . $1,125.00
$62,400.00
Dep. on Buildings. . . . . 400.00 1,525.00
Working & Trading Assets:
Inventory $24,000.00 24,000.00 $19,022.43
Profit & Loss Credit:
Current Assets: *Appreciation of Land. . ... . .. * 300.00

Cash-In Bank. $3,500.00 Net Income—Profit & Loss. . . . . $19,322.43


In Safe... 377.47 $3,877.47 Drawings:
A . . . . . . . . . . . . . . . . . . . . $1,520.00
Bills Receivable. . . .. .. 1,790.50
B . . . . . . . . . . . . . . . . . . . . 1,200.00
Accts. Receivable... . .. 7,260.22
C . . . . . . . . . . . . . . . . . . . . 1,175.00
Total Current Assets. . . ... . ... 12,928.19
Total Drawings .. .. . .. . ... . 3,895.00

Total Assets . . . . . . . . . . . . . . . . . $99,328.19


Net A mount carried to Cap. a/cs. $15,427.43

Liabilities.
*NOTE:—It will be noticed that appre
Current Liabilities: ciation of land is credited to Profit and Loss
Bills Payable. . . . . . . . . . $2,600.00 and therefore distributed to partners as
Accts. Payable. . . . . . . . 8,405.76 profits. This is contrary to established
practice, the custom being to carry such
Total Current Liabilities. . . . . . . $11,005.76 appreciation in a reserve account until the
Reserve for dep. of Bldgs. . . . . . . 400.00 property is sold, no distribution being made
Reserve for dep. of Mach . . . . . . . 495.00 of profits in the meantime. Some account
ants, however, advocate the method used
Total Liabilities & Reserves. . . $11,900.76 in the foregoing, although the practice of
setting up a reserve account may be consid
Net Worth . . . . . . . . . . . . . . . . . . . . . $87,427.43 ered more conservative.

46
BookREEPER, UNITED STATES SERVICE.

Answers to Examination Questions and Specimen


Questions.

United States Civil Service Commission. mony: An estate inherited from one's
father. Certificate: written testimony; as,

A
BOOKKEEPER. marriage certificate. Reservoir: place

A
A of a
storage; as, water reservoir. Privilege:
(Usual Salary,

a
$1,000.)
right; as, the privilege voting.

of
1. Spelling: Twenty words of more than
average difficulty; weight 10. ARITH ETIC.

M
2. Arithmetic: Fundamental rules, frac
tions, weights and measures, percentage, in the following problems, give
solving

In
terest, discount, commission, custom-house the work full, showing all the figures and

in
business, stocks and bonds, partnership, mathematical signs necessary for the solu
analysis, and statement of simple accounts; each problem, without any unneces

of
tion
weight 15. sary figures. Write the word “Answer” or
3. Penmanship: Rated on legibility, rap its abbreviation, “Ans.”, after the answer
idity, neatness, and general appearance; each problem.
weight 10. to
4. Report Writing: Test in writing in Ques. 1:—The following table shows the
letter form an orderly, concise, and gram largest cities New York State
12

1910,

in

in
matical statement of the essential facts in with their population 1890, and the in

in in
cluded in a given statement of 400 to 500 crease (or decrease) population the

in
words; weight 15. two following decades. Find the population
in 1910 of each

of
5. Copying and Correcting Manuscript: the cities mentioned and
Test in making a smooth, corrected copy the total of all the cities:
of a draft of manuscript which includes (Dec. denotes decrease.)
erasures, misspelled words, errors in syn

a
tax, etc.; weight 10. Pop. Increase Increase Pop.
City
in

in

in

in
6. Practice of Bookkeeping, embracing 1890 1900 1910 1910
tests in journalizing and preparation of bal New York. 2,507,414 929,788 1,329,681

.
.
.
.
.
.
ance sheet in mercantile transactions; Buffalo ... 255,624 96,763 71,328

.
.
.
.
.
.
weight 40. Rochester 133,896 28,712 55,541
... .

.
.
.
.
.
.
The following questions and tests, which Syracuse S8,143 20,231 28,875

.
.
.
.
.
.
have been used, indicate tº e general char Albany .... 94,923 Dec.772 6,102

.
.
.
.
.
.
acter of these subjects: Yonkers 32,033 15,898 31,872
..

.
.
......
.
.
.
.
Troy 60,956 Dec.305 16,162
.
.
.
.
.
.

Utica 44,007 12,376 18,036


SPELLING.
.
.
.
.
.

.
.
.
.
.
.
Schenect'y. 19,902 11,780 41,144
.
.
.
.
.
Bingh'mt'n. 35,005 4,642 8,796 .
Spelling is dictated by the examiner. The
.
.
.
.
.
.
Elmira 30,893 4,779 1,504
words are written by the competitor in the

--
.
.
.
.
.
.

Auburn ... 25,828 4,516 4,324


blank spaces indicated on the first sheet of
.
.
.
.
.
.

the examination. All words should be com


Total population
menced with capital letters. The examiner
.
.
.
.
.
.

pronounces each word and gives its defini SOLUTION:


tion. The competitor is required to write Pop.
in

only the word and not its definition.

——
1910
Cylinder: A long round body. Promis 2,507,414 94,923 19,902 4,766,883
sory: Containing a promise; as, a promis 929,788 Dec. 772 11,780 423,715
sory note. Essential: Necessary or indis

-
1,329,681 41,144 218,149
pensable. Discernible: Apparent or visible. 94,151 137,249
Opportunity: A or convenient time. De
fit

4,766,883 6,102 72,826 100,253


tricky. Deference: Respect
or

ceitful: False 79.803


regard. placing
of

Insertion: The act


or

100,253 76,813
in; as, the insertion of an advertisement. 74,419
Facilitate: To make easy; as,
or of to

facilitate 255,624 32,033 35,005 72,826


business. Schenectady: city the United
A

96,763 15,898 4,642 48,443


States. Adjacent: Lying near bordering 71,328 31,872 8,796 37,176
on. Souvenir: token of remembrance.
A

34,668
Conceding: Yielding giving up; as, con
or

423,715 79,803 48,443


ceding point. Lineage: Line
of

descent etc., etc.


a

6,070,397
or ancestry; as, royal lineage. Deleteri
of

ous: Harmful injurious; as, deleterious NOTE:—Only the figures col


or

the last
in

health. Horizontal: On level. Patri umn are required,


to

47
Ques. 2:—Multiply 3 9/16 by 20.73, divide Ques. 4:—A public square is surrounded
the product by 6.91 and to the quotient add by a walk which contains an area of 1 acre
the difference between 734 and 98 7/125. and is 2 rods wide, the walk being entirely
Change all common fractions to decimals outside of the square. What is the area of

-
and solve by decimals. the square? 1 acre-160 square rods.
SOLUTION:

-
16) 9.0000(.5625
80
3.5625 SOLUTION:
100 20.73
96 2 rods
106875 2 walk 2 2 rods
40 24.9375
32 71.250 2 | 18 rods 2 4 sq. rods each

--
corner
80 73.850625 Park 4 corners
80
2 | 18 rods 2 16 sq. rods
6.91)73.850625 (10.6875
691 2 walk 2

-
4750
4.146 98.056
7.75
6046 160 sq. rods of walk
90.306 16 sq. rods in corners
4)3.00 5528
10.6875
.75 5182 4)144 sq. rods side pieces
4837 (Ans.) 100.9935
2)36 sq. rods each side
125)7.000 3455
3455 18 rods long each side of park
18
.056

Ques. 3:—New Brunswick has 66,224 pu 144


pils enrolled in her elementary schools out 18
of a population of 397,344. Saskatchewan
has 36,225 pupils enrolled out of a popula 324 sq. rods in park
tion of 289,800. If the same per cent of the
population were enrolled in Saskatchewan 160)324
that is enrolled in New Brunswick, would
Saskatchewan have to increase or decrease 2 1/40 acres (Ans.).
her present enrollment, and by how many
per cent of the present enrollment?
SOLUTION:
397344)66224.00(.16 2/3
397344 Ques. 5:—Make in the given form an
itemized statement of the following account
2648960 as it should appear taken from the books of
238,4064 Vogel & Son; make a proper heading; close
the account; and bring down the balance
264896 as it should have appeared June 1, 1911:
16 2/3=1/6 = 2/3 During the month of May, 1911, Vogel &
397344 Son had the following transactions with
Benton Van Riper: May 1, he owed them
36225)12075.00(.33 1/3 increase on account $59.80. May 2, he gave them his
108675 (Ans.) note due in 10 months for $42, receiving
credit for its present worth, $40. May 4,
120750 he sold them 64S pounds pork at 12% cents
108675 per pound. May 5, he bought of them 216
bushels wheat at 97% cents per bushel. May
12075 22, he sold them 75,850 bricks at $9.40 per
= 1/3 thousand. May 16, he transferred to them
36.225 by endorsement a note given him by John
Doe; face of note, $900; accrued interest to
6).289800 date, $36. May 23, he bought of them 880
pounds pork at 12% cents per pound, agree
48300 pupils=16 2/3% ing to pay freight also at 16% cents per 100
36.225 pupils enrolled pounds, the freight to be prepaid by them.
May 31, they sold him 14,560 pounds coal
12075 would have to increase (Ans.). at $6.90 per ton of 2,240 pounds.
48
soLUTION:
Benton Van Riper In account with Vogel & Son
1911 1911
May May

10
1 Balance on a/c. . . . . . . . $59.80 Note due mos. $42

2
5 216 bu. wheat (a) $.97%. 210.60 present worth........ $40.00
pork

(a)
23 880 lbs. pork (a $.12%. 106.70 648 lbs. $.12%.. 81.00

16 4
Freight on pork .16% c. 1.45 Note John Doe by

of
endorsement ........

. (a)
31 14,560 lbs. coal $6.90 900.00
2,240 lbs. 44.85 Accrued int. on note... 36.00

.
.
.
.
.
.
.
.
.
.
.
31 Balance on aſc. 1,346.59 22 75,850 bricks $9.40 M... 712.99

.
.
.
.
.
.
.
$1,769.99 $1,769.99

June Balance (brot. down). $1,346.59

.
(NOTE: Do not carry less than 1c.)
to
T

Frt. amounts $1.452.

have had some success. now looks like

It
NOTE:—This specimen
of
official
is

an

to
the type Question the regular First they will limit its usefulness cauterizing
of

Inin
5

Grade Clerical test. the Bookkeeper or burning; for there lots of cases where

is
the ordinary methods cauterizing fails

of
examination probably eliminated and
is
it

another arithmetic question substituted, because they can't get the diseased parts

to
in

and the radium rays can, as

of
problems
of

the case

in
view the
in

the more elaborate


bookkeeping branch the Bookkeeper test. inaccessible tumors or cancers which are
of

very difficult get to; and which the radi

to
PENMANSHIP. um rays has been quite effective

to
remove
The rating on Penmanship will be deter Radium rays, however, like the

in
them.
mined by legibility, rapidity, neatness, and case of the X-rays, sometimes causes se
general appearance, and by correctness and vere and practically incurable ulcers the

if
uniformity words, let application these rays are kept up too
of

of

the formation
in

ters, and punctuation the exer long, and the ulcers formed this way are

in
in

marks
the fourth subject—Report Writing. very difficult cure. This was first dis
of

to

cise
No particular style penmanship pre covered by an experimenter who carried
of

is

in a
tiny vial
of
ferred. radium around
of

the salts
his pocket very careless and caused ulcers it
REPORT WRITING. very

be
on his body; from which we should
it;

Directions the competitor: and they now keep


to

Condense
of

careful the use


in

the printed exercise into not more than 200 lead because the rays from
of

bottles
in
it

words, retaining all the facts. No effort the radium can't pass through them.

It
follow the language
of

they could isolate


to

should be made has been said that


if

a
the text. - pound radium and place
of

room that
in
it

In rating this exercise the arrangement, no human being or animal couldn't remain
that room and live; which shows that
of

completeness, exactness, and conciseness


in

the statement, its adherence the subject the ravs must be very strong. But perhaps
to

matter, its style, and freedom from errors there isn't so great supply
of

in

radium
a

grammar, etc., will be considered. The the world as this, or there is, no one
in

if

exercise follows:
of

hasn't discovered yet. The question


it

The element which cost perhaps more the possible distribution radium through
of

than any other element the world and out the earth's crust and perhaps
in

also
it

also the rarest radium and the sun and the stars have caused
is

exists
in
is
is

it
it

characterized by the ability send out very some interesting speculations by scientists
to

strong rays; which has much common concerning


in

it.

with the Roentgen X-rays, and resembles


or

them very much, though they aren't the


X-rays.
ANSWER:
The ones that first discovered
radium was Professor and Mme. Curie of Radium, discovered by Professor and
Paris; and we obtain radium now chiefly Paris, perhaps the costli
of

Mme. Curie
is

from pitchblende est and rarest element and characterized


is

which uranium ore


is
a

and they find mostly the Hartz Moun by its ability emanate strong rays similar
to
in
it

No one hasn't yet succeeded sepa the Roentgen X-rays. The rays are
or
to
in

tains.
rating the element radium by itself, but so powerful that no animal could remain
of

its
of

room containing pound


of

they sell the form some one


in

alive
in
it

a
a

the chlorid; and


of

salts, generally this not radium.


only ex Pitchblende, uranium ore, found chiefly
to

be
in

few ounces believed


is

a
a

Radium has the same effect on the Hartz Mountains, its principal
in

istence.
is

bodily tissues like X-rays and they have source. Scientists believe that some radium
employ the earth's crust and the sun
in

made many attempts exists


in
to
to

cure
it

organic diseases, but they haven't had and stars. The element radium has never
a

great degree yet, but they been isolated, its commercial form being
of

in

success
a
it

49
salt, generally chlorid, of which only a few COPYING AND CORRECTING MANU
ounces are known to exist. SCRIPT.
Radium affects bodily tissues like X-rays Spelling, use of capitals, and all omissions
do, but its use to cure organic diseases has
and mistakes will be taken into considera
not been very successful and will likely be
limited to cauterizing and burning diseased tion in rating this subject.
parts which are inaccessible to ordinary Directions to the competitor: Write a
cauterizing methods, but which radium rays
corrected copy of the manuscript accom
have removed. An experimenter carelessly
carrying a tiny vial of radium salts in his panying. Correct all errors in syntax, spell
pocket discovered that radium rays, like
rays, if applied too long, sometimes cause
X ing, punctuation, and capitalization; write
in full abbreviated words, etc., as indicated.
severe and practically incurable ulcers. Ra Do not change or paraphrase the language
dium should be carefully handled and kept
in lead bottles, which the rays cannot pene of the copy, or insert, omit, or modify
trate. words, phrases, or punctuation marks, ex
(196 words.) cept as may be necessary to correct errors.

SPECIMEN MANUSCRIPT FOR COR RECTION.


CORRECTED MANUSCRIPT. The following questions and tests, which
have been used, indicate the general char
The following is a corrected copy of the acter of the sixth subject:
accompanying fac-simile manuscript:
The efficiency of the consular service is
of vital concern to our foreign trading in First Exercise.
terests, and it ought not to be overlooked Instructions.—On the blanks furnished
in passing that the diplomatic officers ren make the necessary double entry journal
der considerable assistance in the develop entries for the transactions found in the
ment of trade. It is exceedingly difficult, memoranda below. Keep bank account
however, to measure the value of consuls separate from the cash account. Make no
because much of their aid in winning for day-book entries.
eign markets is given indirectly. They are
March 1, 1905. Thomas Morris opened
known, for example, to be successful in pav
new books with the following resources:
ing the way for the success of itinerant
Cash in Citizens' Bank, $7,800; cash in safe,
agents of American firms and this is done $2,687.50; merchandise, $4,768; real estate,
in large part by the maintenance of a cor $9,750; an account against William Rose,
dial relationship between our people and $2,360; a note given by James Wilson for
those of foreign countries in which the con $1,287.50, due in six months without inter
suls are stationed. In many instances they est, accounted at its present worth, $1,250.
are able to assist the trained agents in plac His liabilities were an account due John
ing the orders. It was a conscious recog West, $2,976; a note for $900 due Arthur
nition of this assistance and a realization Moore, accrued interest to date, $27.50; an
of the necessity of having an effective corps acceptance at fifteen days for $840, dated
of intelligent workers in the service that February 15, 1905, drawn upon him by Abra
have prompted the business men's organiza ham Naylor in favor of Joseph Kemp.
tions to clamor for consular reform. Final
ly the victory for merit and efficiency were March 2. Bought of Jones Bros., mer
chandise, $5,400. Gave in payment a note
won when Congress passed the Remedial at six months, interest, $90, included in face
Act in 1906.
of note; face of note, $3,090. Balance on
account.
The Corrections. March 6. Sold to Samuel Aikens mer
The principal corrections made are: chandise, $2,600. Received in payment the
Efficiency, foreign, diplomatic were mis acceptance mentioned in liabilities, $840,
spelled in the original. and a draft on Chase Chemical Bank for
“However” requires a comma before and balance (not deposited).
after, as also does “for example,” which March 13. Morris sold to John Kelso a
was abbreviated in the original (e. g.). half interest in the business for $13,000 in
“Indirect” is an adjective modifying the cash. (Account the difference between this
verb “is given”; as an adjective can never valuation of the business and the valuation
correctly modify a verb, it is necessary to as shown by the opening entry as good
change it to an adverb, “indirectly.” will.)
“He,” personal pronoun, singular number, March 18. Sent Jones Bros. a draft at
was used in connection with its antecedent ten days' sight drawn by us upon William
“consuls,” which is plural; therefore, “they” Rose in favor of Jones Bros. Face of
should be used. draft, $600.
Successful, itinerant, maintenance were Wilson,
March 25. Bought of James mer
misspelled in the original. chandise, $2,400. Gave in payment his note
American was abbreviated in the original. mentioned in resources. Face of note, $1,
“Among” is incorrectly used instead of 287.50; present worth, $1,254.87. Balance
“between.” On account.
“Them,” a personal pronoun, is used as March 27. Sold Jarvis & Co., merchan
an adjective modifying the word people— dise, $3,600. Received in payment a sight
understood—(of foreign countries); the ad exchange for $800 on New York accepted
jective “those” is the correct word to use. at 34 per cent premium, and their note at
Necessity, effective, corps, business were twenty days for balance.
misspelled in the original. March 29. Paid by check on Citizens'
“Men's” is plural possessive, with the Bank the note due Arthur Moore mentioned
apostrophe BEFORE the final “s”. in liabilities. Face of note, $900; accrued
“And” and “Congress” abbreviated. interest to date. $31.70.
When the same error was made more March 30. Discounted at the Citizens'
than once in the exercise, one note to that Bank our note at six months, and received
effect was considered sufficient under the credit for proceeds. Face of note $800, dis
heading of “corrections made.” count $24. -
Remedial Act is begun with capitals be
cause it is the name of a particular act of Second Exercise.
Congress.
From the data given below make out a
balance sheet which shall include a trial
PRACTICE OF BOOKKEEPING. balance, the inventory, and statement of
The first exercise is given a weight of 2, losses and gains, proprietor's accounts, and
and the -second exercise a weight of 1. resources and liabilities.

51
On March 1, 1905, Charles Martin had on tate, $8,000; Traders' Bank, $4,780.25; bills
hand as per inventory merchandise, $5,400; receivable, $4,620; expense, $90.25; interest
real estate, $7,600; unused office stationery, and discount, $148.60; Joshua Miller, $2,
$20.75. The balance of the cash account, 897.50.
as shown by the ledger, was $3,640, and Credit footings: Charles Martin (propri
the balance of the bills payable account was etor), $6,797.85; merchandise, $7,800; real
$2,365. The footings of the other ledger estate, $120; Traders' Bank, $3,600; bills
accounts were as follows: receivable, $3,640.75; interest and discount,
Debit footings: Charles Martin (propri Joshua Miller, $
$197.80; (amount to be
etor), $970; merchandise, $12,800; real es supplied by the competitor).

BOOKKEEPER AND JUNIOR BOOKKEEPER, NEW


JERSIEY AND ILLINOIS SIERVICES.

Specimen Questions.

State Civil Service Commission of New COMMERCIAL ARITH METIC–1st


Jersey. Class.

BOOKKEEPER. Ques. 1:—A merchant bought a bill of


goods amounting to $1,568.95 at a discount
EXPERIENCE. of 10 and 5 per cent and sold them at the
list price. What per cent did he make?
This will cover a general examination on SOLUTION:
the past experience and education of the
applicant. Cost $1,568.95=List Price minus 10 and
5%.
100 90
SPELLING. 10 and 5%=14.5% –90 5
Accessible—Approachable.
Preliminary—Introductory. 10+ 4.50=14.5%
Resources—Means; material. 100%–14.5% =85.5%.
Surety—That which confirms or makes Cost $1,568.95=85.5% of List Price.
sure; security. Discount from List Price= Profit.
.1695
Solicitor—One who asks earnestly.
Monopoly—Sole right to deal in a
thing. 85.5)14.50
Liquidate—To clear away. 855
Intrinsic—Real; true.
Insufficient—Deficient. 5950 (Ans.) 16.96% (approx.)
Sagacious—Shrewd, discerning. 5130 Profit
10.
11. Insolvent—Unable to pay one's debts. 8:200
12. Financier—One skilled in finance.
13. Explicit–Plain; direct. 7695

14. Concession—Act of conceding. 5050


15. Biennial—Happening once in two
years. 4275

16. Colleague—A partner.


775
17. Lien—A legal claim.
18. Dissuade—To advise against.
19. Chargeable—Capable of being charged. Ques. 2:—What must be the face of a 6.
20. Recapitulate—To relate in brief, to day note to pay a bill of $2,539.85? Interest
summarize. 6 per cent.
Assets—Property belonging to or due
21. SOLUTION:
to a person, corporation or estate.
22. Liabilities—As the “entire debts” of a 6% for 60 days=1%
firm. 1% of $2,539.85=$25.40
23. Bankruptcy—State of being legally $2,539.85+$25.40=$2,565.25 face of note
bankrupt; complete loss. (Ans.).
24. In debtedness—The sum owed; debts N. B.-The note, of course, would make
collectively. no mention of interest, as it would already
25. Partial—As “partial payments.” be included on the face.)
-

52
Ques. 3:—What will be the proceeds of a Or, 10c on $1,000-15c on $1,500.
draft at 30 days after sight; interest 5 per $1,500—15–$1,499.85 (Ans.).
cent; brokerage % per cent?
SOLUTION: COMMERCIAL ARITH METIC–2nd
Grade.
100%=Amount of draft
5% interest for 1 year 1. Find the proceeds of a 60-day note
1/12X5=5/12% interest for 30 days for dated March 1, 1911, and dis
$2,500,
.416 2/3 counted April 1, 1911, at 6 per cent.
2. What is the net cost of the following:
5/12=12)50 List price $15 per dozen, discount 25, 10
48
and 5; list price $15 per dozen, discount 20
and 20; list price $15 per dozen, discount
20 4.16 2/3%=41/6 15, 15 and 10.
12
3. The wholesale price of coal is $4.50
per long ton. It is sold at retail at $6 per
80
short ton. What will be the gross profit
72
and the rate of profit on a car holding 50
tons?
4. Sold A. B. the following bill of goods:
8
100.00 Draft 26% yards gingham at 12% cents; 27%
.04 1/6 Interest yards ticking at 33 1/3 cents; 45 yards calico
at 8 1/3 cents; 3694 yards denim at 20
100.4 1/6 cents; 18% yards shirtings at 6 2/3 cents;
—0.5 Brokerage, Premium of 94% 25% yards denim at 25 cents. Find the
amount of the bill, and draw a S/D for the
(Ans.) 99.9 1/6 a1110unt.
Assuming draft to be for $100+ interest, 5. Bought goods at 20 per cent discount
the proceeds would be about $99.92. and sold them at 20 per cent advance over
the list price. What is the rate of profit?
Ques. 4:-Which is better for the buyer,
a straight discount of 25 per cent, or a dis

-
PENMANSHIP.

-
count of 12%, 10 and 2% per cent? What
will be the cost for a bill of goods amount
ing to $1,000? This paper will be marked for legibility,
neatness and general appearance, as well

-
SOLUTION:
$1000. as correctness and uniformity in the forma
$1000.
.75 .875 tion of words, letters and punctuation
marks.
5000 5000 As to “compulsory arbitration,” who
7000 7000 wants it? It is a contradiction in terms.
8000 The essence of arbitration is voluntary con
$750.00 (Ans.)
sent to take advice. If its obiter dicta are
Cost of bill of goods $875.000
amicably accepted it is excellent. If it can
amounting .90 compel it to but a new court, and we are
to $1,000
list price. where we started. Agreement and litiga
$787.50 tion are two opposite ways. If arbitration
.975 could be compulsory it would be superflu
OllS.
393.750
Oh, for the frank, hearty and open way,
551250 with real good will and no mental or tech
7087.50 nical reservations on either side, satisfying
the land of the intent of all concerned to
(Ans.) $767.81 meet all open questions “fair and square”!
Why not take the short cut and disappoint
The series of 12%, 10 and 2% per cent is the frantic, the mischievous, and the obtuse.
not as good as the straight discount of 25 All the strong-hearted, the whole land over,
per cent.
would rejoice to see the merely headstrong
Ques. 5:-What will be the cost of a sight set aside.
draft on Chicago for $1,500, exchange 10 “Does 'business' mean “Die you, live I’?
cents premium? Then ‘Trade is trade' but sings a lie;
SOLUTION: 'Tis only war grown miserly 1”
10 cents premium=.0001.=0.01% THEORY AND PRACTICE OF BOOK
100.00 1500
KEEPING—1st Class.
00.01 99.99
1. What is the effect of posting to the
99.99 13500 wrong side of the ledger if the double
13500 entry system is used?
13500 2. Give your reasons for preferring the
13500 double entry system of bookkeeping or vice
(Ans.) $1,499.85 Cost of sight draft Versa.

53
3. When a firm fails owing you $1,000, City of Chicago Civil Service Commission.
and you accept 60 per cent in settlement,
30 per cent in cash and 30 per cent in notes, JUNIOR BOOKKEEPER.
what are the proper entries? July
Date: 17, 1913.
4. State the process of changing a set
of books from single entry to double entry. SPECIAL SUBJECT.
5. What is an Account Current?
Ques. 1:—What does the debit side of a
6. Make a statement showing the condi double entry ledger show?
tion of a business having the following as ANS.:—The debit side of a double entry
sets and liabilities: ledger shows postings from the books of
original entry, reflecting increases in assets
A's investment ... . . . . . . . . . . . . . . . . $5,000
and losses in those accounts which show
B's investment ... . . . . . . . . . . . . . . . . 5,000
debit balances and decreases of liabilities
A's withdrawals . .. .. . .. . .. . .. . . .. 500
B's withdrawals .. ... . .. .... .. . .. . 400 and capital investment in those accounts
salary . . . . . . . . . . . . . . . . . . . . . . . .
which exhibit credit balances.
A's 1,000
Cash on hand. . . . . . . . . . . . . . . . . . . . . 500 Examples:
Merchandise on hand. . . . . . . . . . . . . . 4,000
Merchandise purchases . . . . . . . . . . . 10,000 Accounts showing debit balances:
Merchandise sales . . . . . . . . . . . . . . . . 7,000 Debits to the Cash, Accounts Receivable
Receivable accounts represent
Notes outstanding ................ 750 and Notes
increase of assets.
Due from sundry persons. . . . . . . . . . 1,500
Expenses to date, exclusive of A's Debits to the Interest, Commission and
salary . . . . . . . . . . . . . . . . . . . . . . . . Expense accounts represent increase of
2,500
losses.
Rent due and unpaid, included in
eXPen SeS . . . . . . . . . . . . . . . . . . . . . 200
Accounts showing credit balances:
Discount on notes received. . . . . . . . 15
Due sundry persons............... 500 Debits to the Accounts Payable and
Notes Payable accounts represent decrease
7. What is a contingent liability? Give of liabilities.
examples. How should such liabilities ap Debits to the Proprietor's Investment Ac
pear in a financial statement? count, Capital Stock, etc., represent de
8. Give illustrations and state the effect crease of capital.
of:
Ques. 2:—What is the effect of posting
(a) A qualified endorsement.
to the wrong side of the ledger, if the double
(b) A restrictive endorsement. entry system is used?
(c) A full endorsement. ANS.:—Posting to the wrong side of a
(d) A blank endorsement. double entry ledger has the effect of throw
9. Define the term “Without Recourse” ing the trial balance out of balance, as the
and state when it may be used. aggregate of debits and credits in the vari
10. Define a trial balance and state how ous accounts will not agree. We will use
it differs from a balance sheet. Illustrate. the following entry for illustration:

John Smith............. $100


To Cash $100
THEORY AND PRACTICE OF BOOK If John Smith's account were credited
KEEPING—2nd Class. instead of debited, and no other error were
made, the trial balance total would show
1. What is bookkeeping? the credit side $200 more than the debit
2. What is an account?
advantages of side. This would be caused by two credit
3. What are the relative entries of $100 each having been made with
double entry over single entry?
journal entry for out a corresponding debit. If in the above
4. Make the proper example two errors were made by trans
opening a ledger to represent the following
posing the postings, crediting Smith's ac
assets and liabilities:
count and debiting Cash account, the trial
Notes on hand. . . . . . . . . . . . . . . . . . . . $1,000 balance total would be correct, as one error
Due sundry persons. . . . . . . . . . . . . . . 2,000 would compensate for the other. However,
Due for rent. . . . . . . . . . . . . . . . . . . . . . 100 the two accounts would be incorrect.
Goods on hand. . . . . . . . . . . . . . . . . . . . 5,000
Notes outstanding . . . . . . . . . . . . . . . . 1,500 Ques. 3:—Define the term “Without Re
Due from sundry persons. . . . . . . . . . 2,000 course” and state when it may be used.
Accrued interest on outstanding ANS.:—The term “Without Recourse"
notes . . . . . . . . . . . . . . . . . . . . . . . . . 150 covers the endorsement of a negotiable in
Land . . . . . . . . . . . . . . . . . . . . . . . . . . . . . 5,000 strument, whereby the endorser expressly
Building . . . . . . . . . . . . . . . . . . . . . . . . . . 3,000 stipulates that he will not be liable if the
Equipment (machinery and tools). 1,500 maker of the instrument does not pay. How
Mortgage on building. . . . . . . . . . . . . 1,500 ever, such an endorser would be liable in
the event that the signature of the maker
5. What does the debit side of a double or any prior endorser were not genuine, as
entry ledger show? his own endorsement is a guaranty of the

54
validity of the signature of the maker as SOLUTION:
well as all prior holders. 400
.80
Ques. 4:—Define Balance, and
a Trial
state how it differsfrom a Balance Sheet.
320.00
ANS.:—The Trial Balance is a statement
.90
which shows the total debits and credits of
Or:
each ledger account and the aggregate of
debits and credits, which should be equal.
288.00 4/5×9/(10)×19/(20)×($400)
A all

.95 ($20) $2=$273.60


Trial Balance differs from Balance

a
(Figures parentheses

in
that prepared before closing
in

is

Sheet
it

1440 are cancelled)


the accounts, while the Balance Sheet

is
2592
profits and
of
prepared after determination
closing of accounts. Trial Balance shows
A

273.60
debit and credit post
of

merely the total


268.75
the ledger, real
to

ings all the accounts


in

and nominal, while the Balance Sheet shows


(Ans.) cheaper by this amount

It
only the real accounts

is
$4.85
of

the net balance


buy for the net figure

to

of
after the net results of the nominal accounts
by the proprietorship $268.75.
have been absorbed
account. The Balance Sheet shows condi
Ques. 2:—A man bought

96

of
the business, while the Trial Balance acres land
of

tion
for $5,625. He raised crop wheat aver

of
simply establishes the mathematical accu

a
aging 24% bushels per acre and sold for

it
racy the ledger accounts.
of

87% cents per bushel. He paid for labor


Ques. 5:—What bookkeeping? and seed $14.85, and other expenses $92.25,
is

ANS.:—Bookkeeping systematic rec and then sold the land for $60 per acre.
is
a

Its purpose What was his gain per cent?


of

ord business transactions.


show the progress and condition
to

of
is

the business. SOLUTION:


24.75 bu. per acre
an

$14.85
Ques. 6.—What Account?
is

96 No. of acres 92.25


ANS.:—An Account record of
is
a

transaction, under an appropriate head, af


148 50 $107.10
fecting person, property thing.
or

It
is
a

2227
5
so

usually arranged show two sides,


to
as

debit and credit. These two sides are op 2376.00 Total bu.
posite tendency, one indicating increases
in

7/(8) (2376) 272=


×

and the other decreases; other words, $2079.00


in

positive and negative. The difference be Expense 107.10


these two sides the balance or
is

tween
result of the account. Net Income $1971.90
96 Acres
Ques. 7:—What Contingent Liability? $60.
is
a

Give example.
ANS.:—A Contingent Liability represents $5760. Selling price
credit item the status of which has not 1971.90
a

been definitely determined, pending the


passage the happening $7731.90 Total realized
or
of

of

time an
event. An example Cost
of

such an item would 5625.00


be

Notes Receivable Discounted account.


by

matur Profit (.374+


at

the note met the maker $5625) $2106.90


If

is

ity, then there no liability; on the other 16875


is

hand, not paid by the maker,


if

a is
it

it

proves liability. contingency 41940


be
to

It
is
a

during its term. 39.375

25650
REPORT AND PENMANSHIP. 22500

Write report about 200 words, giving 3150 (Ans.) 37.4+% gained.
of
a

your reasons for preferring the double en


try system bookkeeping over single en Ques. 3:—Two houses were sold for $3,600
of

try bookkeeping, each, one per cent and the


20
or

at

of

vice versa. loss


a

gain per cent. How much


of
20
at

other
a

was gained lost by the transaction?


or

RITH METIC.
A

SOLUTION:
Ques. 1:—Which better, and how much, $3,600-120%
is

or

of
S.

6/5 cost (house


at P. at P.

buy goods for $268.75 net, with dis gain)


or
to

sold
per cent, per cent and
of
20

10

counts $3,600-80%
or

of
S.

4/5 cost (house


5

per cent from list price


of

$400? sold loss)


a

55
$3,600+6/5=(3600) City of Chicago Civil Service Commission.
600X5/(6)= $3,000 Cost BOOKKEEPER.
$3,600-4/5=(3600)
900X5/4= 4,500 Cost SPECIAL SUBJECT.
$7,500 Total cost 1. The following balances appear in the
general ledger of a corporation after the
2X33,600– 7,200 Total realized
books have been closed: Property and
plant, $10,000; manufactured goods, $5,000;
$300 Lost notes receivable, $3,000; bills payable, $2,000;
.04
customers' accounts, $7,500; purchase ledger,
$3,000. The profit and loss account, which
7500)300.00
300.00
is kept in a private ledger, indicates a loss
of $3,000. Cash, for which no account is
(Ans.) 4% Loss. kept in the general ledger, shows an over
Ques. 4:—Find the interest
on $350 for draft of $3,500. The capital stock account
3 years, months, 18 days, at 4 per cent
4 is also kept in a private ledger. Prepare
(30 days to the month). balance sheet and supply the capital stock.
2. The debit side of the trial balance ex
SOLUTION: ceeds the credit side by $37.50. A clerk has
$350 verified the footings of the trial balance of
.04 the ledger, and of the posting mediums, and
has carefully checked the balances from the
14.00 Int. for 1 year ledger to the trial balance. State in what
3 order he should proceed to locate the dif
ference before checking the postings.
$42. Int. for 3 years 3. A buys a typewriter for $100 and
1/(3) × (14.00) 4.67=$4.67 Int. for 4 months charges the expenditure to furniture and
1/4× (4.67) 1.18=$1.18 Int. for 1 month fixture account. Subsequently he purchases
(18)/(30)=3/(5) x(1.18) .236=.71 Int. for 18 a different style of typewriter for $120, and
days at the same time disposes of the first one
Summary of Interest: for $60. The necessary entries are made
$42.00 =3 years in his cash book. What are they?
4.67=4 months 4. In closing the books of a firm, what
.71=18 days accounts should be considered necessary to
accrue in order to show the true financial
(Ans.) $47.38 Total Interest. condition at the closing date, and the actual
result of operations for the preceding pe.

$. 5:—A piece of property pays a tax


of $129.01, which is at the rate of $13.30 per
thousand dollars. What is the value of the
riod?
5. Define: (a) trial balance; (b) balance
sheet; (c) statement of income and profit
property? and loss; show wherein they differ.
6. Would the debits and credits be equal
SOLUTION: in a trial balance of a ledger containing
$13.30 per $1000–1.33% only customers' accounts?
97=1% 7. From what source would entries gen
erally originate that are contained in an
1.33)129.01 accounts receivable ledger?
1197 8. How should a clerk prove the accu
racy of his work for the month if he were
931 keeping but one ledger?
931 9. On January 7, 1906, W, a fruit grower
and dealer, consigns to Y, a New York
100×97=$9,700, value of property (Ans.). commission merchant, 100 crates of berries,
40 boxes to the crate, at the same time
CIVIL GOVERNMENT. drawing on consignees at sight for $100.
The freight charges amount to $17.28. One
1. Define the following words: Citizen; fourth of the goods is spoiled so as to be
Franchise; Tariff; Alien. unsalable through the negligence of the
2. Who presides over the House of Rep railroad company, which pays $50 damages
resentatives, and how is he chosen? to Y in settlement of claim for reimburse
3. How many members compose the ment, and of the remaining fruit, one-third
President's cabinet, and by what titles are is sold at 5 cents a box and the balance at
they known? 10 cents a box, cash, on July 10, the auc
4. What is the term of the following tion and commission charges being 5 per
officers of the City of Chicago: (a) Mayor; cent. W’s draft is paid by Y on July 8.
(b) City Clerk; (c) City Treasurer; (d) Prepare a statement to be sent on July 20
Civil Service Commissioner? from Y to W with check in settlement of
5. Where do presidential electors cast account to that date, interest being charged
their ballots? and allowed at the legal rate.
6. What is the Boulevard Link Ordi 10. The firm of A Z & Co. is about to
nance just passed by the City Council? be turned into a stock company with 2,000

56
shares of $100 (par value) each, one-half 6 lathes, etc., $2,500. At
what sum must the
per cent preferred, one-half common. It good-will be estimated in order that the
owes for merchandise $50,000 and has doubt total assets may appear equal to the total
ful accounts receivable of $40,356, estimated liabilities on the books of the new com
to be actually worth $30,000 net. In addi pany?
tion, expert appraisers' schedules show the REPORT.
following: Merchandise completely manu
factured, $100; bills receivable, $11,500; ac Write a report to the head of the firm
counts receivable guaranteed (in addition for whom you are working in answer to a
to those mentioned above), $10,000; furni request as to what the books of the com
ture and fixtures, $2,400; merchandise partly pany show concerning the advisability of
finished, $40,000; cash, $1,000; industrial catering to the trade of Jones & Co., a
stock (par value $7,000), estimated at $5,500;
trade-marks, copyrights, etc., $19,000; customer. Assume such facts as may be
real
estate (mortgaged for $10,000), $2,000; tools, consistent.

BOARD OF EDUCATION EXAMINATION QUESTIONS


IFOR, ILICIENSE TO TEACH ACCOUNTING AND
BUSINESS PIRACTICE IN HIGH SCHOOLS
AND EVENING SCHOOLS.

Department of Education—The City of Chas. Smith from the Sales Book with $132;
New York. omitted to record a bill for plumbing re
pairs received from A. C. Brown, $35;
EXAMINATION FOR LICENSE TO posted interest $13.80 allowed to a customer
TEACH ACCOUNTING AND BUSI to the wrong side of Interest account; ne
glected to bring down the inventory of un
NESS PRACTICE IN HIGH expired insurance, $212.50, when the Ex
SCHOOLS. pense account was closed; a balance of $250
March, of Notes Receivable was carried forward
Date: 1916.
to the wrong side of the account; posted
BOOKKEEPING. a purchase of $84.60 from A. B. King to
the wrong side of King's account; a short
(Time–Three hours.) age of $3.80 in cash was properly recorded
in the Cash Book but was posted to the
1. Write out or suggest five questions wrong side of Over and Short account; a
that you would consider appropriate for the payment of $85 to James Lark was placed
examination of a high school class in book on the wrong side of Cash Book and cred
keeping that has had the subject for one
ited to his account; a rebate of $216 on a
term of twenty weeks, five forty-five min purchase from L. B. Hall was correctly
ute recitation periods per week. (15) journalized, but was posted as $2.16 to the
2. Howard Cheston and Willard Mills wrong side of Discount account.
are equal partners doing business under the
how each error should be corrected.
name of Cheston & Co. They have agreed
( 25show
to dissolve the partnership on the follow
ing terms: Mr. Cheston is to take over all 4. H. M. Wilson and S. W. Moore are
the assets and to assume all the liabilities partners doing business under the firm name
of the firm and to give Mr. Mills a note of Wilson & Moore, their respective inter
due in one year for Mr. Mills' interest in ests being $8,000 and $6,000. They now wish
the business. A statement of the firm's to incorporate with a capital stock of $20,
affairs shows the following: Bank balance, 000, shares of $100 each. Each partner is
$865.40; notes receivable, $2,000; accounts to turn over his present interest in the
receivable, $3,165.80; office furniture, $325; business in exchange for an equivalent
delivery truck, $865; merchandise inventory, amount of paid-up stock (at par). John
$14,650; good-will, $500; notes payable, $1, Strong has subscribed and paid cash for
875; interest accrued on notes payable, $2,000 of the company's stock. The remain
$87.43; notes receivable discounted, $850; ing stock will remain unsubscribed for the
accounts payable, $2,176.75. present.
Make necessary entries to close the books Make the entries necessary to record the
of Cheston & Co. and to open the books of change from a firm to a corporation, the
Howard Cheston, the same books being same books being used. (25)
used. (25) 5. State briefly, (a) The respective ad
3. A set of books were found to contain yantages of single and of double entry book
the following errors: Omitted to charge keeping; (b) Under what circumstances
57
would single entry be the more desirable? States showing the Mississippi River and
Double entry? (10) the Great Lakes. Indicate the following on
this map:
1. Spring and winter wheat areas and
Department of Education—The City of approximate combined yearly production.
New York. 2. Cotton area and approximate yearly
production.
EXAMINATION FOR LICENSE TO Copper mines and per cent of world's
TEACH ACCOUNTING AND BUSI 3.
yearly production.
NESS PRACTICE. 4. Meat packing centers.
5. Iron ore areas.
Date: March, 1916.
6. Centers in which agricultural machin
ery is manufactured.
COMMERCIAL LAW AND PROCE
7. Tobacco growing areas.
DURE, COMMERCIAL GEOGRAPHY
8. The two leading dairying states. (8)
AND HISTORY OF COMMERCE.
ARITH METIC.
(Answer four questions in group 1-5 and (Time—Two hours.)
four questions in group 6-10.) B.—Let all computations appear in
(Time—Three hours.) f ull.
N.
GROUP 1-5. 1. The sales of a dry goods firm amount
to $90,000 per year; 2/5 of the sales were
1. Adams is an indorser on Cummings'
made at a profit of 25 per cent; 3/10 at a
promissory note, which is overdue, and no profit of 35 per cent; and the remainder at
tice of protest has been served on both a profit of 20 per cent; what was the cost
Adams and Cummings. Adams requests the of the goods? (5)
holder to proceed at once against the maker 2. A man wishing to sell his real estate
as he fears that in a short time Cummings asked 36 per cent more than it cost him,
will be unable to pay. The holder neglects but he finally sold it for 16 per cent less
to do so, and Cummings fails. The holder than his asking price. He gained by the
sues Adams on the note. Judgment for
transaction $740.48. How much did the
whom and why? (7) real estate cost him, what was his asking
2. Compare a partnership and a corpora price, and for how much did he sell it? (5)
tion as to (a) rights and powers of mem 3. The cabin of a steamer is 165 feet long
bers, (b) liabilities of members. (7) and 18 feet v ide. What will it cost to car
3. State the remedies of the seller when pet the same with Brussels carpeting 3% of
the buyer refuses to accept goods accord a yard wide at 80 cents per lineal yard?
ing to contract. (7) (The strips are to be laid lengthwise of the
4. (a) What are the essentials of a good cabin.) (4)
negotiable paper?
4. A gentleman's garden 200 feet long
(b) Define a holder in due course. (7) and 180 feet wide, is enclosed by a tight
5. Define insurable interest in (a) fire board fence 5% feet high. What will it
insurance, (b) life insurance; (c) explain cost to paint the fence at 10 cents per square
and illustrate the effect of a breach of the yard? (4)
following on the validity of a fire policy:
Promissory condition, 5. How many bricks, each 8 inches long
1. and 4 inches wide, will it take to surround
2. Immaterial representation, the above garden inside the fence with a
3. Material representation, walk 6 feet in width 2 What will be the
4. Warranty. (7)
cost of the bricks at $7 per thousand? (4)
GROUP 6-10.
6. If one bushel, of 60 lbs. of wheat,
makes 48 lbs. of flour, how many barrels
6. Mention with approximate dates, pe of flour can be made from the contents of
riods in the history of the United States a bin 10 feet long, 5 feet wide, and 4 feet
when protection of home industries by a deep, filled with wheat, allowing 196 lbs. to
high tariff has been a political policy of the a barrel of flour? (Answer in barrels and
government. (8) pounds.) (6)
7. Briefly describe the industrial revolu 7. $750 Yonkers, N. Y., Nov. 15, 1915.
tion in England, covering prime causes and Three months after date I promise to pay
results. (8) to the order of Richard Fairbanks. . . . . . . . .
8. State general causes and give specific Seven Hundred Fifty 00/100. . . . . . . . Dollars
illustrations of (a) centralization of manu Value received, with interest at 5 per cent.
facturing industries, (b) decentralization of JOSEPH FARLEY.
manufacturing industries. (8)
9. Write a brief survey of Japan covering Find the proceeds of the above note if
approximately: (a) area, (b) population, discounted on December 12, 1915, at 6 per
(c) extent of natural resources including cent. (6)

*
agricultural area and crops, (d) industries, 8. Which is the better investment: N. J.
(e) exports, including destination, (f) im C. R. R. 5 per cent bonds due in 1920 and
ports, including source, (g) present eco purchasable at 105, or Mo. P. 6 per cent
nomic problems. (8) due in 1925 and purchasable at 110?
10. Draw an outline map of the United 6

58
Department of Education—The City of Inventory, goods on hand Dec. 31,
New York. 1911 . . . . . . . . . . . . . . . . . . . . . . . . . . $6,750
Insurance, unexpired (6 mos.). . . . . . ... .. .
EXAMINATION FOR LICENSE TO Furn. & Fixtures, estimated value.. 2,200
TEACH IN EVENING SCHOOLS. Advertising, unconsumed . . . . . . . . . 500
Date: October, 1912. Office Salaries, unpaid. . . . . . . . . . . . . 800
Commissions, due salesmen........ 300
BOOKKEEPING. Interest, accrued on Notes Receiv. 200
(Time, 1% hours.) Interest, due on Notes Payable.... 180
1.From the following trial balance, pre Allow 2 per cent reserve for bad debts
pare a profit and loss statement and bal
- on Accounts and Notes Receivable.
ance sheet:
Profit or Loss apportioned as follows:
Trial Balance of Wise & Altman,
Wise 5/8, Altman 3/8. (45)
December 31, 1911.
J. F. Wise, Capital Acct... $41,130 2. J. J. Smith and M. S. Reed, who have
R. H. Altman, Capital Acct. 37,500 been conducting a partnership business un
Cash . . . . . . . . . . . . . . . . . . . . . $37,900 der the firm name of Smith & Reed, and
Notes Receivable . . . . . . . . . 21,400 sharing losses and gains equally, desire to
Accts. Receivable . . . . . . . . . 40,500 change their books from single to double
Mdse. Inventory, Jan. 1/11. 14,900 entry. Make the proper journal entry to
Furniture & Fixtures. . . . . . 2,600
effect this change.
Notes Payable . . . . . . . . . . . 27,800
Accts. Payable . . . . . . . . . . . 25,000 The accounts on the single entry ledger
Purchases . . . . . . . . . . . . . . . . 73,6 are as follows: Accounts Receivable, $20,
Sales . . . . . . . . . . . . . . . . . . . . . 100,410 000; Accounts Payable, $15,000; Smith's in
Advertising . . ... . .. . .. . .. 2,000
vestment, $18,000; Reed's investment, $12,
General Expense . . . . . . . . . 2,450
Commissions .. . .. .. .. . .. . 18,750 000. Smith's withdrawals, $2,000; Reed's
Office Salaries . . . . . . . . . . . . 10,000 withdrawals, $3,000.
Insurance (1 year). . . . . . . . 200 The assets and liabilities of the firm not
Discount on Sales. . . . . . . . . 800
on the books are: Cash, $9,000; Notes Re
Interest . . . . . . . . . . . . . . . . . . 700
Taxes . . . . . . . . . . . . . . . . . . . . 500 ceivable, $7,000; Merchandise inventory,
Printing and Postage. . . . . 350 $4,000; Railroad stock, $3,000; Real Estate,
Discount on Purchases.... 2,610 $7,000; Unexpired Insurance, $100; Notes
Profit and Loss. . . . . . . . . . . 1,000
Payable, $4,000; Mortgage Payable, $3,000.
J. F. Wise drawing acct... 3,800
R. H. Altman drawing acct. 3,000 3. Raise ledger accounts for Smith &
Reed showing each man's investment after
$234,450 $234,450
books are changed to double entry.

59
PRACTICE QUESTIONS AND ANSWERS ON BOOK
KEEPING, COMMERCIAL PRACTICE AND
TIERIMIS.

BOOKKEEPING PRACTICE. posted daily? There is $1,000 in the bank.


Ques. 1:—What is an Invoice Book? ANSWER:
ANS.:—A book in which record is made
Cash Book.
of invoices for purchases. Sometimes the
original invoices are posted in a scrap-book, Dr.
and in this form it often is used as an On hand–In bank. . . . . . . . . $1,000.00
auxiliary to the regular Purchase Book. In drawer. . . . . . . 30.00

Ques. 2:—What is a Shipping Book? $1,030.00


ANS.:—An auxiliary memorandum book Due to proprietor (cash
in which is kept a record of all shipments slip) 40.00 $990.00
made.

Ques. 3:—What is a Purchase Book? Ques. 10:—What is a Ledger?


ANS.:—A record containing the names of
ANS.:—A book containing all the ac
creditors and amounts of goods purchased
counts of a business as assembled by the
from them. It is the basis for posting to postings from the various books of orig
the Accounts Payable Ledger. Journal, Cash Book, Pur

e.,
inal entry, i.
Ques. 4:—What is a Sales Book? chase and Sales Books.
ANS.:—A record containing the names of
debtors and amounts of goods sold to them. Ques. 11:—What Bound Ledger?

is
a
It is the basis for posting to the Accounts ANS.:—One the leaves of which are

so
Receivable Ledger. stitched together that no leaf can be
removed without being torn out.
Ques. 5:—What is a Cash Book?
ANS.:—A book in which original detailed Ques. 12:—What Loose-leaf Ledger?

is
a
entry is made of all cash receipts and dis ANS.:—A book consisting

of
loose leaves
placed into binder, and there

or
bursements. cover
a

secured. tabbed index running from


A

Ques. 6:—When is a Petty Cash Book cover usually provided with


to

cover

is

a
used? Loose-leaf Ledger. In some the Binders
ANS.:—When a business has many small are locked with key; others they are
in
a

cash transactions with which it is not de together with thumb-screws clamps.

or
held
sirable to burden the regular Cash Book.
Ques. 13:—What Card Ledger?
is
a

Ques. 7:—To what book are postings from ANS.:—One which consists cards,
of

a
the Petty Cash Book made? separate one for each account. These cards
ANS.:—To the regular Cash Book, peri stand upright tray or drawer. Some
in
a

odically; usually monthly. times there hole the bottom of each


to in
is
a

be passed through
to

Ques. 8:—What is a Journal? card enable rod


a

ANS.:–A book of original entry in which so that card can be taken out only by
a

removing the rod, which usually done


are analyzed into their com
is

transactions
ponent parts, debits and credits, prepara: by unscrewing the rod. The rod also can
tory to posting to the Ledger. be locked. Other times the cards stand
Details and
loose the tray. works on the same
in

It

explanations of transactions which in for principle Card Catalog.


as

mer times were found in a Day Book, are


a

in modern practice entered in the Journal.


Ques. 14:—What are some the advan
of

Where a full set of books is used, i. e.,


tages and disadvantages
of

Cash, Purchase, Sales, etc., the Journal is the Bound


Ledger?
used only for such transactions as are not ANS.:—Principal advantage is: (1) The
contained in the other books of original
entry. leaves cannot be removed without detection,
almost impossible
to

and substitute
is
it

9:-When you up cash leaves. faithful record, not easily


It
is

Ques. make at
a

night you find $30 in currency and coin on tampered with.


hand in the drawer and a cash slip for $40 Disadvantages are: (1) They are waste
due to the proprietor for money advanced space. Many the pages are never
of

of

ful
to enable you to make a disbursement for frequently necessary
to to

filled. (2)
It
is

which there were insufficient funds in the continue an account from one filled page
Cash in the drawer, therefore, is different part
of

drawer. another the book.


in
a

$10 minus. How would you show this state (3) Each account must be indexed. (4) As
of affairs in closing the Cash Book for the usually the number pages an account may
of

day, assuming that the Petty Cash Book is require cannot estimated, the large ac
be

60
counts are liable to become scattered, and ANS.:—The account would have to be
accounts which it is desirable to keep to made over by going over the books of orig
gether cannot be kept together. (5) They inal entry. The monthly trial balances
are bulky. (6) When the Ledgers are thick, would be of assistance for checking pur
arm rests must be used when posting at the poses.
bottom of pages. (7) The older the Ledger
grows, the more dead accounts it contains, Ques. 19:—What books are accepted as
and when taking off a monthly trial bal legal evidence, and what ones not?
ance it is necessary to scrutinize many dead ANS.:—Only books showing original de
pages. There is always danger that on an tailed entries are legal evidence. The Ledger
apparently dead page there may be a live is not a book of original entry, and there
account which is so crowded that it looks fore has only corroborative value.
balanced. (8) When the Ledger is filled
it is necessary to transfer all the live ac Ques. 20:—What is the most essential
counts to a new Ledger, which is slow
work, and there is danger of errors being difference between the Double Entry and
Single Entry systems?
made in such transfer. (9) If not bound
so as to open flat, the pages curve and make ANS.:—In the Double Entry system
neat ruling difficult. The curving of pages there always are made two entries, one a
also is liable to lead to posting on the debit and one a credit, while in the Single
wrong line. Entry system there is usually only one entry
for each transaction.
Ques. 15:—What are some of the advan
tages and disadvantages of the Loose-leaf Ques. 21:-Give some of the comparative
Ledger? advantages and disadvantages of Single En
compared try and Double Entry Bookkeeping.
ANS.:—As with the Bound
Ledger, it can be said that it has nearly ANS.:—In a small business Single Entry
advantages frequently is necessary. Advantages of
all of the which have been men
disadvantages Single Entry: (1) Very simple and requires
tioned as for the Bound
Ledger. - the minimum of work. (2) Fewer books
Disadvantages: (1) There is danger of are used and fewer entries made. (3) The
substituting leaves, or of a leaf being co bookkeeping can be made so simple as to
vertly removed. require as little time as the proprietor may
(2) It is somewhat of a
desire to have given to

it.
nuisance to remove old leaves and substi
tute new ones. (3) They are more expen Disadvantages Single Entry: (1) Mis
of

sive, at least at first, than Bound Ledgers. takes charging and posting may be made
in

(4) They do not open as flat as a flat-open

to
and never discovered. (2) easy

It
is
ing Bound Ledger. falsify such books. (3) The books do not
show the complete status
of
the business.
Ques. 16:—What are some of the advan Advantages Double Entry: (1) Any
of

tages and disadvantages of the Card Ledger? error, except posting the wrong account,
to

ANS.:—It may roughly be said that they revealed by the Trial Balance. (2)

It
is
is

have nearly all of the advantages which difficult for dishonestly inclined bookkeep
a

are disadvantages of the Bound and Loose manipulate the accounts without dan
er
to

leaf Ledgers. The cards are alphabetically ger shows not only
of

detection. (3)
It

arranged. (1) An account can be located actual assets and liabilities, but also nom
instantly. (2) When a card fills up it is inal accounts, such merchandise, expense,
as

only necessary to add another. (3) When etc. (4) enables more intelligent man
of It

a card is balanced it can be taken out of the agement Double


of

business. (5) set


A
a

live accounts and placed in a separate place Entry books balance commands the re
in

for balanced accounts; then it can be re spect fire insurance adjusters, and facili
of

called whenever again desired. (4) It is tates settlements, etc. (6) the only
It
is

very convenient and time-saving. complete system.


Disadvantages: (1) Danger of a card be Disadvantages Double Entry: (1)
It
of

ing abstracted or lost through carelessness.


requires much more time than Single Entry,
(2) Danger of substitution. (3) Most of always
to

at

and there be dreaded that


is

the cards obtainable on the market are too


may necessary
be
of

the end the month


it

small and the lines too close together. to check all the transactions of the month
more expen
to

find small error. (2)


It
is

Ques. 17:-—What precautions


a

can be taken sive, requires more competent book


as
it

to enable discovery of loss of a card from


a

keeper and larger number and more ex


a Card Ledger?
a

ANS.:—Keep a register on the lines of pensive books; also more space and larger
which are consecutive numbers, and enter safes are required for the books.
each card in the register as made, giving it
Ques. 22:—What Bank Pass Book?
is

the register number. Occasionally check


a

the cards with the register to see if any ANS.:—A memo book which you take
the bank every time you make deposit,
to

are missing.
a

for the purpose having the teller enter


of

Ques. 18:—If a page be lost from a Loose the deposit.


of

therein the date and amount


leaf Ledger or a card lost from a Card He affixes his initial each such entry.
to

It

Ledger, how can it be replaced? really receipt book.


is
a

61
Ques. 23:—What does the bank do with ANS.:—A Calendar year is one that be
your checks after paying them, and what gins with January 1st and ends with De
final disposition does the bank make of such cember 31st. A Fiscal year is one that
checks? runs for one year from any date fixed by
ANS.:—Upon paying a check, the bank the business itself. If a firm starts in busi
stamps it “Paid,” with the date, and can ness on March 1st, for example, they may
cels it by perforation. The check is then prefer to have their year always end with
charged to your account. At the end of the last day in February and thus avoid
the month all of the cancelled checks are an incomplete year, which creates difficul
returned to you, together with a statement ties when comparing one year with another.
of your account. In other cases the seasonal activity of a
business will determine when the Fiscal year
Ques. 24:—What are cancelled checks should end in order to have a minimum
usually called?
of work in counting inventory and deter
ANS.:—Vouchers. mining profits.
Ques. 25:—What is meant by a Balance?
ANS.:—The difference between the debit
and credit sides of an account. COMMERCIAL PRACTICE.
Ques. 26:—What is Working Capital? Ques. 39:—What is a Partnership?
ANS.:—The capital actually employed in ANS.:—A union of two or more persons
current operations of the business. for the transaction of business or owner
ship of property.
Ques. 27:—Where would you indicate in
the Ledger the stock subscriptions of all Ques. 40:—What are the relative legal
of the stockholders of a corporation? rights and authority of Partners?
ANS.:—In the Capital Stock account. ANS.:—These are dependent upon the
agreement made at the time of forming the
Ques. 28:—What is Appreciation? co-partnership. If two persons agree to
ANS.:—A bookkeeping term for increase be equal partners without qualification, they
in the value of property. have equal rights and powers. If they are
not to be equal partners the agreement
Ques. 29:-What is Depreciation?
ANS.:—Reduction in the value of prop should specify the extent of the interest
erty. and authority of each.

Ques. 303–What are Assets? Ques. 41:—What is a Corporation?


ANS.:—Goods, property, rights or pos ANS.:—An organization formed under
sessions of value. the laws of a State and which has registered
with the State authorities the objects and
Ques. 31:-What are Fixed Assets? scope of the business, its location, the
ANS.:—Assets acquired to enable the amount of capital, the names of the incor
conduct of the business, such as real estate, porators and a copy of its constitution and
buildings, machinery, etc. by-laws. A Corporation is conducted
through a Board of Directors elected by
Ques. 32:—What are Floating Assets?
all of the stockholders. The Directors man
ANS.:—Those that change hands, such age the business according to the constitu
as merchandise, cash and accounts receiv
able. tion and by-laws, and they elect the officers,
who usually are a President, Vice-President,
33:—What are Passive Assets?
Ques. Secretary and Treasurer. The officers have
ANS.:—Those that have no active value, such powers as the constitution and by
such as Patents and Good-will. laws prescribe and as the Directors may
direct.
Ques. 34:—What are Fixed Charges?
ANS.:—Such items as Rent and Salaries, Ques. 42:—What is meant by “Robert
which occur at regular intervals and can be Woodhouse, Ltd.”?
estimated in advance. ANS.:—It means that regardless of what
ever assets he may have, it is to be under
Ques. 35:-Define Net Profit. stood by all who deal with him under that
ANS.:—Gross income minus all expenses. title that his liability is limited to the cap
ital actually invested in that particular busi
Ques. 36:—What is the difference between ness and that if the business fails his other
Investment and Expense? assets cannot be touched.
ANS.:—Investment is putting money into
something that has a value that can be in Ques. 43:-What are some of the advan
ventoried. Expense is putting money into tages and disadvantages of Incorporating a
something that cannot be inventoried. business?
ANS.:—Advantages: (1) It perpetuates
Ques. 37:—What is a Nominal Account? the business, through provision for its au
ANS.:—One that exists in name only, a tomatic continuance, unaffected by deaths
thing having no tangible, actual existence.
or quarrels among the owners. The busi
Ques. 38:—What is the difference between ness can be dissolved only by voluntary
a Fiscal and a Calendar year? agreement of the majority of the stockhold

62
ers, or by order of a court. (2) The liabil Ques. 48:—What is a Monopoly?
ity of the stockholders is limited to the ANS.:—A combination of capitalists
amount they have invested in the business, formed for the illegal purpose of controlling
and any outside assets they may have can a commodity by crushing out competition.
not be touched. (3) The interests of the
stockholders to a certain extent are pro
tected by the State, because the officers COMMERCIAL PAPER.
must act in conformity with certain provi
sions of law designed to protect even the Ques. 49:-What is the difference between
smallest shareholder. a Demand Note and a Promissory Note?
Disadvantages: (1) Owing to the State ANS.:—A Demand Note is a promissory
regulation and the necessity of making all note or acceptance payable on presentation
acts conform with the constitution and by or on demand. A Promissory Note is a
laws, and the right of voice of even a very promise, signed by the maker, to pay a
small shareholder, the freedom of action of stated sum at a specified time and place.
the management is to some extent limited.
Ques. 50:—What is an Endorsement on a
There is more complexity in the transaction note?
of business. (2) It costs money to incor
porate, and there are corporation ANS.:—It is the signature on the back

.
taxes to
pay, although not heavy, but in a small of the note of a person who undertakes to
guarantee the payment of the note should
business the costs may be an item. Cor
porations are made the object of more reg maker of the note fail to pay it when
ule.
ulatory legislation than co-partnerships and
have to pay more taxes, one way or an Ques. 51:—When is a note Protested?
other. (3) The legal rights of minority ANS.:—When it is payable at a bank and
stockholders are sometimes taken advantage has an endorser and is not paid within
of through a rival or enemy coming into banking hours on the day it falls due.
possession of a single share of stock and
annoying the corporation with litigation Ques. 52:—When can a note be protested,
which only a stockholder could force. and when not?
ANS.:—It can be protested with legal
Ques. 44:—What are some of the advan effect only if it has both an endorser and
tages and disadvantages of a Co-partner is payable at a bank, and it can be protested
ship? only after banking hours on the day on
ANS.:—Advantage: which it falls due.
It is the simplest
form of doing business, and very satisfac Ques. 53:-What is the object of Protest
tory if the partners agree, because of their ing a note?
complete freedom of action. ANS.:—To hold the endorser liable. If
Disadvantages: (1) The owners are liable there is an endorser and if it is payable
for the full amount of their assets, regard at a bank, and the note is not protested at
less of the fact that only a portion of their the end of the day it falls due, the en
assets may be invested in this particular dorser cannot be held liable.
business. In the case of a suit for dam
ages or libel this may be a serious consid Ques. 54:—How is a note Protested?
eration. (2) The minority partner frequent ANS.:—The bank presents it for payment
ly may be largely at the mercy of the ma or notifies the maker that it holds it and
jority partner. that it is due at such and such date. If not
(3) The heirs of a deceased
paid by the hour for closing a Notary cer
partner can force dissolution of the busi
Iless. tifies that the note was presented for pay
ment and that payment was refused. The
endorser receives a copy of the protest.
Ques. 45:—If two persons wish to go into
partnership under a certain title, what can Ques. 55:—Can a paper other than an
they do to protect that title from being endorsed paper be Protested?
used by others? ANS.:—It is sometimes done by request,
ANS.:—File in the office of the County but it is meaningless unless there is an en
Clerk a statement of the nature of the busi dorser to be held liable.
ness, the names and signatures of the own
ers, and the title under which it is con Ques. 56:—What is meant by “Accept
ducted. ance” of a Sight Draft?
ANS.:—It means that the person upon
whom it is drawn writes his signature across
Oues. 46:—What is a Syndicate? the face of the draft, with the word “Ac
ANS.:—A combination of capitalists, usu cepted” and with the date of acceptance.
ally temporary, for the conduct of some
financial enterprise. Ques. 57:-What is the effect of Accept
ance of a Sight Draft?
Ques. 47:—What is a Trust? ANS.:—It makes it as binding as a note.
ANS.:—A combination of capitalists Ques. 58:—Why and when are Sight
formed for the purpose of pooling their Drafts employed? Illustrate.
interests in a legal manner. Also, the hold ANS.:—Jones & Co. of Yonkers owe
ing of an estate for the benefit of another. Acker, Merrall & Condit of New York $100.
-
63
The bill is due in 30 days. Instead of wait Ques. 67:—What is a Certified Check?
ing for Jones & Co., who usually are slow, ANS.:—A check which has been certified
to remit, Acker, Merrall & Condit make a by the bank on which it is drawn as being
Sight Draft on them for the amount of the protected by the necessary funds.
bill. The draft is sent to a Yonkers bank
for collection and the draft is presented Ques. 68:—What is the effect of such cer
the day the bill is due. The object of thus tification?
drawing is to induce prompt payment, as ANS.:—The certification by the bank is
it hurts the standing of a business man an assurance that there are on hand suffi
with the bank to acknowledge that he al cient funds with which to pay it and that
lows bills to go unpaid. the bank has taken cognizance of the check
and guarantees to honor memo of

it.

A
Ques. 59:—What is meant by a “Draft the certification made on the depositor's

is
with Bill of Lading Attached”? When is account, and his balance considered to

is
it used? be reduced by that amount, the same as
ANS.:—It is used in C. O. D. transactions though the check had actually been cashed.
usually in connection with freight ship
ments. The consignee cannot obtain the Ques. 69:—Explain the following signa
goods without the Bill of Lading, and he check: “William Smith, by Ed

to
ture

a
cannot obtain the Bill of Lading without ward Anderson, Attorney.”
paying the draft to which it is attached. ANS.:—It means that Edward Anderson
holds from William Smith Power of At

a a
Ques. 60:—What other kinds of Drafts torney under which he has right sign

to
are there besides Sight Drafts? checks for Smith.
ANS.:—Demand Drafts, Time Drafts and
Bank Drafts. Ques. 70:—How payment stopped on

is

a
Lost Check?
Ques. 61:—What is a Demand Draft? ANS.:—By notifying the bank on which
ANS.:—The same as a Sight Draft, ex was drawn, giving number check, date

of
cept that it is payable it
on demand. drawn, amount and whom payable, and

to
requesting pre

to
them not honor

if
it
Ques. 62:-What difference as regards sented. Such notification may be by 'phone
days of grace is there between a Draft pay gain time, but always should be verified
in to

able At Sight and one payable On Demand? writing.


ANS.:—In many States three days' grace
is allowed for payment on a Sight Draft,
Ques. 71:-How does bank know what

a
but this does not apply to a Demand Draft. corpora

of
or
of

members firm officers


a

a
tion are authorized sign checks

to
and
63:-What is a Time Draft?
Ques. whether the signatures such persons are

of
ANS.:—One that allows a certain num genuine?
ber of days for payment after being pre ANS.:—The firm or corporation instructs
sented, as, for example, “30 days after sign
to

to
A is If as

the bank who are authorized


sight.” In a State where grace is allowed, checks. firm, all the members sign
a

this is equal to 33 days. instructions; corporation,


of

the letter
if
a
the letter signed by the duly authorized
Ques. 64:—What is a Bank Draft? corporation also usually trans
ANS.:—It is a substitute for money, is officers.
its by-laws support.
of

mits section
in

sued by a bank, that if issued by an indi


a

The bank requires all persons who are

to
vidual or business house would be called
sign checks specimen signature
to

It is drawn on some other bank furnish


a

a check.
on signature card, which kept on file.
is

with which the issuing bank has funds on


a

deposit. Oues. 72:—Mention circumstances under


which would be an advantage and dis
it

Ques. 65:—What is a Cashier's Check? advantage the following for


to

of

use each
ANS.:—It is a draft issued by a bank and making remittance out of town: Pri
A
a

drawn on itself instead of on some other


vate Check, Bank Draft, Post Office
It is an order on the Cashier, pay
a

bank. Money Order, an Express Money Order


able on demand.
and Registered Letter.
a

ANS.:—Private Check—Advantage: Gains


Ques. 66:—Why and when are Cashier's
Checks issued 2 the interest on the money while the check
transit; no trouble making the re
in

If in

ANS.:—Theoretically, they
is

are issued Disadvantage: an un


to

only for local circulation by a bank of not mittance. sent


known party might be refused.
it

sufficient standing to insure that its checks


would everywhere be accepted as equal to Bank Draft—Advantages: Safe and easily
negotiable. Often issued an accommoda
as

drafts. However, some very large banks


tion without fee. Disadvantages: Have
to

of national reputation issue Cashier's Checks


re
it,

bank for and spot cash


to

for drafts. In issuing a send


is

as a substitute
check payable upon itself the bank avoids quired.
disturbing its deposit with its depository Office Money Order—Advantage:
Post
bank, and in the case of a large bank it is all. Disadvantages: Will forward
of

Safest
asserting its own independent standing and only limited amounts, and charge higher
responsibility. exchange. More liable
to
of

rates lose time


64 -
standing in line than at a bank or express MISCELLANEOUS COMMERCIAL
company office.
TERMS.
Express Money Order—Advantages: Very
safe, exchange charges reasonable and ser Ques. 81:—What Good-will?

is
vice prompt. Disadvantages: Often there
is no nearby office. ANS.:—The monetary value the repu

of
Letter—Advantages: Safe up

its
Registered

of
tation business over and above

a
to $50, and desirable where it is preferable visible assets; the value of business name.

a
to send actual cash up to that amount. Dis
advantages: The limited liability and slow Ques. 82:-Explain “Going Value.”
ANS.:—A term used designate busi

to
ness of the service, including delay in the

a
Good-will,

or
ness actual operation. the

in
mails to the letter itself.
reputation business, has value only

of
a

a
long

so

as
kept going. When
Ques. 73:—What is a Letter of Credit, the business

is
and how is it used? business discontinues, only the physical

a
assets or actual properties

of
ANS.:—A paper issued by a banking con are value.
cern to a traveler entitling him to claim

of
Ques. 83:—What Balance Trade?

is
credit up to a specified amount upon pres ANS.:—The difference value between

of in
entation of same. Each time he secures
the imports and exports country.

a
credit it is entered upon the Letter by the
concern honoring
it,

thus showing how Ques. 84:—What Consignment?

A
is
a
much of the authorized credit has been Consignor? Consignee?

A
used up. ANS.:—A Consignment merchandise

is
sent for sale on commission. The Consign
Ques. 74:—What the difference between the one who ships the goods. The
is

or
is

Negotiable and Non-Negotiable paper? Consignee the one who receives them.
is
ANS.:—A Negotiable instrument one
is

which the holder thereof can transfer to Ques. 85:—What Tare?

is
another by endorsement. Examples: draft ANS.:—The deduction fromthe gross
A

order or bear

to
note payable weight merchandise sold by weight,
or

or

to

of

check
Non-Negotiable paper one payable allow for packing material and leave only
A

is

er.
only Example: note pay the net weight be paid for.
to
to to

the owner.
A

able John Jones, without specifying “or


Ques. 86:—What Ad Valorem?
is

order”; paper issued with the condition


is a

ANS.:—Literally, Latin phrase


or nego
is
to it
to

a
that not be transferred
it

meaning “according value.” indicates

It
tiated.
that custom duties are payable on the value
Hypothe quantity weight

of
Ques. 75:—What meant by rather than on the or
is

cate? merchandise.
ANS.:—To attach to note collateral as
a

remains the prop Ques. 87:—What Bonus?


is

security. Such collateral


a

erty but passes out ANS.:—Extra compensation for efficient


of

of

the control
of

the
performance premium gift
or
of

work.
A

owner while being held security.


as

paid sum originally agreed


of

excess
to in

four common forms upon compensate for efforts above stand


of

Ques. 76:—Name
Collateral Security. ard. Also, extra compensation for the loan
ANS.:—Stocks, money.
of

Bonds, Life Insurance


policies and notes other parties.
of

Oues. 88:—What an Assignment?


of is

ANS.:—The act voluntarily turning


Ques. 77:—Generally speaking, what
is
a

over to another the conduct of business


a

Voucher?
safeguard the interests
to

of

receipt; (2), the creditors.


ANS.:—(1) document
A

which proves the accuracy an account;


of

89:-What
Ques. Insolvency?...
is

(3) written authority for an expenditure. ANS.:—The inability pay liabilities on


to
a

account of insufficient assets.


Ques. 78:—What Bill Lading?
of
is
a

ANS.:—A receipt issued by an agent 90:-What Bankruptcy?


of

Ques.
of is

common carrier for goods accepted for ANS.:—A state insolvency when so
a

transportation specified point. by judicial authority.


to

It

determined
is
a
a

contract which, when transferred third


to
a

Oues. 91:—What the difference between


is

party, becomes an absolute title


to

the Voluntary and Involuntary Bankruptcy?


goods.
ANS.:—In Voluntary Bankruptcy the
bankrupt himself applies
to

to

Ques. 79:—What Warranty? the court


is

In Involuntary
a

be declared Bankrupt.
ANS.:—A Warranty agreement
a
to

an
is

Bankruptcy the application made by cred


assume responsibility
is

certain facts do not


if

itors who wish safeguard their interests.


to

represented.
as

prove
Ques. 92:—When the court issues De
it a

Ques. 80:-What, general sense, Bankruptcy,


in

is

of

cree what effect does


have
a

Bond? upon the bankrupt'sresponsibility for debts?


ANS.:—A signed agreement do or not ANS.:—Unless
to

can be shown that the


it

do certain thing. decree was issued due to fraud or misstate


to

65
ment by the bankrupt, he is absolved from cover damages from any person who sells
all indebtedness up to the date of filing such article or an article infringing upon
the petition. But any indebtedness incurred the idea, unless such person pays an agreed
after the filing of the petition, regardless upon royalty for the privilege. An inven
of whether the discharge has yet been tion is protected from the time the Patent
granted, is unaffected. is applied for, provided that the Patent ul
timately is granted.
Ques. 93:—What is a Receiver?
ANS.:—A person appointed by a court to Ques. 98:—What is a Copyright? How
conduct the business of one who has made is it obtained? -

an assignment. The Receiver acts as an ANS.:—An exclusive right to the publica


agent of the court. tion of an original writing. It is obtained
by sending to the Register of Copyrights,
Ques. 94:—What is meant by Fiduciary? Library of Congress, two copies of the
ANS.:—A person holding a position of writing, accompanied by a fee of $1 and an
trust; a trustee. affidavit. It is then registered. No Letters
of Copyright are issued, but the receipt of
Ques. 95:—What is a Power of Attorney? the Copyright filing is acknowledged, and
ANS.:—A written instrument, duly wit if any person thereafter publishes such writ
nessed and acknowledged, in which one per ing or portion thereof for gain, without
son of legal age authorizes another person agreement of the owner of the Copyright,
of legal age to act for him with the same the latter can proceed for the recovery of
authority that the grantor could act for damages, unless it can be shown that the
himself. The power may be limited in time matter was not original and not entitled to
or extent of power, or may be unlimited, Copyright.
according to the provisions made.
Ques. 99:—What is a Royalty?
Ques.96:-What is a Trade Mark? ANS.:—A percentage paid by a manufac
ANS.:—A distinctive design or title under turer or publisher to an inventor or author
which a business is conducted or an article out of the gross receipts or net profits re
sold. It may be protected from infringe sulting from the sale or use of the article.
ment by registering it in the Patent Office
at Washington. Ques. 100:—What is the meaning of Pro
Rata? Illustrate.
Ques. 97:—What is a Patent? How is it ANS.:—It is a Latin phrase, meaning a
obtained? distribution of money in proportionate
ANS.:—The sole right in an invention. parts. If there are three partners in a
It is obtained by sending a model to the business, and one has a one-sixth interest,
Patent Office at Washington, accompanied one a one-third and one a three-sixths in
by the required fees. If it is found that terest, and there is $1,000 to distribute
the idea involved is original and not an among them, the money is divided Pro Rata,
infringement upon any existing patent, Let each receiving his proportion, one receiving
ters of Patent are issued. After receiving one-sixth of that sum, one one-third and
the Letters of Patent, the Patentee can re the remaining one three-sixths.

66
ACCOUNTANT, NEW YORK CITY SERVICE.

Answers to Examination Questions and Specimen Ques


tions for 4th and 5th Grades.

Municipal Civil Service Commission, Y. 000, divided into 25,000 shares

of of
N. common
ACCOUNTANT, and 5,000 shares preferred the par

of
4th GRADE.
All the common stock

of
(Salary, $1,800 to $2,400.) value $100 each.
exchange for

to
was issued license use

in
Date: July

a
2, 1917. certain patent and $500,000 worth

of
it
a
TECHNICAL. raising funds

to
donated back be used

in
PART I (to be finished by 1 P. M.). for promotion purposes. 500 shares

of
the
preferred stock was sold par and

at
bonus

a
Ques. 1:—Freeman and Guilford formed a pre

of

of
to
of
co-partnership. three shares common one
Freeman invested three
ferred was given with Formulate jour

it.
fifths of the capital and Guilford two-fifths.
How much did each partner invest, the fol nal entries covering these stock transac
tions.
lowing being the present state of affairs:
Assets: Merchandise, $4,700; Cash, $1,875; ANSWER:
Accounts Receivable, $2,840; Bills Receiv Patent Rights $2,500,000

.
.
.
.
.
.
.
.
.
.
.
able, $2,400; Real Estate, $12,500. Unsubscribed Pref. Stock. 450,000

. .
Liabilities: Accounts Payable, $8,000; Cash 50,000
.
.
.
.
.
.
.
.
.
.
.
.
.
.
.
.
.
.
.
.
Bills Payable, $4,625; Mortgage Payable, To Preferred Capital Stock... $500,000
$3,560; Interest, Common Çapital Stock.... 2,500,000
$72.
ANSWER: Treasury Stock $500,000
Freeman, $4,834.80.
Surplus 500,000
Guilford, $3,223.20. from Donated Stock...
Assuming that the business neither has Surplus from Donated Stk. $150,000
enjoyed a gain nor suffered a loss since the Treasury Stock 150,000
partnership was formed.
.
.
.
.
.
.
.
.
.
.
.
.
.
.
.
Ques. 2:—Differentiate between a Trial Ques. 5:—A village made the following
Explain the appropriations for the year 1915, and tax
Balance and a Balance Sheet.

a
uses of each. was levied therefor:
ANS.:—A Trial Balance is a statement Bond Redemption $5,000
.
.
.
.
.
.
.
.
.
.
.
.
.
.
.
.
proving the equality of the debits and cred Bond Interest 2,000
.
.
.
.
.
.
.
.
.
.
.
.
.
.
.
.
.
.
.
double entry ledger, and Salaries .
its

to

used 6,700
in

is
a

.
.
.
.
.
.
.
.
.
.
.
.
.
.
.
.
.
.
.
.
.
.
.
.
.
.
show that the correct amounts have been Contingent Expenses 1,000
................
.
.
.
.
.
.
.
.
.
.
.
.
.

the proper sides the ledger ac Police


of

4,000
to

posted
.
.
.
.
.
.
.
.
.
.
.
.

counts (provided no compensating errors Care of Streets. 3,000


.
.
.
.
.
.
.
.
.
.
.
.
.
.
.
.
.
.

have been made). Lighting 2,500


.
.
.
.
.
.
.
.
.
.
.
.
.
.
.
.
.
.
.
.
.
.
.
.
.

Balance Sheet statement showing Education 7,500


A

is
a

.
.
.
.
.
.
.
.
.
.
.
.
.
.
.
.
.
.
.
.
.
.
.
.

the resources, liabilities and net worth of


concern, and show the condi
to

used $31,700
is
2

tion of the business at certain date. Set up the accounts necessary


to
a

show
Ques. 3:—How may the profit and loss these various appropriations.
of

business be ascertained from set of ANSWER:


a

books wherein no accounts are kept with Taxes for Budget Requirements
is,

property, that by what known as the


is

single entry method? Explain how such $31,700


|
a

Appropriation—Bond Redemption
books may be converted
of

to

set double
-
a

entry set.
$5,000
ANS.:—An inventory
.
of

merchandise and Appropriation—Bond


.

Interest
other property should be taken, and state
a
of

ment resources and liabilities made (the $2,000


latter must include the net investment). Appropriation—Salaries
If

the resources exceed the liabilities the dif


ference represents the profit; the reverse, - $6,700
if

the balance determines the loss. Appropriation—Contingent Expenses


single entry ledger
In

order
to

convert
a

$1,000
into double entry ledger, journal entry
Appropriation—Police
a

debiting the resources and crediting the lia


capital (including the former
as

bilities
in

$4,000
a

the loss, any, and the latter the gain, Appropriation—Care


in
if

of

Streets
there was one, and posting the ledger
to
if

only those items not previously “set up”), $3,000


|

accomplishes the desired result. The loss Appropriation—Lighting


gain may be posted the Profit and
to
or

$2,500
the Capital account.
or
to

Loss account
Appropriation—Education
Ques. 4:—A, corporation was organized
an

capital $3,000, $7,500


of

with authorized stock


|

67
verify ning the present accounting period the

to
of of
Ques. 6:—How does an auditor
the cash receipts and payments made during fire by 1.20,

to
date order determine

in
a period under review? the goods sold during that time.

of
Of what does the the cost
verification of the cash on hand at the end This amount subtracted from the inventory
the beginning the present period, plus

of
at
of a period consist?
ANS.:—First, by checking back from the net purchases, any, since stock-taking,

if
ledger to the cash book all receipts and will give approximate cost

of
merchandise
fire. This method presupposes

of
posted during the period;

at
disbursements time
then, by comparing the receipts shown by that reasonably normal price conditions
the cash book with the deposits entered were prevalent the years used for ascer

in
in the bank pass book (or preferably state taining average rate profit.

of
ment); checking the payments with the
previous

of
checks returned to the bank for payments (b) Determine from number

a
closing the

of
by check and receipts (vouchers) for cash inventories (taken time

of at
payments. - books) the number each article on hand
The cash on hand is verified by compar and the cost thereof. The sum of the total
cost divided by the sum

of
ing with the balance shown by the bank the different
statement or pass book, making allowance quantities on hand will give fair average

a
for checks not returned to bank, plus cash Multiply the

of

of
the cost each article.
and checks in safe. number of each kind on hand time of

at
fire (as shown by the card loose-leaf

or
Ques. 7:—Explain how you would treat record) by its average cost, and find the
an account for “Patents” in the profit and sum of the results so obtained. Here, too,
loss account and the balance sheet of a cor assumed that prices were reasonably

is
it
poration. uniform during the years used for figuring
ANS.:—The expired portion should be average costs.
charged to profit and loss, and the remain
der appear in the balance sheet as an asset. (c) From the monthly inventory immedi
ately preceding the date on which the fire
occurred, subtract the cost of sales from
Ques. 8:—A company receives a large inventory fire (taken from

to
of

of
date time
number of notes (bills receivable) from its
customers and discounts or sells them in sales record), and add purchases, labor and
other manufacturing expense during said
order to maintain a sufficiently large cash period.
balance. What accounts, if any, should ap
pear, to provide for these discounted notes
to satisfy you as an auditor, and how should Ques. 10:-Define what meant by bad

is
they appear on a balance sheet? debts and doubtful debts, and explain the
ANS.:—Notes Receivable and Notes Re difference between them, there any.

is
if
The latter can appear What the duty an auditor regard
of

in
is

ceivable Discounted.
in the balance sheet in various ways, viz.: to them?

1st—As a deduction from Notes Receiv ANS.:—Bad debts are those which are
hopeless by reason the Statute
of
able. either
Limitations, discharge bankruptcy or
of

in
2nd–In a footnote. compromise accepted.
3rd—As a contingent liability. called when they
so

Doubtful debts are


In the third method, which is considered are considerably overdue but not yet con
the best, there is shown the total asset from sidered “bad.”
this source and, as a contingent liability, the duty
of

write off
to
It

the auditor
is

the amount of notes discounted. the bad debts, where no reserve has been
created for that purpose, by charging Profit
Ques. 9:—If you represented an insurance and Loss and crediting the individual debt
company, how would you determine the ors' accounts. Where reserve ex
in
is
a

value of a stock of goods on hand at the istence, the debtors' accounts should be
date of a loss by fire under each of the closed out against the reserve account. Fur
following conditions, assuming that the ther, the auditor should make provision for
books and records mentioned had not been
destroyed: doubtful debts (where no reserve exists)
by charging “Profit and Loss” and credit
(a) Where inventories are taken only once ing “Reserve for Doubtful Debts” or “Re
a year at the annual closing of the books. serve for Bad and Doubtful Debts.” The
(b) Where a card or loose-leaf record is method described above conformity
in
is

kept of quantities only of each article. with popular practice among public account
antS.
(c) Where monthly book inventories are
arrived at by extending cost of sales in the Another method follows: Transfer
as
is

sales records. all doubtful accounts from the current ac


ANS.:—(a) Ascertain the average yearly account entitled
to

counts receivable an
gross profit on sales for a number of fiscal “Doubtful Debts,” on the asset side. When
years immediately preceding
to

an account ascertained be “bad”


is

the one in
it
is

which the fire occurred, and if the result the “Doubtful Debts” ac
of

credited out
is,

say, per cent manufacturing cost, counts and charged


20

“Reserve for Bad


to
of

sales from the begin Debts.”


of

divide the amount

68
PART II
(to be finished by 4.30 P.M.) Cash paid to creditors. . . . . . . . . . . . . 27,000
Notes Payable paid at maturity.... 9,000
Ques. 11-14:—The following items com Mortgage reduced to $5,000 on June
prise the balance sheet of J. & C. Jobbers
30 and interest paid in full from
as at January 1, 1915: January 1, 1915.
Investments . . . . . . . . . . . . . . . . . . . . . . $25,000 Bank balance June 30, 1915, at close
Cash . . . . . . . . . . . . . . . . . . . . . . . . . . . . . 12,000 out
of business—no checks
Accounts Receivable . . . . . . . . . . . . . . 18,000 18,
standing
Accounts Payable . . . . . . . . . . . . . . . . 13,000
Cash discounts on sales. . . . . . . . . . . .
Notes Payable . . . . . . . . . . . . . . . . . . . . ,000
Cash discounts on purchases. . . . . . .
Loans Receivable . . . . . . . . . . . . . . . . . 5,000
Notes Receivable on hand June 30,
Office Equipment . . . . . . . . . . . . . . . . . 5,000

#
1915
Mortgage Payable (5 per cent). . . . . 15,000
Notes Receivable discounted at bank
Inventory .. ... . . .. . .. .. . .. .. . .. .. ,000 Inventory June 30, 1915 (cost price)
Notes Receivable . . . . . . . . . . . . . . . . . 6,000
Loans Receivable, Investments and
Capital 50,000
.. .. . .. ... .. . .. ... . .. . .. .. . Office Equipment are the same
On July 1, 1915, a fire partially destroyed as at January 1.
the books of account that had been fully Notes Payable issued during six
posted in anticipation of preparing financial months 8,00 0

statements. The trial balance, which was 3 per cent income on Investments
collected for six months on
in proof, had been lost. From the books
and other records the following facts were June 30.
ascertained: Prepare a balance sheet showing the net

-
Sales-net . . . . . . . . . . . . . .. .. .. . .. .. $48,000 worth of the partnership as at June 30,
1915.
Gross profit on sales 20 per cent º
of sales. ANSWER:
J. & C. Jobbers
Balance Sheet as at June 30, 1915.
Assets Liabilities.
Cash . . . . . . .. . . . . . . . . . . . . ." $18,000 Notes Payable. . . . . . . . . . . . $3,000
Notes Receivable. . $12,000 Accounts Payable......... 14,020 $17,020
Less N. R. Disctd..
Accounts Receivable......
-— 7,000 5,000

15,015 $38,015
Mortgage Payable
Proprietorship—
.............. 5,000

-
Capital or Net Worth....... 60,595
Inventory . . . . . . . . .
...

.. $9,600
.
.

Office Equipment. 5,000


.
.
.
.
.
.
.
.

Loans Receivable. 5,000 19,600


.
.
.
.
.
.
.
.

Investments 25,000
.
.
.
.
.
.
.
.
.
.
.
.
.
.
.
.
.
.
.
.

$82,615 $82,615

Working Sheet
Trial Balance

Purchases $37,000 Sales $48,000


.
.
.
.
.
.
.
.
.
.
.
.
.
.
.
.
.
.
.
.
.
.
.
.
.
.
.
.
.
.
.
.
.
.
.
.
.
.
.
.
.
.
.
.
.
.
.
.
.

Interest on Mortgage. 375 Notes Payable 3,000


.
.
.
.
.
.
.
.
.
.
.
.
.
.
.
.
.
.
.
.
.
.
.
.
.
.
.
.
.

Investments 25,000 Mortgage Payable 5,000


. .
. .
- .

.
.
.
.
.
.
.
.
.
.
.
. ..
. .
. .
. .
. .
. .
. .
. .
. .
. .
. .

.
.
.
.
.
.
.
.

Loans Receivable 5,000 Discount on Purchases. 980


.
.
.
.
.
.
.
.
.

Office Equipment 5,000 Notes Receivable Discounted.... 7,000


.
.
.
.
.
.
.
.
.
.
.
.
.
.

Cash 18,000 Income on Investments. 750


.
.
.
.
.
.
.
.

..............
. .
. .
. .
.
.
. .
. .
. .
. .
. .
. .
. .
. .
. .
. .
. .
.
.
.
.
.
.
.
.
.
.
.

Discount on Sales. 360 Accounts Payable 14,020


Notes Receivable 12,000 Capital 50,000
.
.
.
.
.
.
.
.
.
.
.
.
.
.
.
.
.
.
.
.
.
.
.
.
.

.
.
.
.
.
.
.
.
.
.
.
.
.
.
.

Accounts Receivable 15,015


.
.
.
.
.
.
.
.
.
.
.
.

Inventory 11,000
.
.
.
.
.
.
.
.
.
.
.
.
.
.
.
.
.
.
.
.
.
.

$128,750 $128,750

Trading and Account.


P.

L.
&

$37,000 $48,000
Purchases Şāles
.
.
.
.
.
.
.
.
.
.
.
.
.
.
.
.
.
.
.
.
.
.
.

.
.
.
.
.
.
.
.
.
.
.

.
.
..
.
.
.
.
.
.
.
.
.

Interest on Mortgage. 375 Discount on Purchases. 980


.
.
.
.
.
.
.
.
.
.
.
.
.
.
.
.
.
.
.

Discount on Sales. 360 Income on Investments. 750


.
.
.
.
.
.
.
.
.
. .
. .
. .

1915..........
.
.
.
.
. .
. .
. .
. .
. .
. .
. .

Inventory Jan. 11,000 Inventory June 30, 9,600


1,

1915.
10,595
Balance (Gain)
.
.
.
.
.
.
.
.
.
.
.
.
.
.
.
.
.

$59,330 $59,330

69
Ques. 15–17:—The Do-Well Manufactur On June 30, 1916, a balance sheet was ap
ing Company placed in service on January proved by the three interested parties as
1, 1916, a piece of machinery which it was follows:
estimated would become obsolete and use
less at the end of six years. The original Assets.
cost of the machinery was $45,000 and the Plant and Equipment...... $45,000
board of directors decided to provide an Goods on hand. . . . . . . . . . . . 20,000
nually a sinking fund, which including in Accounts Receivable...... 13,000
terest will amount to the original cost at Bills Receivable........... 7,000
the end of the life of the machine as esti Cash on hand and in bank. 11,000
mated. The sinking fund is to be deposited
with a trust company on December 31 of $96,000
each year during the life of the machinery;
interest is to be credited by the trust com
pany at the rate of 4 per cent per annum. Liabilities.
Show how the amount of the annual sink Bills Payable ............ $5,500
ing fund contribution may be determined, Accounts Payable . . . . . . . . 7,500
and prepare a statement for the board of Jones' Account . . . . . . . . . . . 3,500
directors, to prove that the amount so ob Smith's Account .......... 2,500
tained is correct. Brown's Account . . . . . . . . . 2,000

ANSWER: $21,000
$45,000
= $6,784.29 annual contribution Capital Accounts.
$6.632975 Ans.). Jones . ... . .. ... .... . .. .. . 7,500
Smith .. .. . ..... ... . .. .. .. 37,500 75,000
At 4% interest compounded annually.
Operation:
$96,000
$1 deposited Dec. 31, 1916, will on
Dec. 31, 1921, amount to. . . . . . . $1.216652
$1 deposited Dec. 31, 1917, will on . Thereafter the business was sold as a go
Dec. 31, 1921, amount to . . . . . . . 1.169858 ing concern and the partnership dissolved.
$1 deposited Dec. 31, 1918, will on The purchaser assumes all outside liabili
Dec. 31, 1921, amount to. . . . . . . 1.124864 ties and pays $112,000 cash, of which the
$1 deposited Dec. 31, 1919, will on plant and equipment is valued at $60,000.
Dec. 31, 1921, amount to. . . . . . . 1.0816 Prepare a statement of the settlement of
$1 deposited Dec. 31, 1920, will on the partners' accounts, using the balance
Dec. 31, 1921, amount to. . . . . . . 1.04 sheet given as a basis.
$1 deposited Dec. 31, 1921, will on
Dec. 31, 1921, amount to. . . . . . . 1.00 ANSWER:
Statement of the Settlement of the Accounts
$6 deposited as above will on of Jones, Smith and Brown.
Dec. 31, 1921, amount to. . . . . . . _$6.632975 Jones Smith Brown Total
Net invest
Proof: In ent (in
cluding per
$6,784.29 deposited Dec. 31, 1916, sonal liabil
- --
will on Dec. 31, 1921, amount to $8,254.12
. . . . . . . $41,000 $40,000 $2,000 $83,000
$6,784.29 deposited Dec. 31, 1917, Profit on
will on Dec. 31, 1921, amount to 7,936.65 Sale of Plant
$6,784.29 deposited Dec. 31, 1918,
& Equipment 5,000 5,000 5,000 15,000
will on Dec. 31, 1921, amount to 7,631.40 Good-will.. 7,000 7,000 14,000
$6,784.29 deposited Dec. 31, 1919,
will on Dec. 31, 1921, amount to 7,337.88
Cash Pay
$6,784.29 deposited Dec. 31, 1920, ment for the
will on Dec. 31, 1921, amount to 7,055.66
Business... $53,000 $52,000 $7,000 $112,000
$6,784.29 deposited Dec. 31, 1921,
will on Dec. 31, 1921, amount to 6,784.29

6 Annual Installments deposited


as abovewill on Dec. 31, 1921, Municipal Civil Service Commission, N. Y.
amount to . . . . . . . . . . . . . . . . . . . . $45,000.00
ACCOUNTANT, 4th GRADE.
Ques. 18-20:—Jones and Smith are equal (Salary, $1,800 to $2,400.)
partners, and admit their superintendent
Brown as a partner. Brown is to share Date: March 23, 1909.
equally with the other two in the profits, TECHNICAL.
but does not furnish any capital. Jones and
Smith stipulate that they shall be entitled (To be finished by 2 P. M.)
to any good-will which may accrue to the
business in case it is sold, or the partner 1. A is the proprietor of a shoe factory,
ship dissolved. and desires to secure new capital in the

70
business. B, who intends to become a part Traveling expenses..... 399.82
ner and invest $20,000, has had no business Freight outward.

...
248.71
In order to safeguard and pro

.
.
.
.
experience. Disc. on purchases. 1,148.21
........

.
.
.
.
tect his interests he consults you in regard Taxes 268.00

.
.
.
.
.
.
.
.
.
to the partnership agreement. Set out Stationery and printing. 89.51
clearly what suggestions you would offer Reserve for bad and
from the accounting standpoint. doubtful debts 1,142.86
.........

.
.
.
.
.
.
.
2. In an audit of a business, what are Sales account 75,293.87
an auditor's duties regarding (a) Cash, (b) Elkins drawing acct.... 1,400.00
Accounts Receivable, (c) Unfinished Con Williams drawing acct. 1,750.42
tracts?
3. (a) What contracts are required to be $154,027.34 $154,027.34
in writing? (b) How may debts be revived
Goods on hand December 31, 1908, $17,
that have become outlawed by Statute of Charge per cent depreciation on

10
438.92.
Limitations?
furniture and fixtures. Credit capital ac
4. A corporation which receives a large count with per cent interest. esti

It
is
8 5
number of notes from its customers, dis mated that per cent the accounts re

of
counts them when in need of funds. Ex ceivable are bad or doubtful. Profits to be
plain fully how the ledger accounts should divided proportion capital invested.

to
in
be kept to show the liability of the com 9-10. The Consolidated Manufacturing
pany as indorser on the paper discounted. Company organized under the laws

ofof of
of is
5. Mention the requirements to which a the State New York, with capital

a
negotiable instrument must conform. An $2,500,000, consisting

of
10,000 shares
alyze the following instrument with respect preferred and 15,000 shares

of
common
to its negotiability: stock, all par value The com
of

of
$100.

a
pany has entered into contracts with the
New York, July 4, 1908. In
of

firm Jones Jackson and with the

&
Sixty days after the death of my father dependent Manufacturing Company for the
purchase their plants and other assets,
of

I promise to pay to Jasper Robinson at


in

the First National Bank, New York, five


consideration of
$1,000,000 and $1,500,000,
respectively, payable two-fifths preferred

in
hundred pounds sterling, or its equivalent
stock and three-fifths common stock. The
in U. S. Government 2 per cent bonds of liabilities in
of

to
1908. the vendors are be assumed
by

the purchaser. The vendors agree

to
H. P. S. the treasury
to

donate
of
the Consolidated
Company per cent
15

of

the common stock


6. It
has been proposed that the Munici received by them furnish additional
to

pal Civil Service Commission enact a fee working capital.


of $1 when each application is filed, and Five hundred shares the stock so do
of

that the fee be returned to those candidates


nated are sold from the treasury $40 per
at

who are successful in passing the examina


at of

share. $500,000 first mortgage bonds are


tion. The number of candidates averages 90, the purchaser receiv
issued and sold
15,000 a year. Outline fully and give forms ing with each bond
of

per cent
15

bonus
a

of a complete system that you would install in common stock.


if such a rule were adopted. Below are given the balance sheets
of

the
7-8. From the following trial balance of vendors. Frame the closing entries for the
the books of Elkins & Williams on Decem
Jones Jackson,
of

books and the opening


&

ber 31, 1908, prepare (a) Trading Account;


entries for the Consolidated Company, and
(b). Profit and Loss Account; (c) Balance prepare balance sheet
of

the latter.
Sheet: -
Balance Sheet—Jones Jackson.
&

John Elkins capital acct. $40,000.00


Assets.
Chas. Williams cap. acct 20,000.00
Inventory, Jan. 1...... $23,450.32 Real estate and building........... $246,000
ent . . . . . . . . . . . . . . . . . . 1,200.00 Plant ................ 137,000
.
.

.........
.
.
.
.
.
.
.
.
.
.

Office salaries . . . . . . . . . 2,952.86 Qºsh 28,000


.
.

Bills receivable ...................


.
.
.

.
.

.
.
.
.
.
.
.
.
.
.
.

Bills payable . . . . . . . . . . 7,896.18 76,000


Cash in bank.......... 10,876.91 Accounts receivable .............. 43,000
Petty cash . . . . . . . . . . . . 74.53
Horse, wagon and harness........ 7,000
Purchases . . . . . . . . . . . . . 49,871.33 Jones drawing account............ 8,000
Disc. on sales. . . . . . . . . . 1,652.81 Jackson drawing account.......... 5,000
Legal expenses . . . . . . . . 1,349.17
Advertising . . ... . . .. .. 986.72 $550,000
Furniture and fixtures... 1,487.00
Accts. receivable . . . . . . 29,034.69 Liabilities.
Packing and shipping... 1,372.36 Jones' capital account............. $300,000
Accts, payable . . . . . . . . . 7,586.22
Jackson's capital account.......... 200,000
Inv. Bldg. and mtge. . . . 24,000.00 Bills payable ..................... 50,000
Com. Acct. (salesman). 1,275.00
Insurance . . . . . - - - - - - - - 287.18
$550,000
Int, on Bldg. and mtge. 960.00

71
Balance Sheet—Independent Manufacturing 4 months at 9% per cent per month, $5,000
Company. for 10 months 15 days at 7 per cent, and
the balance for the full time at 5% per cent
Assets. per month. When the first loan was due
Real estate and building. . . . . . . $365,000
. . .. he invested its proceeds in wheat at $1.60
Plant . . . . . . . . . . . . . . . . . . . . . . . . 252,000
... . per bushel, and 7 months 15 days later sold
Cash . . . . . . . . . . . . . . . . . . . . . . . .. .
. . . 48,000
Bills receivable . . . . . . . . . . . . ... .
. . . 37,000
the same at $1.80. How much did he gain?
Accounts receivable . . . . . . . . . . 115,000
. . ..
Horse, wagon and harness. . . . . . . . 9,000
Good-will . . . . . . . . . . . . . . . . . . . . . . . . 50,000

$876,000 Municipal Civil Service Commission, N. Y.


ACCOUNTANT—5th GRADE.
Liabilities (Salary, $2,400 and up.)
Capital stock . . . . . . . . . . . . . . . . . . . . . $700,000
Surplus . . . . . . . . . . . . . . . . . . . . . . . . . . 100,000 Date: May 15, 1912.
Undivided profits . . . . . . . . . . . . . . . . 36,000
TECHNICAL–Weight
Loans . . . . . . . . . . . . . . . . . . . . . . . . . . . . 40,000 7.

$876,000 PART I (to be finished by 12.30).

NOTE:—The answers to the questions in


this examination of May 15, 1912, are a
-
MATHEMATICS. copy of the author's papers in that exam
ination. As the official rating received was
1. Find the average due date of the fol 97 per cent, they are presented here intact.
lowing account:

O. B. Hutchins. Theory of Accounts.


Ques. 1:—On what general principles

is
1908
...

Jan. 1 To balance . . . . . . . . . . . . . . $694 double entry bookkeeping based?


Jan. 28 To mdse. at 30 days. . . . . . 1,400 ANS.:—Double entry bookkeeping

is
Mar. 20 To mdse. net. . . . . . . . . . . . . 510 based on the general principles debit and

of
By cash . . . . . . . . . . . . . . . . 550 credit. This method bookkeeping recog

of
Feb. 20
By drafts at 20 days..... 700 nizes and records two phases every trans

to
Feb. 28
Mar. By cash . . . . . . . . . . . . . . . . 845 action, one phase being opposite the other

to
31
tendency, one debit (or several debits)
in

2. I bought a farm for $5,600 and ex and the other credit (or several credits).
a

pended 8 per cent as much for improve balanced by


to

debit one account


A

is

a
ments. The cost of the stock for the farm credit to another account.
was 225 per cent of the sum expended for Thus, double entry bookkeeping, the
in

improvements. These sums were drawn aggregate always


of

all the accounts

in
is
from my bank and constituted 25 per cent equilibrium,
of

state the total debits


a

of my original deposit. The net profit from equalling the total credits.
the farm for two years was $500, which sum
This recognition two phases every
of

I deposited. I then sold out at 10 per cent to


transaction calls into existence an extra set
advance on total cost, and deposited the
of accounts, which are accounts name
in

proceeds. What was the bank account be only, nominal (sometimes called Profit
or

fore the purchase? What was per cent of


increase in bank account? and Loss) accounts. This enables us to
find the profit loss through two meth
or

3. A debt of $7,500 is to be canceled by ods: one, the resource and liability method
proceeds of two notes. One note is for single entry), and the other the
(used
in

$6,000 maturing in one month. The other nominal accounts method. We therefore
is to be drawn at 30 days for such a sum proof the accuracy of our
to
as

have
that when discounted on its date the pro
a

work.
ceeds of both notes will pay the debt. What
shall be the face of the second note, the chang
Ques. 2:—Describe the process
to of

rate of discount in both cases being 5 per ing books from single
of

set double
a

cent? entry. Give an example.


4. A contractor undertakes to execute a ANS.:—A set changed from
of

books
is

certain work in a given time; he employs single entry double entry simply by mak
to

55 men who work nine hours daily; when ing journal entry and posting those items
a

three-quarters of the time has expired he the ledger which are not already con
to

finds that only three-sevenths of the work gather the facts


In

to

tained therein. order


is done; how many men must he now em make this journal entry,
to to

from which
it

ploy eleven hours a day to fulfil his con necessary inventory prop
of
an

take
is

tract? erty accounts not shown the books and,


in

5. A speculator borrowed $15,000 for one sometimes, compilation ex


to

of

make
a

year at 6 per cent and loaned $4,750 for penses from the cash book.

72
Example: Accounts—Police Department.
Journal Entry. Receipts. Disbursements.
Sundries Dr. Fines of Police Court. Salaries.
Budget Appropria- Real Estate—Station
L.F. To Sundries tions. Houses—Horses.
1 Cash . . . . . . . . . . . $2,000 Maintenance of Sta
Real Estate . . . . 20,000 bles, incl. board

of
25
50 Expenses ....... 850 horses and re
75 Mdse. Inventory. 8,000 pairs.
(posted) Accts. Rec'ble... 7,000 Equipment.
(posted) To John Jones, Prop.. $32,850 Ma in ten ance of
(posted) Accts. Payable.... 5,000
Equipment, incl.
repairs.
Ques. 3:-A mining corporation has a:
sets comprising, among others, leases, good Accounts—Department

of
Education.
will, patents, rent and royalties paid in ad Receipts. Disbursements.
vance. How would you deal with them in Budget Appropria- Salaries.
the profit and loss account and balance tion. Real Estate—School
sheet? Bldgs.
ANS.:—Lease—This should be depreci Ma in ten ance of
ated over the term of its existence, so that School Bldgs.
will have been entirely
its

at expiration Supplies.
it

wiped off the books. The Profit and Loss


account should periodically Accounts—Dept. Streets and Highways.
be

of
charged with
depreciation, Receipts. Disbursements.
of

the amount and the Balance


an

Sheet should carry Appropria-


of as

asset its true


at

Budget
it

Salaries.
the financial state Paving.
as

of

value the date tion.


ment. Repair

of
Streets

&
mining company depreci Highways.
of

Good-will
a

Equipment.
of

ates with the extraction ore which was


lying Horses.
be

the mine.
to

estimated should
in

It

handled exactly like Lease Carts.


be

both Profit
in

and Loss Account and Balance Sheet. Ma in ten nee of

a
Patents are an asset and should appear Horses (board).
such, but like Lease
as

on the balance sheet Accounts—Department Courts.

of
and Good-will should be depreciated
(Court) Receipts. Disbursements.
through the Profit and Loss Account, due -
mining com Fines for violation of Salaries.
the peculiar situation
of
to

various municipal Real Estate—Court


pany.
laws, as well as Houses.
Rents and Royalties paid advance are Ma in ten ance of
in

civil cases.
advance charges which should be tempo Budget Appropria-
rarily carried Court Houses.
assets, and are trans
be
to
as

t1On. Supplies.
the Profit and Loss Account
to

of

ferred
the period which they accrue. Auditing.
in

Ques. 5:—State the objects


to

Ques. 4:—The budget municipality be attained


of

by an audit.
a

includes appropriations for five separate de


ANS.:—The objects be obtained by an
to

you
as

partments... Set up such accounts


audit are:
believe will cover the financial operations
The detection of fraud.
3. 2. 1.

incident thereto, with supposed receipts and


The discovery
of

disbursements. technical errors.


af
of

of

ANS.:—Such departments would be: The verification condition


a

shown by financial statement sub


as

fairs
a

Fire.
5. 4. 3. 2. 1.

mitted.
Police.
Education.
Ques. 6:—In case an unexplained ab
É.
of

Streets and Highways. bookkeeper who has the en


of

sence
a

Department
of

Courts. tire charge books, state the steps


of

of

set
a

that should be taken to determine whether


Accounts–Fire Department.
or not irregularities had been committed.
Receipts. Disbursements. ANS.:—I would in this case first make
a

Fines for violation reconciliation of the bank balance as shown


of

Salaries.
Fire Ordinances. Real Estate—Stables by the bank's statement with the amount
Appropria- —Horses. be on deposit by the check book,
to

Budget shown
tion. Maintenance of Sta which should also agree with the cash book,
bles incl. repairs. after taking into consideration all the checks
which have not been pre
e.,

outstanding
(i.

Equipment.
Ma in ten ance of the bank for payment).
to

sented
Equipment incl. would then check back from the custom
I

all

repairs.
to

ers' accounts the cash book the cred:


its, see that all remittances
to

Supplies. have found

78
their way into the bank, allowances at the State of New York to conduct a manufac
same time being vouched. turing business. The authorized capital is
Then I would verify the additions of the $500,000, divided into $250,000 common and
sales and the cash book, and construct a $250,000 preferred
stock, par value of shares
control account (if there is none already on $100. Five incorporators subscribe each
the books) to see if the actual accounts re for one share of common stock at face value.
ceivable agree with the controlling account. John Peters, one of the incorporators, pur
Statements would be sent to customers chases from three manufacturing companies
for verification, to be returned to the au their complete plants for $499,500 and trans
ditor's office. fers said plants to the Washington Com
I would analyze and check to some extent pany for the remaining $499,500 of common
the nominal accounts, to make sure that and preferred stock and $100,000 of first
cash was not extracted and charged to these mortgage 5 per cent bonds out of a total
accounts. issue of bonds amounting to $150,000, leav
ing $50,000 of bonds in the treasury. The
Ques. 7:—A company takes a large num incorporators then pay in cash for their re
ber of notes (bills receivable) from its cus spective subscriptions.
tomers and when in need of funds discounts
The individual assets acquired are as fol
or sells them. What accounts, if any, should lows:
appear to care for the contingent liability
Land and Buildings. . . . . . . . . . . . . . . $75,000
thus created to satisfy you as auditor? Plant and Machinery. . . . . . . . . . . . . .
ANS.:—An account called “Notes Receiv Tools, Equipment and Fixtures.... 50,000
able Discounted” or “Notes Receivable Inventories . . . . . . . . . . . . . . . . . . . . . . . 00,000
Sold” should be opened and credited at the Accounts Receivable . . . . . . . . . . . . . . ,000
time cash is received, for the full amount Cash .. .. .. ... . .. ... .. .. .. .. .. .. .. 12,000
of the note, in order to show the contingent
liability. Required: (a) Opening entries for the
At maturity of the instrument, when ad books for the Washington Company. (b)
vice is received that it has been paid, then Initial balance sheet, showing the company's
a transfer entry can be made, debiting financial condition.
“Notes Receivable Discounted” (or Sold)
and crediting “Notes Receivable” account,
ANSWER:
thus wiping out the item from both ac
counts. New York, May 15, 1912.

The Washington Company


ues. 8:—Give examples of such assets Organized under the Laws of the State of
and liabilities not usually found on books
New York with an authorized capital
of accounts, as should be considered by the stock of $500,000, divided as
auditor when preparing an income and follows:
profit and loss account at the close of a
fiscal period. Preferred Stock—2,500 shares, par
ANS.:—Items paid in advance, items ac value $100 . . . . . . . . . . . . . . . . . . . . . $250,000
crued but not due, inventories, depreciation Common Stock—2,500 shares, par
and reserves are not usually found on books value $100 250,000
of account, but should be considered by the
auditor when preparing an income and profit Total—5,000 shares. . . . . . . . . . . . $500,000
and loss account at the close of a fiscal
period.
Subscription Account $500
Examples: To Capital Stock (Com
Assets. mon) $500
Rent paid in advance. for subscription to 1 share
Royalties paid in advance. of common stock at par by
Insurance paid in advance. each of the following in
paid in advance. corporators:
Commissions
Interest (bonds and investments, accrued John Peters, 1 share. . $100
1 share... 100
but not due). .
1 share... 100
Merchandise inventory.
1 share... 100
Liabilities. 1 share... 100
Bond interest accrued but not due.
Note interest accrued but not due. 5 $500
Wages accrued but not due.
st
Rent accrued but not due. -
Commissions earned but unpaid. Plant and Sundry Assets. . $599,500
Consisting of Land and
PART II (to be finished by 4 P. M.) Buildings, Plant and Ma
chinery, Tools and Equip
Practical Accounting.
ments and Fixtures, Inven
I tories, Accounts Receivable,
Ques. 9 and 10:—The Washington Com Cash and Good-will.
pany is organized under the laws of the To John Peters, Vendor.. $599,500

74
For his surrender of his
- -
right, title and interest
- Sundries—
to To Plant & Sundry Assets $599,500
-
the above mentioned as- Land and Buildings. . . . . . . $75,000
Plant and Machinery. . . . . . 200,000
sets. Tools, Equipment & Fixt. 50,000
Inventories . . . . . . . . . . . . . . . 100,000

:
--
Accounts Receivable. . . . . . 33,000
Y Cash . ... .... .... ... . . 12,000

... "...”.
John Peters, Vendor. . . . . . $599,500 Good-will . . . . . . . . . . . .

...
...
129,500
- Transfer—To place indi
To Capital Stock (Pref.). $250,000 vidual assets on books un
Capital Stock (Com.). 249,500
-
tion of Board of Directors

--
First Mtge. 5% Bonds 100,000
found on page of Min
Payment ute Book.
of

full a/c.
in

Dr. Cash.
Account to
General
Date be credited Explanation L.F. Net Cash Ledger
May Sundry Assets
“ 15

Plant
&

See Journal Page $12,000.00

5
$12,000.00
Subscription a/c

1
Paymt. by incorporators 500.00 500.00
11

Ques. and 12:—The American Trad


ing were paid, and the expense

of
Company has been granted permission and liquidation realization

to
to dissolve its corporate existence. You are amounted $3,200.
Required: (a). All closing entries for the
.

consulted about the procedure closing its


of

dissolution of
the company. (b) state

A
books and are given the following informa
of

tion: ment realization and liquidation showing


the amounts distributed

to
the stockhold
An the ledger on July ers.
of

abstract 15, 1910,


discloses:
Land and Building. ANSWER:
...

$30,000
.
.
.
.
.
.
.
.
.
.
.
.
.

Plant and Machinery. 50,000


Journal Closing Entries.
.
.
.
.
.
.
.
.
.
.
.
.
.

Merchandise Inventory 22,500


.
.
.
.
.
.
.
.
.
.
.

Notes Receivable 10,500


New York, May
.
.
.
.
.
.
.
.
.
.
.
.
.
.
.
.

Accounts Receivable 16,800 15, 1912.


.
.
.
.
.
.
.
.
.
.
.
.
.
.

Contingent Fund 15,200


Reserve for Depreciation
of
.
.
.
.

.
.
.
.
.
.
.
.
.

Mortgage Bonds (on machinery and


.
.
.
.

Plant Machinery.
&

plant) $9,500
25,000
.
.
.
.

To Plant Machinery
&
.
.
.
.
.
.
.
.
.
.
.
.
.

$9,500
.
.
.
.

Accrued Interest on the mortgage.


.
.
.
.
.
.
.

52
Notes Payable 27,000
.
.
.
.
.
.
.
.
.
.
.
.
.
.
.
.

Accounts Payable
.
.
.
.

Reserve for Depreciation


of

28,000
Buildings ..............
.
.
.
.
.
.

.
.
.
.
.
.
.
.
.
.

Capital Stock (authorized, issued 1,950


and outstanding) 50,000 To Land Buildings.
&

1,950
.
.

of .
.
.
.
.

.
.
.
.
.
.

Reserve for depreciation plant


and machinery 9,500
Accrued Mtge. Interest....
.

.
.
.
.
.
.
.
.

52
. of .
.
.
.
.
.

Reserve for depreciation build Mtge. Bond ..............


ings 25,000
1,950 Cash 9,948
.

.
.
.
.
.
.
.

.
.
.
.
.
.
.
.
.
.
.
.
.

Reserve for Contingencies.


.
.
.
.

.
.
.
.
.
.
.
.
.
.
.
.
.
.
.
.
.
.
.
.
.

15,200
To Plant Machinery
&
.

of ; .
.
.
.
.
.
.

Surplus 798 35,000


.
.
.
is ..
a ..
.

.
.
.
.
.
.
.
in..
.

There
.
.
.
.
.

balance the bank $12,500.


Realization Account....... 93,050
To Sundry Assets—
by

report rendered
A

the Secretary for Land Buildings..


&

the real
28,050
the company shows the result
of

Mdse. Inventory... 22,500


ization as follows: Notes Receivable... 10,500
The mortgagees bought the plant and ma Accts. Receivable... 16,800
chinery for $35,000, paying cash for the dif Contingent Fund... 15,200
ference between the amount the mort
of

gage and the accrued interest, and the pur


Sundry Liabilities
chase price. The land and buildings were
sold for $33,000. The inventory To Liquidation Acct.. 55,000
of

merchan Notes Payable 27,000


dise was disposed for $20,000. The Notes
of

.
.
.
.
.
.
.
.
.
.
.

Receivable were paid; the Accounts Receiv Accounts Payable 28,000


.
.
.
.
.
.
.
.

able realized $15,150, and the securities


of

the Contingent Fund realized $14,700. Reserve for Contingencies. 15,200


All Notes Payable and Accounts Payable To Surplus ........... 15,200

75
Cash


Mdse.
Notes
Accts.
º§Receivable.
Receivable.
Contingent Fund..
--

. . . . . .. . . .. . .. . . . . . . .

Inventory...


20,000
10,500
15,150
14,700
93,350
93,350
To Plant & Machinery

To
it.

Realization Account.......

For loss on foreclosure.


Realization Account.......
Cash . . . . . . . . . . . . .
Expenses of realization
and liquidation.
5,500

3,200
5,500

3,200

Surplus or Deficit......... 8,400


To Realization Acct..
*-
8,400
Liquidation Account...... 55,000
To Cash ............. 55,000 Capital .. .. .... . .. .... . ... 50,000
Accounts Payable...

-
$28,000 Surplus . . ... . .. .... ... .. . 7,598
Notes Payable...... To

-
27,000 Cash . . . . . . . . . . . . . 57,598

Balance Sheet of The Washington Company


as at May 15, 1912.
Assets. Liabilities.
Fixed— Capital Stock—
Land & Buildings..... $75,000 Preferred . . . . . . . . . . . . $250,000
Plant & Machinery... 200,000 Common . .. ... . .. . .. . 250,000 $500,000
Tools, Equip. & Fixt... 50,000 $325,000
First Mtge. 5% Bonds. . $150,000
Current— Less unissued ........ 50,000 100,000
Cash .. . .. .... .. .. .. .. $12,500
Accounts Receivable... 33,000
Inventories ........... 100,000 145,500

Good-will .............. 129,500

--
$600,000 $600,000

-
Realization and Liquidation Account of The American Trading Co.
Assets to be Realized. Liabilities to be Liquidated.

-
Land & Buildings.330,000 Mtge. Bonds .. . ... .. .. . $25,000
Less Depreciation. 1,950 $28,050 Accrued Int. Payable.... 52
.........

-
Notes Payable 27,000
Plant & Machin'y. $50,000 Accounts Payable. . . .. .. 28,000 $80,052
Less Depreciation. 9,500 40,500
Assets Realized—
Merchandise Inventory... 22,500 Plant & Machinery.... $35,000
Notes Receivable . . . . . . . 10,500 Land & Buildings. . . . . 33,000
Accounts Receivable.... 16,800 Mdse. Inventory...... 20,000
Contingent Fund secur.. 15,200 $133,550 Notes Receivable...... 10,500
Accounts Receivable... 15,150
Liabilities Liquidated— Contingent Fund Sur. 14,700 128,350
Mtge. Bonds . . . . . . . . . $25,000
Accrued Int. Payable.. 52 Assets Not Realized—
Notes Payable . . . . . . . . 27,000 Mdse. Inventory...... $2,500
Accounts Payable..... 28,000 80,052 Accounts Receivable. . 1,650
Contingent Fund...... 500
$213,602 Plant & Machinery.... 5,500
Supplementary Charges—
Expense of Realization....... 3,200 $208,402
Loss on Real. & Liq. . . .. . .. . .. . . 8,400

$216,802 $216,802

Cash on hand (distributed). . . . . $57,598 Capital . . .. . .. .. .. . .. .. . . . .. .. . . $50,000


Loss on Realization. . . . . . . . . . . . . 8,400 Surplus—
July 15, 1910. . . . . . . . . . . . $798
Additions through Real.
& Liq. . . . . . . . . . . . . . . . 15,200 15,998

$65,998 $65,998
Municipal Civil Service Commission, N. Y. resented by the coupons as received from
time to time and the premium paid on the
ACCOUNTANT, FIFTH GRADE. bonds when the bonds mature.
9. The balance sheet of the National
Date: February 2, 1916.
Manufacturing Company showed a deficit
TECHNICAL–Weight 6. of $10,000, as of December 31, 1913. During
the year of 1914, the company operates at
PART I–GENERAL PAPER. a net profit of $25,000 and the directors
declare a dividend of $20,000. Assuming
1. Define: Funded Debt; Betterments;
that the cash condition of the company as
Intangible Capital; Nominal Accounts; of December 31, 1914, justified the declara
Stock Dividends. tion of the dividend, give reasons for and
2. The Interstate Phosphate Company against the action of the directors and jour
has a controlling account in the general nal entries necessary to reflect the matter
ledger for accounts receivable called “Cus on the books.
tomers' Ledger,” the detail of which is kept 10. A loans B, a retailer, certain sums of
in a subsidiary customers' ledger. Describe money on B's note, secured by B's assign
the mechanism of such an account, showing ment of his accounts receivable to A. B is
clearly how the general ledger account ac supposed to deposit all collections of these
tually controls the customers' account. assigned accounts as soon as received in
3. (a) What is a warranty? Does the his account and to immediately send A his
breach of a warranty discharge the entire check therefor. A suspects that B is tem
contract? - porarily withholding some of the collections
(b) Name three kinds of negotiable in of the assigned accounts. Assuming that
struments. Which, if any of the ones named,
if dishonored, must be protested and due
A has access to B's books, and engaged you
to investigate the matter, what plan would
notice thereof given, in order to hold the 'you follow to prove or disprove A's sus
indorser? picions?
4. The books of an advertising corpora
tion chartered under the laws of the State
of New York show that 1,000 shares of the PART II—PRACTICAL ACCOUNTING.
authorized capital stock of the corporation
Assume that your firm has designated you
were originally sold at par, $100 per share,
with an allowance of $5 per share to the to make an audit for the month of August,
1915, of the accounts of the La Presse Pub
purchaser as a cash discount. Make what lishing Company, publishers of a French
ever criticisms you think necessary of this
transaction, from the viewpoint of the cor daily newspaper. The work has previously
poration, of the holder of this stock, and been handled from a branch office and they
intimate that it may have been done in a
of a prospective purchaser of stock.
5. In setting up a reserve for deprecia lax manner. No previous reports or work
tion, what elements are generally recognized ing papers are obtainable, but they suggest
as contributory to depreciation? How can the following form for your Income State
ment.
the account be arranged so that the total
of charges for repairs and for depreciation Make an Analysis of Operating Expenses,
will bear a constant ratio to production using the following Group Headings:
from month to month? Cost of Manufacture—
6. Two manufacturing companies, A and Material
B, are engaged in the manufacture of the Labor
same commodity. Company “A” leases a
Power and Plant Expense
factory and pays rent; Company “B” bor Editorial Expense
rows money and builds a factory. Discuss Delivery Expense
methods of treating “A’s” rent and “B's" Cost of Securing Business
interest in a statement of income and profit General and Office Expense
and loss of each company?
7. On the New York-Buffalo line of the Use totals in Income Statement.
telephone company 1,000 loading coils, cost You complete your examination and find
ing $3 each, were installed during 1913 at that the books are correct and in good con
a cost of $1 each for installation. In 1914, dition, except as follows:
because of the unexpected extension of the On August 1st, 1915, the company pur
New York-Buffalo line to Chicago, the orig chased 100 B. A. & G. Company General
inal loading coils had to be replaced by new Mortgage three per cent (3%) bonds, $1,000
coils of greater capacity, costing $5 each, each, at par, with accumulated interest for
and $1 to install. The original coils were fifteen (15) days.
returned to stock at cost, being as good On August 1st the company issued a de
as new, and the cost of taking them out was mand note for $50,000 to the estate of P.
$0.50 each. Show the necessary entries to Bouton—note to bear interest at six per
properly reflect the transaction. cent (6%).
8. A block of two-year 4 per cent bonds, The above items were omitted from the
par $20,000, bought at $20,385.44, interest books by the bookkeeper, pending a con
payable semi-annually, yields 3 per cent. sultation with the accountant.
Frame journal entries, giving accurate fig You further note that a bill covering
ures to treat properly both the interest rep freight on rolls of paper returned by the
77
publishing company has been charged to The following facts, outside of the books,
Delivery and Cartage out—$33.46. should be used in preparing the financial
An item of $10 was charged to Stuffing statements at August 31st, 1915:
Labor. You ascertain that it covered the
company's portion of one man's salary em:
off Accrued Salaries and Wages—
ployed by five newspapers to throw
bundles from a suburban car. Office Salaries . . . . . . . . . . . . . . $36.00
-
You also discover that the Authorized Collection Salaries . . . . . . . . . 12.00
Capital Stock is really $100,000, and that Labor—Press and Machinists 16.00
Delivery Salaries . . . . . . . . . . . 32.00
haſ of it had been donated back to the Editorial Salaries . . . . . . . . . . . 60.00
company at the time of organization. This Labor–Stereotyping ........ 24.00
fact is not stated on the books. .
In addition to the above, you are informed Labor—Compositors ........ 495.00
that the directors have declared a fifty per
cent (50%) stock dividend, payable as at - $675.00
August 31st, 1915, and for that purpose have Unpaid Subscriptions—estimated. 3,346.00
Subscriptions Paid in Advance—
applied for and received authorization from
the State to increase the capitalization to estimated . .. .. . .. . .. .. . .. . . 342.61
$150,000. You are requested to furnish en
tries to place these facts on the books, The changes in estimated subscriptions
showing the payment of the dividend. You should be adjusted with Newspaper Sales.
ascertain that the stock has been issued as
of August 31st, 1915. From the facts stated, and the accom
panying trial balance, prepare a Balance
The following facts, outside of the books,
were used in preparing the financial state Sheet as at August 31st, 1915, and a State
ments at July 31st, 1915: ment of Income and Profit and Loss for
Accrued Salaries and Wages—La the month of August, 1915, for the La Presse
bor, Compositors ... . .. .... . $285.00 Publishing Company.
Unpaid Subscriptions—estimated. Give adjusting Jour
3,496.00
Subscriptions Paid in Advance— nal Entries affecting the increase in Cap
estimated . .. .. . .. .. . .. . .. . . 342.61 ital Stock, and the payment of the Dividend
Balance of Surplus Account. . . . . . 168,137.90 declared.

La Presse Publishing Company


Trial Balances

At July 31st and August 31st, 1915.

July 31st, 1915 August 31st, 1915


Cash in Banks. . . . . . . . . . . . . . . . . . . . . . . . . . . . $162,555.61 ... . . .. . $164,395.22 ... . . ...
Petty Cash . . . . . . . . . . . . . . . . . . . . . . . . . . . . . . . 200.00 .. . . .. .. 200.00 .. . . . . ..
Accounts Receivable . . . . . . . . . . . . . . . . . . . . . . 47,050.32 .. . .. . .. 44,155.35 .. . .. ...
Bills Receivable . . . . . . . . . . . . . . . . . . . . . . . . . . 202.72 .. . . .. . . 202.72 .. . ... ..
Inventory . . . . . . . . . . . . . . . . . . . . . . . . . . . . . . . . 2,227.85 ... ... . . 2,034.34 ... ... ..
Reserve for Trade Rates.................. 666.19 . . . .. . .. 192.12 . . .. . .. .
Machinery . . . . . . . . . . . . . . . . . . . . . . . . . . . . . . . 88,500.48 . . . .. . .. 88,500.48 .. . ... ..
Furniture and Fixtures. . . . . . . . . . . . . . . . . , 12,042.07 12,042.07
...

.
.
.
.
.
.
.
.

.
.
.
.
.
.
.
.

Library 484.35 484.35


.
.
.
.
.
.
.
.

.
.
.
.
.
.
.
.
.
.
.
.
.
.
.
.
.
.
.
.
.
.
.
.
.
.
.
.
.
.
.
.
.
.
.
.
.
.
.
.
.
.

Type and Metal. 1,884.49 1,871.70


Electros.........................
.
.
.
.
.
.
.
.
.
.
.
.
.
.

.
.
.
.
.
.
.
.

.
.
.
.
.
.
.
.
.
.
.
.
.
.
.
.
.
.
.
.

Cuts and 3,573.70 3,545.33


.
.
.
.
.
.
.
.

.
.
.
.
.
.
.
.

Automobile 4,436.56 4,441.56


.
.
.
.
.
.
.
.
.
.
.
.
.
.
.
.
.
.
.
.
.
.
.
.
.
.
.
.
.
.

.
.
.
.
.
.
.
.

.
.
.
.
.
.
.
.

Insurance Unexpired 1,252.16 1,192.16


.
.
.
.
.
.
.
.
.
.
.
.
.
.
.
.
.
.
.
.
.

.
.
.
.
.
.
.
.

.
.
.
.
.
.
.
.

Accounts Payable—Commission and Trade


Accounts $3,640.13 ........ $4,475.38
.
.
.
.
.
.
.
.
.
.
.
.
.
.
.
.
.
.
.
.
.
.
.
.
.
.
.
.
.

.
.
.
.
.
.
.
.

Estate—P. Bouton 57,564.95 57,045.27


.
.
.
.
.
.
.
.
.
.
.
.
.
.
.
.
.
.
.
.
.
.
.

.
.
.
.
.
.
.
.

.
.
.
.
.
.
.
.

Accounts Payable—Creditors. 10,207.91 7,458.20


........
.
.
.
.
.
.
.
.
.
.
.
.
.

.
.
.
.
.
.
.
.

.
.
.
.
.
.
.
.

Reserve for Bad Debts. 4,403.81 4,291.08


.
.
.
.
.
.
.
.
.
.
.
.
.
.
.
.
.
.
.

.
.
.
.
.
.
.
.

Reserve for Income Tax. 100.00 120.00


.
.
.
.
.
.
.
.
.
.
.
.
.
.
.
.
.

.
.
.
.
.
.
.
.

.
.
.
.
.
.
.
.

Reserve for Depreciation—Plant and Fix


tures 33,890.19 34,190.19
.
.
.
.
.
.
.
.
.
.
.
.
.
.
.
.
.
.
.
.
.
.
.
.
.
.
.
.
.
.
.
.
.

.
.
.
.
.
.
.
.

.
.
.
.
.
.
.
.

Capital Stock 50,000.00 50,000.00


.
.
.
.
.
.
.
.
.
.
.
.
.
.

.
.
.
.
.
.
.
.
.
.
.
.
.
.
.
.
.
.
.
.
.
.
.

.
.
.
.
.
.
.
.

Surplus 165,447.14 165,447.14


.
.
.
.
.
.
.
.
.
.
.
.
.
.
.
.
.
.
.
.
.
.
.
.
.
.
.
.
.
.
.
.
.
.

.
.
.
.
.
.
.
.

.
.
.
.
.
.
.
.

Newspaper Sales 9,622.31 16,754.63


.
.
.
.
.
.
.
.
.
.
.
.
.
.
.
.
.
.
.
.
.
.
.
.
.

.
.
.
.
.
.
.
.

.
.
.
.
.
.
.
.

Advertising Income—Display 23,937.34 32,786.04


.
.
.
.
.
.
.
.

Advertising—Classified ...
.
.
.
.
.
.
.
.
.
.
.
.
.

.
.
.
.
.
.
.
.

1,553.67 2,382.43
.
.

.
.
.
.
.
.
.
.
.
.
.
.
.
.
.
.
.
.
.
.
.
.

.
.
.
.
.
.
.
.

Interest Earned 1,159.09 1,229.23


.
.
.
.
.
.
.
.
.
.
.
.

Profit on Premiums Sold.................


.
.
.
.
.
.
.
.
.
.
.
.
.
.

.
.
.
.
.
.
.
.

.
.
.
.
.
.
.
.

256.43 450.54
Bad Debts—Provision for.................
.
.
.
.
.
.
.
.

.
.
.
.
.
.
.
.

273.00 378.00
.
.
.
.
.
.
.
.

.
.
.
.
.
.
.
.

Depreciation on Plant—Written Off....... 600.00 900.00


.
.
.
.
.
.
.
.

.
.
.
.
.
.
.
.
Cash Discounts - - - - - - - - - - - - - - - - - - - - - - - - - - 267.07 . .. .. . .. 375.12 . ... . ...
Paper—Consumed 8,370.35 . . . . . . .. 12,021.65 .. . .. . . .
Ink—Consumed 276.66 .. . . . . . . 407.16 .. . .. . . .
Matrices—Consumed - - - - - - - - - - - - - - - - - - - - - 767.70 . . . . . . .. 1,066.60 .. . . . . . .
Type and Metal Written Off 200.00 . . . . . . .. 300.00 .. . .. . . .
Gen. Plant Supplies—Consumed 357.40 .. . . . . .. 445.62 . . . .. . . .
Labor—Compositors 5,755.46 . . . .. . . . 8,364.05 . . . .. . . .
Labor—Stereotypers 632.90 . . . . . . . . 905.30 . . . .. . . .
442.00 . . . .. . . . 642.00 . . . .. . . .
Labor—Stuffing 67.50 . . . . . . . . 97.50 . . . .. . . .
Rent two-thirds Power and Plant Expense.
One-third Off. Expense 508.34 .. . ... . . 762.51 . . .. . .. .
Light, Heat and Power. three-quarter
Power and Plant Expense. One-quar
ter Office Expense 867.25 . . . . . . . . 1,181.87 .. . . . . . .
Maintenance and Repairs 347.10 . . . . . . . . 445.43 .. . . . . . .
Insurance—Written Off ........... - - - - - - - 240.00 . . . . . . . . 360.00 .. . . . . . .
Cuts and Electros—Written Off 140.00 . . . . . . . . 210.00 .. . . . . . .
News Service - - - - - - - - - - - - - - - - - - - - - - - - - - - - - 1,211.01 . . . .. . .. 1,752.34 .. . . . . . .
Editors and Reporters—Salaries - - - - - - - - - - - 1,624.50 . . . .. . .. 2,331.00 .. . . . . . .
Editors and Reporters—Expenses 719.68 . . . .. . .. 1,250.03 .. . . . . . .
Telegrams . . . . . . . . . . . . . . . . . . . . . . . . . . . . . . . . 32.12 . . . .. . .. 84.95 .. . . . . ..
Telephone—One-half Editorial
One-half Office Expense 179.31 . . . . . . . . 259.08 .. . . . . . .
Delivery and Cartage—Out 903.48 . . . . . . . . 1,277.34 :. . . . . . .
Shipping and Delivery—Salaries 886.00 . . . . . . . . 1,294.00 .. . . . . . .
Shipping Supplies Consumed 179.50 . . . . . . . . 305.09 .. . . . . . .
Solicitors' Commissions 2,669.00 .. . . .. . . 3,904.53 .. . . . . . .
Solicitors' Expense 120.82 . . .. . .. . 183.20 .. . . . . . .
Advertising Agents and Special Commis
sions 1,508.93 . . . .. . .. 2,279.18 .. . .. . . .
Advertising—General -- -- ---- -- - - - - - -- -- - - 116.46 .. . ... .. 167.67 .. . .. . . .
Officers' Salaries - - - - - - - - - - - - -- - -- - - - - -- -- 4,000.00 .. . .. . . . 6,000.00 . . ... . ..
Office Salaries - - - - - - - - - - - - - - - -- ------- ---- 1,076.29 .. . . . . .. 1,573.29 .. . .. . . .
General and Office Expense - - -- ---- ------- 132.60 .. . . . . . . 226.33 .. .. . .. .
Collection Salaries - - - - - - - - - - -- ------- ---- 338.00 .. . .. . . . 474.12 .. . .. . . .
Collection Expenses - - - - - - - - - -- ---- -- ----- 97.68 .. . . . . .. 163.32 .. .. . .. .
Legal and Auditing Expenses 319.66 .. . .. . .. 320.66 .. . .. . . .
Taxes Paid - - - - - - - - - - - - - - - - - - - ------------ 18.20 . . . .. . .. 26.21 ... .. . ..
460.50 .. . .. . .. 637.58 .. ... .. .

$361,782.97 $361,782.97 $376,630.13 $376,630.13

79
Account ANT, NEw York state service.
Specimen Examination Questions for Accountant and
Junior Accountant, Public Service Commission,
and Expert Accountant, Dept. of Efficiency
and Economy.

State of New York—The Civil Service Operating revenues:


Commission. Freight ... . . . . . . . . . . $3,488,041
Passenger . . . . . . . . . . . 1,291,317
JUNIOR ACCOUNTANT, Mail, Express, and
Miscellaneous ... .. 452,627
(Salary, $1,201 to $1,500.) $5,231,985

Public Service Commission, Second District.


Operating expenses and taxes:
Maintenance of wa
Date: November 29, 1913. and structures. . . . . . $627,364
Maintenance of equip
N. B.-Candidates are advised to take ment . . . . . . . . . . . . . . 1,177,997
about two hours on this part of the exam Traffic expenses. . . . . 154,663
ination. Discuss fully one and only one of Transportation exp... 1,736,409
the following three topics: General expenses..... 126,847
1. (a) The Purposes for which valua
tions of Public Service Corporations are $3,823,280
made. (b) The Principles and Methods in Taxes .. ... ... .. . ... . 176,041
3,999,321
volved in an efficient and economical de
termination of accurate information rela
tive to one of the above purposes, with Operating income . . . . . . . . . . . . . . $1,232,664
reference to any one of the Public Service State percentage of operating expenses
Corporations under the jurisdiction of the
to operating revenues; state percentage of
Public Service Commission, Second Dis operating income to operating revenues;
trict. state statistics per mile of road operated
2. (a) Cost of Reproduction, (b) Actual for the above three items.
cost, and (c) Cost less depreciation, with 4. A, B and C enter into a partnership.
reference to the determination of the pres A contributes $14,500 in cash and $2,500 of
ent value of the physical property of a typi merchandise. B contributes $2,500 in cash
cal public service corporation, outlining and $6,000 of accounts receivable. C con
(d) the chief advantages in your opinion tributes $4,250 in cash. Show how these
of the most practicable and equitable of transactions should be recorded on the vari
the above methods. ous books of account of the firm.
5. Referring to question 4—the net prof
3. (a) Distinguish between physical and its at the end of the first year's business
functional depreciation as to (1) causes, (2) is $14,350 which is to be credited to each
three methods of determination, and (3) of the partners in proportion to the amount
methods of handling on the books of a pub invested by each partner. Give the journal
lic service corporation. (b) Fluctuation, as entry making this distribution of profits.
distinguished from the above, together with
6. How should taxes accrued, prepaid
(1) its causes, and (2) its effects on the interest on loans payable, and unexpired
valuation of the company's fixed assets. insurance be considered in the accounts at
the close of the fiscal period? How would
ACCOUNTING PRINCIPLES. each of these amounts be ascertained?
7. How should a bank balance be veri
1. Give short definitions of the follow fied, and state what precautions should be
ing accounting terms: (a) Capital, (b) cap observed in this connection.
ital expenditures, (c) fixed assets, (d) cur 8. What are the differences in the meth
rent assets, (e) current liabilities, (f) in ods of using a voucher record and an ac
come account, (g) profit and loss account. counts payable ledger, and the advantages
and disadvantages of each?
2. State the various books of account in
9. What are the advantages of having
use in a commercial business and explain controlling accounts on the general ledger
the purposes for which they are each used
and how are they conducted?
and the relation to each other as part of 10. Give a balance sheet and a profit and
the system of accounting. loss account with estimated amounts of
3. The A. G. S. R. R. operates 309 miles some business with which you are familiar
of road. The income account for the past and show therein at least ten assets and
year shows the following: about eight liabilities.

- 80
of
State of New York—The Civil Service (c) Describe two kinds credit.

in
Commission. (d) Mention three different cost units
public utilities.

of
the operation
illustration pro rate.

an

of
ACCOUNTANT, (e) Show

a
(a) What are the principles

of
11. double
entry bookkeeping?
Public Service Commission.

of
parts

or
(b) What accounts accounts
the make-up

of
(Salary, may be eliminated an

in
$1,801 to $2,400.)

of
inter-company balance sheet for group

a
Date: April 8, 1916. commercial businesses?
the subjoined statement point

In
ECONOMICS, STATISTICS AND COR 12. (a)
out any errors and show any correspond
PORATION FINANCi. ence between particular items

of
debits and
credits:
(One of the following 9 questions may be
Dr. 161.33 Cn 389.42
omitted.) 541.89.
225.01
Is public utility income like public 604.00
revenue, and how do both differ from pri
1.
3.08
vate income? 62.11
of

Describe some the functions the


of
2.
...

1,931.32
New York State Public Service Commission 1,931.30
either for the First District or for the Sec
ond District. $31,211.59 for
(b) Compute the interest on
What corporation statistics might be
113 days 2% per cent.
3.

at
prepared manage
in
to

show wastefulness
street railroad collects 437,200 five
A
ment? 13.
its operations.

of
cent fares the first year
in
What are sinking funds, their purposes three succeeding years there are accumu
4.

In

accounting form? per cent, 17% per cent

10
and lated increases of
Mention two kinds of dividends with 3/11 per cent respectively. Income
27
5.

and
comments upon each. charges every kind are the rate of
of

at
earnings.
of

per cent no dividends are

If
72

What an average, and how could


it
is
6.

paid, how much sinking fund can be


of

be abused? Illustrate.
a

each year retire per cent


to
in

set aside

6
the fourth
of
notes maturing
at

the end
PRINCIPLES AND METHODS OF

of
year? What will be the amount the
ACCOUNTING. notes so retired? Describe mode of in
a

dependent proof both calculations.


of

“Important
15
of

State some the


7.

of

Changes During the Year” required an 14. Set down the squares the numbers
in

inclusive; strike out the mul


60

the Public Service Com


5;to

nual reports from


to

foot the remaining figures; show


of

mission. tiples
four decimal
to

their average carried out


Describe briefly the Commission's plan places.
8.

classifying balance sheet and


of

and scope
of
as

income account items, applied


to

one
the following:
State of New York—The Civil Service
(a) Steam Railroads;
(b) Telephone and Telegraph Corpora Commission.
tions;
ACCOUNTANT,
(c) Street and Electric Railways;
(d) Electric Corporations; Public Service Commissions.
(e) Gas Corporations.
To what different accounts would you (Salary,
to

$1,801 $2,400.)
9.

charge the electrical corporation items


Date: January 11, 1913.
which follow:
Cash fees paid promoters; ECONOMICS, STATISTICS AND COR
to

clerks engaged on construc


of

Salaries PORATION FINANCE.


tion accounts;
Salaries, fees and expenses surgeons
of

the leading
of
at

(a) Name least three


1.

and doctors; causes which have led to concentration of


Amounts owing the corporation upon
to

capital. (b) Give and discuss the economic


accounts with solvent concerns; capital.
of
of

advantages concentration
Rents paid the enjoyment
of

advance
in

(a) Differentiate between “trusts” and


2.

the term. the pur


of

“holding corporations.” What


is

(a) Define: Lessor, tort, cross foot pose (b) What meant by the
of

each?
is

10.
ing, residual, termini, intercorporate hold term “stock watering"? full.
in

Discuss
ings, revenue car miles, intra state, original Name some of the remedies which
3.

entry, condensed comparative balance sheet. have been suggested for solving the trust
(b) Distinguish between “road” and “sec problem. Which do you consider the best?
ond track." Why?

81
4. Discuss the value of (a) comparative (funded debt), $380,750; Loans payable, $53,
statistics, (b) the application of statistical 000; Other reserves, $13,104; Prepayments,
methods to public service corporations. $939; Production expense, $26,780; Sales

of
5. Name and describe at least three meth electric current, $186,481; Taxes, $13,700;
ods of setting forth statistical data. What Uncollected bills (bad), $1,673; Utilization
are the advantages and also the disadvan expense, $8,984. Prepare balance sheet and
tages of each method named by you? 1ncome account for the year.
6. Name and discuss some of the potent The United Iron Company acquired by

3.
agencies for industrial peace. cash purchase on January 1911, 90 per

1,
a
Discuss the problem of the inventory the outstanding stock the North

of

of of
7. cent
in connection with public service corpora ern Iron Company

at
cost $1,350,000.

a
tion accounting. In connection there with, The balance sheet of the Northern Iron
discuss the valuation of a franchise. How Company that date shows: Capital stock

at
would it be treated in the books of a pub outstanding, $1,000,000; Profit and loss sur
lic service corporation? plus, $240,000. On December 31, 1911,

it
Discuss the economc question of de shows no change the capital stock out

in
8.
preciation in its relation to corporation standing; the profit and loss surplus had in
How would you treat this subject The North

at

to
finance. creased that date $320,000.
in the books of a railway corporation? erri Iron Company declares on December 31,
Why? What difficulties are likely to arise? 1911, per

of
cash dividend 20 cent on the

a
Discuss also the legal aspects as determined outstanding stock, payable on January 15,
by various court decisions in the United 1912. Make the necessary ledger entries
States. Name at least three methods of de both companies regard

in
of
the accounts

in
termining the amount of depreciation to be to the above dividend as on December 31,
charged. Which do you consider the best? 1911, and January 15, 1912, respectively.
Why? The car barn of the Suburban Street

4.
9. How should each of the following be Railway Company was destroyed by fire.
treated in the books of a corporation: (a) The insurance collected amounted $150,

to
Unsubscribed and unissued stock; (b) un 000, and represented recovery

on
$50,000
paid subscriptions to stock; (c) stock re building, and $100,000 recovery on cars. The
it;

acquired by the corporation issuing (d) the building, valued

at
$5,000,

of
foundations
stock received by person full paid with were not damaged and are
as

to
be used

in
a

it;

out his having actually paid for (e) stock the rebuilding. The salvage on the cars
bought less than par? Give full reasons The expense
at

of
amounted to
$5,500. clear
each case for the course taken by you. ing away the debris amounted
in

to
$2,500.
(a) Define describe “sinking fund” From the ledger accounts appears that
or

10.

it
its usual meaning and give its purpose. the original cost the building was $70,000
in

of
(b) What do you think the practice of and the equipment $135,000.
of

The reserve
American railways charging discount on
of

for accrued amortization capital

at
of
the
bonds to the construction account? Dis date of the fire was: account of this build
cuss full, giving reasons for your conclu ing, $10,000; account equipment, $12,
in

of

this
sions. Show the appropriate ledger ac
in

000.
counts how this fire loss and insurance re
PRINCIPLES AND METHODS OF AC covery should Explain such
be

of
COUNTING, WITH ESPECIAL REF recorded.
ERENCE TO THE SYSTEMS OF AC the ledger accounts

as
the entries
in

are
considered desirable.
COUNTS PRESCRIBED BY THE
PUBLIC SERVICE COMMISSIONS. The City Street Railway Company
5.

in
order to obtain funds for the construction
The balances the ledger accounts
of

and equipment its railroad, arranges for


of

1-2.
the Union Electric Company on Decem
of

$2,000,000 first mortgage 20-year


of

the sale
ber 31, 1911, which includes the operating per cent bonds 85. The legal and other
at
5

accounts for the year ended that date, were expenses incidental
to

the issue amounted


cap
as

follows: Accrued amortization


of

$35,500 and were paid by the company.


to

ital, $119,177; Accrued liabilities, $29,420; The proceeds


of

the bonds were obtained


Bound investments (permanent), $111,047;
as

from the bankers the work progressed.


Capital stock, $501,000; Cash, $40,158; Cas the bonds for the year, less
on

The interest
ualty and insurance reserve, $14,848; Com the interest allowed by the bankers on the
mercial expense, $12,123; Construction work deferred installments, amounted
of to

$80,000.
progress, $22,368; Corporate surplus, De
in

The construction and equipment the road


cember 31, 1910, $295,657; Current liabili was completed one year, and then the
in

ties, $12,208; Consumers' accounts, $37,614; company began operating its railroad. Show
Distribution and repairs, $28,935; Dividends the appropriate ledger accounts the en
in

per cent on capital stock (declared and tries referring


to

the transactions regard


in
8

paid), $40,080; Dividends received, $1,008; the bonds, discount and ex


of

to the sale
Due from associated companies, $13,436; pense issue, and the interest paid thereon,
of

Fixed capital, $1,178,433; Free investments and the status of the accounts the close
at

(marketable), $16,094; General and miscel Explain such


the first year's operation.
of of

laneous expenses, $26,678; Interest revenues, the ledger accounts


as

the entries
in

are
$8,779; Interest on funded debt, $19,038; considered desirable.
Materials and supplies (inventory), $23,273; The Western Street Railway Company
6.

Miscellaneous rents received, $1,748; Miscel has $5,000,000 20-year per cent bonds
5

laneous revenue, $4,173; Mortgage bonds which mature July The company
1,

1911.

82
ºn account of the general financial condi State of New York—The Civil Service
tions do not consider it advantageous to Commission.
sell any long term obligations. They there
fore, in order to provide for their maturing EXPERT ACCOUNTANT,
bonds, arrange with their bankers for the
Department Efficiency and Economy.

of
sale of $5,102,041 one-year 6 per cent notes
at 98. During the year subsequent to July 25, 1913.
Date: October
1, 1911, the railway company arrange for
the sale of a new issue of $5,000,000 20 ACCOUNTING.
year 5 per cent mortgage bonds at 96, dated
You and two assistants are required

1.
July 1, 1912. The legal and other expenses
make an audit of the accounts for

to
The

a
incidental to this issue are $50,000.
fiscal year large institution.

of
proceeds of the bond issue in addition to How

a
cash supplied by the company are used to would you conduct this examination? State
take up the one-year notes at their matur the various steps you would take, and show
program

of
The company charges at June 30, 1912, the work which would be done

a
ºv.
by you and each your assistants.

of
$16,764, being a proportion of the combined
discount and expenses on the one-year notes comparing

In
the accounts for the

2.
same fiscal year simi

of
and that to be incurred on the 20-year bonds, two institutions

is of
as an income deduction under “amortiza lar character, found that the costs of

it
tion of debt discount and expense.” operations the two institutions vary con

of
State
reasons for approving or disapproving the siderably. How would this be ascertained,
charge as above made. and under what circumstances would there
justification for this condition?
be

7. The Southern Gas Company erected a


a

new gas holder at the following cost: Land, large manufacturing concern has
3.

A
$35,000; excavations and foundations, $10, its head office New York and factories

in
Wilſ); plumbing equipment, $10,000; gas hold located several other cities. Describe
of in

of a
er,

$45,000; total, $100,000. estimated system organization for the conduct


It
is

that the pumping equipment will be


at

at
use
in

the business the head office and the


ten years and that there will be salvage factories.
a

$500 when retired from service; that


of

is
it

Prepare balance sheet with estimated


4.

is a a

the gas holder will be use twenty years


a in

amounts, for steam railroad company

in
and that there will be salvage $2,000
of

usually issued its stock

to
the form
it

dismantled. Prepare the first


is

when
it

holders.
journal entry or entries providing for ac
In connection with work which you
5.

crued amortization capital.


of

have done revising the accounting meth


in

The Eastern Gas Company order


in
8.

large business, state the methods


of

improve its service replaces its street ods


a
to

which were use, and the various changes


in

mains with mains of one-third additional


you made, and wherein each your changes
of

capacity. The books show that the orig improvements.


resulted
in

imal cost of installation of the old mains


was $100,000. The accrued mortization of How would you make the verifica
6.
a

$50,000; salvage recovered, an inventory finished merchan


of

of

capital thereon tion


is

The cost installing the new dise, materials and supplies? State de
in

$10,500.
of

tail the various steps


to

$175,000. How should this matter be taken.


is

mains
considered by the company?
be

corporation issues its funded debt


a 7.

Under what conditions would discount, the proceeds


9.

be
at
it

of

which are
consider expenditures for the fol used for construction and equipment ad
to

of

proper
lºwing items fixed capital: Damages
to
as

ditional plants. How should the discount


persons and property, insurance, interest on be considered the accounts?
in

debt, salaries officials, taxes.


of

"Inded negotiable and


In

of

the examination
8.

10. One public service corporation dis non-negotiable securities, what precautions
its 20-year, mortgage bonds
at
of

Mºses should be observed?


a

iscount, another public service corporation


Who are the principal financial offi
9.

its 20-year mortgage bonds


at

disposes
of

cials of the State? Describe the functions


premium. the corporations
of

Each
a

of each one.
ther charges credits, the corporate
or

in

essay
an
In

of

surplus account during the first year the 10. about three hundred
bonds are outstanding, words, state the functions the Depart
of

the entire discount


Efficiency and Economy, and the
of
on

premium realized ment


or

illowed the sale.


State reasons for approving disapproving improvements which should result there
or
the

charges from.
or

above credits.

S3
IDIGEST OF UNIFORMI ACCOUNTING SYSTEMIS PRIE
SCRIBED BY THE PUBLIC SERVICE COMMIS
SION, FIRST DISTRICT, NEW YORK.

capital, but shall be debited to an account


(NOTE:—As the fundamental principles called “Unamortized Debt Discount & Ex
governing the accounts prescribed by the
Public Service Commissions in the various pense.”
States, as well as the Interstate Commerce To this account should also be charged
Commission, are practically the same, this all expense connected with the issue and
digest should prove of aid to students all sale of evidences of debt, such as fees for
The definitions drafting, mortgages and trust deeds, fees
over the United States.
here given apply alike to water, gas, elec and taxes on mortgage having more than
tric, telephone, traction and railroad com one year's life. Every fiscal period, a pro
panies in their general application.) portion of such discount and expense shall
be credited to “Una mortized Debt Discount
PURPOSES OF UNIFORM ACCOUNTS. & Expense" and charged to “Amortization
Debt Discount & Expense.”
1. To establish uniformity between all
corporations of the same class, so as to Journal Entries.
afford a ready comparison with a view Dr. Cr.
toward ascertaining efficiencies. Unamortized Debt Discount
2. To establish systems of account which $5,300
& Expense . . . . . . . . . . . . . . .
will clearly and accurately show the spe Discount . . . . . . . . . . $5,000
cific sources of income and the purpose of Engraving ........ 100
Commission ........
every expenditure. 200
3. To state the fundamental principles Cash . . . . . . . . . . . . . . . . . . . . . . . 44,700
according to which accounts should be kept
To Bonds (10 years).......... $50,000
so as to prevent the charging of items to
wrong accounts. At End of First Period.
(a) One danger is that not a sufficient Amortization of Debt Dis
up
amount will be laid aside to keep plant count & Exp. (10%) . . $530*
to standard. To Unamortized Debt Discount
(b) An undue amount will be taken out
and spent on extensions.
* Exp. . . . . . . . . . . . . . . . . . . . . . $530

The primary purpose of the system is to *Straight line method used here; sinking
insure the integrity of “capital” (assets) fund method is preferable.
and the correctness of the charges to “cost
of operation.” Capitalization of Franchises.
An allowance must be made periodically Only the amount paid to a government
for depreciation of assets, based on esti Price
mated life. should be charged to this account.
paid for franchise is a result of bargain
Problem: between government and corporation. Taxes
Original Cost . . . . . . . . . . . . . . . . . . . . $15,000 should not be charged to “Franchises.”
They are obligatory.
Bettered by improvement costing. . 3,000
pe
Estimated cost of replacement..... 16,000 Franchise value should be written off
riodically (amortized).
ANSWER:
Journal Entry. Effect of Requirements.
Dr. Cr. Every charge to capital shall represent
Betterment (Capital)........ $1,000 actual expenditure of full amount of such
Repairs to Asset (Expense). . . 2,000 charge. Every entry represents actual cash
To Cash . . . . . . . . .............. $3,000
value. Legitimate organization expenses
can be included in capital account, but this
is no inflation.
AMORTIZATION SCHEME. Thereby
Journal Entry. Required to Express Depre Investor is given guarantee against im
ciation on Books. pairment.
Operating Expense. . . . . . . . . . $1,000 Capital will be attracted at lower rate to
To Accrued Amortization of corporation.
Capital . . . . . . . . . . . . . . . . . . . $1,000
way Appreciation.
Landed capital is treated in the same
in a separate account.
Appreciation is not an offset to deprecia
Discounts and commissions upon securi tion. No appreciation shall appear in the
ties and commercial paper (issued in
pay accounts until it is an actuality proven by
ment for stock) shall not be charged to sale of this particular asset.

84
BALANCE SHEET. CLASSIFICATION OF FIXED AND
FLOATING CAPITAL ACCOUNTS.
Definition of Capital Accounts.
Fixed Capital Account.
Capital is the ren

all
property devoted (a) General

to
dering producing commodity

of or
Landed
of
services
within the purpose corporation. Non-Landed
Fixed Capital that which has expecta Tangible
is
more than one year (except Intangible
of

of

tion life
small tools). (b) Departmental
Floating Capital all other than fixed. Landed
is
Fixed Capital divided into two classes— Non-Landed
is

General and Departmental. Tangible


General that which available for the Intangible
is
is

uses of two or more classes operation. of Floating Capital Account.

of
Departmental Cash
to

limited one class


is

operation. Bills Receivable


Fixed Capital also divided into two Accounts Receivable
is

other classes—Landed and Non-Landed. Material and Supplies


Interest and Discounts Receivable
by

Landed Capital represented


or
land
ofis

Other Current Assets


land more than one year's
in

interests
life, minus improvements on the land.
Non-Landed Capital all other than INVESTMENTS.
is

landed. Investments are properties acquired not


for use present operations, but

as
Non-Landed Capital means
in
turn divided
in

is

a
obtaining exercising control over other
of

or
into tangible and intangible.
corporations, or for income

to
Intangible Capital comprises Organiza be derived
from them, or values, or for de

in
tion, Franchises, Patent Rights and all other rise

a
votion future operation
to

time when

at
intangible property within the definition
of

a
fixed non-landed capital. seems probable that they cannot be so
it

of
advantageously acquired

at
as
the time
Tangible Capital comprises structures, and
actual acquisition.
equipment having expectation
an

of

life
in

Bound Investments mean those which are


more than one year.
of

service (Because held subject


to

of
lien some character.
a

liability
or

theft, small tools


of

of

loss Free Investments are not subject any

to
are excepted from tangible capital and the 1611
S.
li

required
of

cost such tools when issued


is

an operating expense.) Bound Investments.


as
to

be treated
Mortgaged pledged.
or

Contractual fund.
Other Classification Capital. Miscellaneous.
of

Original Capital Free Investments.


that which invested
is
is

All reported
be

to

enterprise (assets). should Commission


at

of

the outset an
period and classified
to
at

as
of

end Funded
Additions are things not taking the place Debt, Stocks Controlling, Affiliated, Con
of

of things previously existing. trolled and Other Corporations; Advances


are improvements exist
of

Betterments Controlled, and Land, Improvements on


to

ing facilities.
Land and Other. Investment.
Renewals are extensions of terms years
in

Special Deposits are money the hands


in

land, franchises, etc. Fiscal Agents for payment


of
in

of

interest
Replacements are substitutions for ex and dividend coupons, etc.
latter; when Prepayment payment made before
hausted capital equal
is
it
is
to

value
in

greater value, the excess accrued.


betterment.
is
a

Withdrawals retirements.-Anything
or

Journal Entries.
withdrawn from service should be taken
When prepayment made:
is

off the books. Original entry should be


- r. Cr.
referred when making this closing entry.
to

PrepaidTaxes $1,000
.
.
.
.
.
.
.
.
.
.
.
.
.
.

Purchase capital assets from another


of

PrepaidInsurance 100
...............
.
.
.
.
.
.
.
.
.
.

corporation.—Where excess over estimated PrepaidRent 200


value paid for purchased capital, such Other Prepayments ........
is

400
excess should be charged “Other Intan To Cash
to

- $1,700
.
-
-
-
-
-
-
-
-
-
-
-
-
-
.
.
.
.
.
.

gible Street Railway Capital” account. Full When accrued:


report must Public Service Com
be

Taxes
to

made $1,000
.
.
.
.
.
.
.
.
.
.
.
.
.
.
.
.
.
.
.
.
.
.

mission any such purchase.


of

Insurance - 100
-
-
-
-
-
-
-
-
-
.
.
.
.
.
.
.
.

Repairs are not capital. They are the Rent 200


.
.
.
.
.
.
.
.
.
.
.
.
.
.
.
.
.
.

.
.
.
.
.

replacement part To Prep. Taxes...............


.
structure which $1,000
of

of
a

does not affect identity. This an operat Insurance 100


is

.
.
.
.
.
.
.
.
.
.
.

ing expense. Rent 200


.
.
.
.
.
.
.
.
.
.

.
.
.

85
Suspense is expenditure made the appro Second Preferred.
priate disposition of which has not yet been Common Stock. -
determined; also loss to be spread over a Debentures pay specified return at speci
period of time. fied time. -
Amortization is the gradual extinction of First Preferred have first claim on divi
tangible and intangible property. dends.
Depreciation applies ordinarily to tangible Second Preferred have second claim on
property only. dividends.
Reacquired Securities comprises securi Preferred may be:
ties acquired after being duly sold, charged Cumulative—Lien on future profits if divi
at face value. dend lapses.
Debt is all absolute obligations to pay Non-cumulative—Dividend lapses.
money at a given time. (Does not contain Participating—Unlimited dividends.
contingent liabilities.) Non-participating—Limited to specified
Debt is divided into funded and unfunded. dividends.
Funded Debt matures in over a year from Common Stock has dividend claims sub
d ate. sequent to all others.
Unfunded Debt matures in less than a Voting Power differs in different stocks.
year. Retirement—Corporation can retire stocks
if in contract.
Characteristics of Funded Debt.
Characteristics of Stocks.
1. Mortgage or other lien.
2. Rate of interest. 1. Intereston dividend rights.
Interest date.
3. 2. Voting rights.
Date of maturity.
4. 3. Conditions of retirements.
Only items agreeing in all characteristics Stocks entered into one account must
agree in all of these characteristics.
should be charged to one sub-account.
Debentures are unsecured promissory
notes or “plain bonds.”
INCOME ACCOUNT.
Reserves.
These are of two kinds: Permanent and When corporation begins operations, ac
Temporary. counts other than indicant are needed in
Permanent Reserves are those which must which to classify various changes under
kept intact during the life of the cor gone; that is, Income Account. Corporate
be
poration. Surplus ties together Balance Sheet and the
They are: Premium on Stocks; Other Income Account.
Permanent Reserves.
Divisions of Income Account.
Temporary Reserves are not intended to
Revenues.
remain intact during the life of the corpora
Revenue Deductions.
tion. They are:
1. Contractual. Income Deductions.
2. Non-Contractual. Appropriations.
Contractual Reserves are required by sink Revenues are all moneys received or re
ing fund provision in mortgage, etc. ceivable for services rendered or product
Non-Contractual Reserves are not required sold or property owned.
by contract. - Revenue is divided into two classes: Oper
Non-Contractual Reserves are divided into: ating and Non-Operating.
Required. Operating Revenue is that which “de ‘is
1.
Optional. rived from services rendered or merchandise
2.
sold or property used operation.
in

Required. Non-Operating Revenue that which


is

is

return on property or inter


as

derived
a

1. AccruedAmortization of Capital. property


ests which the corporation
in

2. Unamortized Premium on Debt. owns, but which the hands of others.


in
is

(Credit amount received by bonds in ex


cess of par.) Non-Operating Revenue.
Optional Reserve Accounts. Rent.
1. Casualties and Insurance Reserve. Interest.
2. Other Optional Reserves. Dividends.
Miscellaneous.
Stocks. Revenue Deductions are expenses, taxes
Stocks are securities representing perma and uncollectible bills.
nent interests in the corporation or inter Erpenses
to

are outgoes necessary the


est in the corporation which, if terminable, production
or
of

commodities sold services


are such only at the option of the corpora rendered or collection of revenues.
tlOn. Tares are annual
or

other payments
to

government for public uses.


Classification of Stocks.
by

Uncollectible Bills are those which


Debentures. diligence are in
of

reasonable exercise
First Preferred. capable
of

collection.

86
Expenses are divided into Operating and INCOME ACCOUNT
Non-Operating.
of the
Operating Expenses are those which are
incident to operation of road. - - -- -- - - - - - - - - RAILWAY COMPANY
Non-Operating Expenses are those which for year ended December ...., 1917.
are not incident to operation of road.
Gross Income is Revenues (Operating Operating Revenue:
and Non-Operating) minus Expenses (Op Revenue from Transportation—
Passenger
erating and Non-Operating), Taxes and Un Revenue. . $600,000
collectible Bills. Freight Revenue.... 400,000
From Gross Income deduct Income De Mail Revenue. . . . . . . 300,000
Express Revenue.... 210,000
ductions (Rent, Interest, etc.), leaving Net
Corporate Income. Other Car Earnings. 106,000 $1,616,000

Net Corporate Income is carried to Cor Other St. Ry. Oper. Revenue—
porate Surplus. Advertising ......... $60,000
Appropriation Account covers yearly Other Car & Sta. Pr. 12,000
changes in Corporate Surplus. It contains Rent of Bldgs. &
also adjustments of errors previously made Other Property... 16,000
and extraordinary expenses, profits on sale Rent of Equipment. . 4,000
of capital and profits and losses on sale of Rent of Tracks &
investments. Terminals . . . . . . . . 8,000
Operating Expenses are those which are Sale of Power....... 9,000
necessary to rendering of service or sale Joint Elec. Power
of goods. Revenues ......... 7,000

Cost is cash or money cost, and not price Misc. Receipts....... 32,000 148,000
based on term of credit.
Labor is human service of whatever char Total Operating Revenue. . . . . . . . $1,764,000
acter. Deduct:
Cost of Labor is wages plus salaries plus Operating Expenses:
fees paid employees and expenses paid by Maint. of Way & Struc
corporation. tures . . . . . . . . . . . . . . . $242,000
Cost of Material and Supplies is cost of Maint. of Equipment... 165,000
transportation on same (inward), cost of Traffic Expenses. . . . . . 12,000
tests, cost of storage (estimated), cost of Transportation Exp.–
transportation to place of consumption Power Supply. §89,000
(outward), and proportionate share of ex Oper. of Cars. 100,000 189,000
penses of purchasing department.
Cost of Repairs is cost of labor plus ma General & Miscel. . . . . . 200,000 808,000
terial, minus salvage.
$956,000
Taxes. Uncollectible Bills....... $14,000
Taxes . . . . . . . . .. . . .. . .. . 112,000 126,000
Account should be kept of taxes charge
able to each particular class of operation Income from Operation......... $830,000
and also non-operating.
Non-Operating Revenue:
Taxes assignable to Street Railway Oper Rent Accrued from
ations. Lease of Road. . $16,000
Taxes assignable to Non-Operating Reve Misc. Rent Rev... 12,000
nue S. Interest Revenue... 8,000
Dividend Revenue. 9,000
They must show separately: Profits from Oper.
of Others. . . . . . . 6,000
Misc. Non-Operat
State Franchise Tax on Capital Stock. ing Revenue. ... 10,000 $61,000
State Franchise Tax on Gross Earnings.
Special Franchise Tax.
Taxes on Lands (exclusive of improve Deduct:
ments). Non-Operating Deductions:
Taxes on Land Improvement. Rent Expense..... $12,000
Taxes on Personal Property. Interest Expense... 6,000
Other Taxes. (Detailed report to Commis Dividend Expense. 2,000
sion.) Other Oper. Exp. 3,000
Taxes should be charged monthly as they Misc. Non-Operat
a CCTue. ing Exp. . . . . . . . . 4,000
Three classes of Transportation Corpora Non-Oper. Taxes.. 5,000
tions—a, b, and c; Uncollectible Non
(a) Gross Revenue of over $500,000. Oper. Revenue.. 7,000 $39,000 $22,000
(b) Over $100,000 up to $500,000.
(c) $100,000 or less. Gross Income .................... $852,000

87
Deduct: sociation, held in New York City in 1916.
These charts in a simple and unique way
Income Deductions:
illustrate to the eye the system of classifi
Interest Deductions. . . . . $12,000
cation employed in public utility accounting.
Rent for Lease of Other Taking the Balance Sheet for example:
Rd. & Equip . . . . . . . . . . 6,000
Assets are divided into four classes—fixed,
Other Rent Deductions. 7,000
floating, investments and deferred charges.
Sinking Fund Accruals. . 8,000
Fixed capital, in turn, is divided into landed
Guarantee of Periodic and non-landed. Non-landed is divided into
Payment . . ... . ... . . . . 16,000 tangible and intangible. Tangible is divided
Loss on Operation of into wells and suction, filter plant, pumping
Others . . . . . . . . . . . . . . . 12,000 stations, distribution system, etc., etc.
Other Contractual De Against the total assets are measured two
ductions from Income. 18,000 general items, Liabilities and Proprietor
Amortization of Landed ship, which in turn are divided and sub
Capital . . . . . . . . . . . . . . . 20,000
divided as shown in the drawing. This chart
Amortization of Debt illustrates the equation:
Dis. & Exp. . . . . . . . . . . 13,000
Amortization of Prem Assets=Liabilities-H Proprietorship.
on Debt . . . . . . . . . . . . . 12,000 124,000
The Income Account is illustrated simi
larly. Here three sections are shown,
Net Corporate Income. . . . . . . . . . . . $728,000
headed respectively Operating Income,
(N. B.-Figures are inserted for purposes Non-operating Income and Corporate In
conne.
of illustration only; are not intended to be
representative in amount. As these drawings are measured to scale,
the eye, almost without mental effort, is
enabled to gauge the relative importance
of the various items comprising a Balance
PRESENTING A MIND PICTURE BY Sheet and Income Account.
USE OF CHARTS. The modern tendency is to prefer charts
of financial facts to the usual complex ac
While the foregoing digest of the ac counting and statistical statements, which
counting requirements of the Public Service require much study and analysis for intel
Commission should furnish the student with ligent understanding.
a fair understanding of public utility ac It is the author's impression that this
counts, the author has deemed it desirable graphic method of presenting a Balance
to supplement this information with a Sheet and Income Account is here for the
graphic presentation in chart form of In first time shown. These charts could be
come Account and Balance Sheet taken supported sectionally with further details,
from a paper read by him before the Con as was done by the author in the paper
vention of the American Water Works As referred to.

88
INCOME ACCOUNT
OPERATING IN COME

º:
OPERATING REVENUES
OTHER
OP
Sale or war ER
$325,000. $75,000.

OpenArung ExPENSEs
Maintenance General -
-
Operation fg - -- and .. Income from operation
878,000
§§
& ºt
“..."
40,000.
Administrative
* 50,000.
* 175,000.

NON-OPERATING INCOME_{
f
É?:
| N.comE FROM operation

t-º
§§§
* 175,000.
#:

º:#
cºntifiedpublic
1328broadwa
MinYonxtury º
g
ity
* I60,000. o aso-ooo ------>

CORPORATE INCOME {

GRoss Income
&180,000.

-º: Returnon investment lappropiatorſ:


c wo e-
§ {:}l interest Dividend #s ##3
on bonds on stock §§§ {{#3#
§§§§
#g §: £3
$33:
E* 1 ->
+ º
"50,000. *75,000.
i- :
o
DIAGRAM W.

89
'000's. $5WTº
FOODGIFTSMITIRS

"Ooo'04-1
949ssw ºut ºvoid

‘000'09ſ s
21q15ueºuſ

snoouetºs: W
-FFIFFTI
pu- ~an-tuºun:
-
łuoudinbo. o
puv Slool Io. 3
-- o
vo wo

-s
or
o
k-
ul
or
ch +:

5
cr
-:
:
re a-
3.
— §
t 3.
U
#

2
5|,
£3.
; §;3§-
3
s

;
3.

º
wn
-
2|-

“DoO'ozi
5 3. 3

se!; (113°
17

:
3.

5ulºvo[3
-
*

.* TöööSTSU5Uſºſºld
- -
£13
a

Isuºx
I

shisodoo
*

Ooooººººow
2. 3.

— o
--

Qo
u-J -
H.
3 3

co

-
3

wn
<
-~
—l
-
: 5

5
2. C.

ch
ty
3.
o

-
E

d.
º

uold 121114
3

suo (12ns
puw silowa

000'Gls
popuwn

90
ACCOUNTANT, NEW JERSEY AND ILLINOIS
SERVICES.

Specimen Examination Questions for Accountant and


Statistician, Public Utilities Commission, and
Accountant, Rank 1.

State Civil Service Commission of New that the amount realized on land and build
Jersey. ings will be sufficient to satisfy the mort
gage only, and that plant and equipment
ACCOUNTANT AND STATISTICIAN will realize only 6 per cent of the book
value.
Public Utilities Commission. Prepare a statement of affairs and defi
EXPERIENCE. ciency account.
2. Name the methods of bookkeeping in
This will cover a general examination on general use. What books are kept ordi
the past experience and education of the narily in each case? How is the profit or
applicant. loss ascertained in the different systems?
3. Explain the distinction between ex
ACCOUNTING. penses and disbursements.
1. The Parker Construction Company is 4. What is the difference in the manner
unable to meet its obligations and is forced of keeping the accounts of a public institu
into liquidation. At the time, the receiver tion or public office and a private business?
takes charge of its affairs the following 5. If an audit reveals that capital ex
trial balance is prepared from the com penditures have been charged to expense
pany's books: accounts, or expense items charged to im
provement accounts, what should the audi
Cash .. .. .. .. . .. . .. . .. .. .. . . $500
tor do with such charges?
Land and buildings. . . . . . . . . 10,000
Mortgage on land and build 6. The accounts relating to income, ex
ings . . . . . . . . . . . . . . . . . . . . $8,000 penses and dividends declared for the year
Plant and equipment. . . . . . . . 20,000 to June 30, 1910, of A. B. C. Railway Co.
Creditors . . . . . . . . . . . . . . . . . . . 59,400 are as follows:
Completed contract accounts Dividends on stock................ $45,000
(losses) . . . . . . . . . . . . . . . . 18,000 General expenses ................. 22,248
Capital ... . . ... . . .. .... .. .. . 50,000 Injuries to persons and property.. 5,775
Uncompleted contract ac Interest on funded debt. . . . . . . . . . . . 25,000
counts (outlay). . . . . . . . . 30,000 Interest on unfunded debt......... 300
Securities acquired in settle Maintenance of equipment......... 27,162
ments . . . . . . . . . . . . . . . . . . 15,000 Operation of cars................. 92,030
Debtors' accounts for com Operation of power plant.......... 79,226
pleted contracts. . . . . . . . . 6,000 Other street railway operating rev
Expenses . . . . . . . . . . . . . . . . . . . 6,500 Cillies . . . . . . . . . . . . . . . . . . . . . . . . 4,121
Inventory of materials. . . . . . 2,000 Maintenance of way and structures. 35,225
Profit and loss (deficiency).. 9,400 Revenue from transportation....... 345,182
Traffic expenses ................. 4,498
$117,400 $117,400 Taxes .. . .. . . .. . . .. . .. . . .. .. . . .. . . 2,660

The sureties on the unfinished contracts Prepare income account for year and
estimate that a further outlay of $20,000 show: (a) Percentage of operating expenses
will be required to complete the work and to operating income; (b) percentage of net
realize the contract price of $40,000, and corporate income to operating income.
their offer to take over the materials on 7. An electric company disburses $60,000
hand for $1,500, as part of said cost, is ac on June 30, 1910, in payment of six months'
cepted by the receiver. Of the securities interest on its bonds to that date. How
acquired $5,000 is pledged to secure $11,000 should this transaction be shown on the
due creditors, and $10,000 is pledged to se ledger, and what did the books show in
cure $9,000 due creditors. The company this connection on March 31, 1910, and
owes for taxes on real estate $100 and for what will the books show on July 31, 1910?
salaries and wages of employes $1,200, which 8. A gas company extends its mains at
sums do not appear on the books. The a cost of $5,500, which it charges to “oper
company has discounted customers' notes ating expenses—repairs of mains.” State
for $3,000, of which subsequent advices in reasons for approving or disapproving the
dicate that $1,000 will be dishonored, and charge as above made.
a debtor owing $1,500 on unsecured account 9. What are shown by (a) balance sheet 3
has failed and disappeared. It is estimated (b) income account?
91
costing $60 per lamp per year. Also

25
10. Define (a) revenues; (b) operating arc
expenses. lamps burning 2,600 hours, each per year,
using 455 kilowatt hours electric energy

of
ECONOMICS AND STATISTICS. per lamp per year, costing $45 per lamp
per year.
1. Name and explain five advantages to Farmington: arc lamps burning each

76
the State or to the corporation from a Uni 4,000 hours per year, using 718 kilowatt
form System of Accounts for public service electric energy per lamp per year,

of
hours
corporations. costing $56 per lamp per year. -
2. To what accounts on the books of an Hopeville: arc lamps burning

36
each
electric corporation would you charge: 3,960 hours per year, using .177 kilowatt
(a) Money paid to another corporation energy per lamp per hour,

of
hour electric
sufficient to cover the interest on the lat costing $0.10 per kilowatt hour.
ter's funded debt in accordance with the
Everson: 125 incandescent lamps burning
terms of a contract by which its property
on an average hours per night, using 100

of 10
is held under lease;
kilowatt hours electric energy per lamp
(b) Cost of material and labor necessary per year, costing $7.50 per lamp per year.
to restore damaged electric generators to Monocking: arc lamps burning 4,000

23
their original state of efficiency; hours per year, using 562 kilowatt hours of
(c) Cost of coal, wire, lamps and tools electric energy per lamp per year, costing
added to the general storeroom stock;
$72 per lamp per year.

56
Also incandes
(d) Cost of a new transformer installed cent lamps burning 4,000 hours per year,
in the generating plant; using 110 kilowatt hours electric energy

of
(e) The amount estimated to be neces per lamp per year, costing per lamp per

$6
sary to cover obsolescence, wear and tear, year.
etc., not made good during the year. Tabulate the above information such

in
3. A railway company of New Jersey you most significant, giv

to
as
form seems
scrap two cars which originally cost $3,000 ing title table, column headings, etc.

of
each; the salvage obtained is $150 per car. given
Where the same information

in
is
The company have been setting up a de different forms for different municipalities,
preciation reserve for only two years, at may be reduced common basis for

to
it

a
the rate of 10 per cent per annum on the tabulation purposes.
cost. To replace these cars and to improve 10. What do you understand by (a) car
tle service the company purchased five cars mile; (b) train-mile; (c) passenger-mile;
at a cost of $3,250 each. Show how the (d) ton-mile? The freight revenue

of
above matters are taken up in the equip

a
certain railroad 2,242 miles length $25,

in

is
ment account on the ledger.
revenue freight

of
451,236. The total amount
4. Define revenue and operating expenses.
carried 22,815,528 tons and the average
is

Give three accounts which would be classi


miles hauled per ton 102.84.
of

number
fied under each of the above headings by Compute the total number

of
ton-miles of
(a) railway, (b) gas corporation, (c) elec revenue freight, and the average freight
tric corporation. revenue per ton-mile.
5. In compiling statistics to throw light
upon the proper maximum rate for trans
portation of passengers on any one of the COMMERCIAL ARITH METIC.
steam railways of New Jersey, what statis
NOTE:-All work perfect
on
tical material would be needed, to what must appear the
candidate's paper. mark will not
A
it,

sources would you go for what tabu


in

be given when answers only are submitted.


lar form would you present it?
The assessed valuation of certain
1.

What meant by “weighting the


to a
6.

is

city $1,650,780. voted erect


It

average"?
is

is

a
building cost $25,
to

Show some one peculiarity which char new school which


is
7.

the following kinds se 000. What must be the tax rate in order
of

of

acterizes each
railway company: First just cover the cost the building and
to

of

curities issued by
a

mortgage bonds; debenture bonds; collat allow per cent for collecting the tax?
2

eral trust bonds ;income bonds; equipment certain machine depreciates


at
If
2.

trusts. 3% per cent per annum, and


of

the rate
(a) What strong argument there after three years inventoried $1,576.20,
at
is
8.

is

for the regulation the capitalization what was its original value?
of
of

public service companies by permanent paid $4,432.50 for draft on Syra


3.

a
a

cuse, N. Y., payable days after sight,


30

administrative board?
at

(b) State two arguments cap per cent discount, interest per cent,
of

favor
in

at
1

no days grace.
of

italization based upon earning capacity. What was the face


of

Would these arguments apply the cap the draft?


to

public service companies gen partnership Janu


of

italization and formed


4.

erally? ary A's capital was $7,500 and B's


1,

1907.
The lighting companies several dif capital was $7,500; each partner was allowed
in
9.

ferent municipalities furnish street lighting per cent interest on his capital and was
5

service as follows: charged per cent interest on the sums


6

Green Falls: 100 arc lamps burning each withdrawn. withdrew April 24, $300;
A

hours per year, using 648 kilowatt September 12, $650. June
6,

4,000 withdrew
B

electric energy per lamp per year, October What was each
9,

hours $550;
of

$200.

92
partner's share in the business January 1, Organization costs . . . . . . . . . . . . . . . . $5,000
1908? Land . . . . . . . . . . . . . . . . . . . . . . . . . . . . . 20,000
5. Copy the following table showing the Capital stock (common). . . . . . . . . . . 200,000
annual appropriations for the years 1893 Production expense . . . . . . . . . . . . . . . 65,000
and 1907: Production equipment . . . . . . . . . . . . . 150,000
Municipal street lighting earnings. . 20,000
% of In Municipal power earnings. . . . . . . . . . 5,000
Appropriations crease Or
Transmission expenses . . . . . . . . . . . . 15,000
Purpose 1893 1907 Decrease
Distribution expenses . . . . . . . . . . . . . 20,000
Canal... 1,623,250.00 1,190,702.00 Accounts receivable . . . . . . . . . . . . . . . 25,000
Schools. 3,972,500.00 6,125,887.48
Transmission equipment . . . . . . . . . . . 10,000
Income. 1,346,019.64 5,386,888.11
Accounts payable . . . . . . . . . . . . . . . . . 8,000
General.$10,484,566.34 $16,162,377.87 Depreciation 40,000
reserve . . ... . .. . .. . . .
Depreciation . .. .. .. .. .. ... . .. .. . . . 10,000
Totals, Depreciation reserve fund. . . . . . . . . . 25,000
Distribution equipment . . . . . . .. . . . 350,000
Find the totals of the first two columns. expense . . . . . . . . . 9,000
Commercial .. . . .
Compute and insert the five percentages
lacking in the last column, giving them in General and miscellaneous expense. 18,000
Earnings from residuals and by
per cents and hundredths of a per cent. products 15,000
. . . . . . . ... . . . . . . . . . . . .
6. Copy the following table, a compara
Materials and supplies on hand.... 10,000
tive summary of track mileage classified
according to motive power: Consumers' deposits . . . . . . . . . . . . . . 5,000
Sinking fund reserve. . . . . . . . . . . . . . . 20,000
Miles % of In Funded debt ... . . . . . . . . . . . . . . . . . . . 340,000
of Track in crease or Commercial heating and lighting
Power 1902 1890 Decrease earnings . . . . . . . . . . . . . . . . . . . . . . 150,000
Electric . . . . . . . . . . 21,908 1,262 Taxes . . . . . . . . . . . . . . ... .... ... . . .. 6,000
Animal . . . . . . . . . . . 259 5,661 Taxes accrued . . . . . . . .. . .. .. . .. . .. 6,000
241 488 Insurance prepaid . . . . . . . . . . . . . . . . . 1,000
Cable .. . .. . .. . .. .
Interest on funded debt. . . . . . . . .. . . 15,000
Steam . . . . . ... . . . . 170 711
Cash on hand. . . . . . . . . . . . . - - - - -- - - 14,000
Total. Common stock dividends. . . . . . . . . . 20,000
.. . . . 22,578 8,122
Material dividends unpaid. . ... . .. .. 5,000
Compute and insert the five percentages Uncollectible bills .. . .... ... . .. . .. 500
lacking in the last column, giving them in Rents . . . . . . . . . . . ... . . . . . . . . . . . . . . . 4,000
per cents and tenths of a per cent. Unextinguished discount on bonds. 30,000
7. From the following figures prepare a Reserve for amortization of intangi
table showing the average number of rides ble capital . . . . . . . . . . . . . . . . . . . . 10,000
on street and electric railways per inhabi Audited vouchers and wages unpaid 2,000
tant, arranging the States in the order of Matured interest and dividends re
decreasing average: ceivable . . . . . . . . . . . . . . . . . . . . . . . 1,500

Average Securities of other corporations. ... 25.000


Fare, Rides per Rent from lease of gas plant. . . . . . . 2,000
Miscellaneous rent revenues. . . . . . . 1,000
Pas- Capita Surplus balance July 1, 1913. . . . . . . . 20,000
Population, sengers, per
State 1900 1902 Annum Instructions: Draw up a (1) trial bal
California 1,485,053 182,196,999 ance, (2) income account, (3) profit and
D. of Columbia 278,718 66,162,321 loss statement, and (4) balance sheet.
Massachusetts 2,805,346 461,745,615
New Jersey 1,883,669 188,976,899
New York 7,268,894 1,144,491,509 ILLINOIS PUBLIC UTILITY LAW.
Pennsylvania 6,302,115 642,513,812
Rhode Island 428,556 62,279,165 1. What are the powers and limitations
of the Illinois Public Utilities Commission
with regard to systems of accounts for pub
lic utilities?
Illinois State Civil Service Commission. 2. What is a “public utility” in Illinois?
ACCOUNTANT, RANK 1.
What reports may the commission require
from utilities?
Illinois Public Utilities Commission. 3. State fully what power the commis
sion has over accounts in cases where a
Scope and Weights: Training and ex public utility engages in some other busi
perience, 4; accounting problem, 2; Illinois ineSS.
Public Utilities Law, 1; auditing, 1%; ac 4. What does the Public Utilities Law
counting methods and problems, 1%. provide with regard to (a) keeping records
outside State? (b) The power of the com
ACCOUNTING PROBLEM. to examine utilities' books, records,
mission
etc. :
The following is a list of the general 5. (a) Under what conditions and for
ledger balances of a gas utility as of date what purposes may public utilities in Illi
June 30, 1914: nois issue bonds? (b) If you were asked
93
to audit the books of a utility which seeks property. The reserve on the books for
to issue bonds, what would be the lines of the entire property is $10,000. Show by
inquiry which you would pursue? journal entries the steps necessary to care
for these transactions.
AUDITING. 4. An electric utility makes an extension
costing $200 to reach to an individual con
1. In what manner may audits be classi Sumer with the agreement that the consum
fied? Which do you think are of the most er shall pay the cost of the extension when
importance in the regulation of public utili made but that no payment is to be made
ties? by the consumer for the first $200 of cur
2. In an audit you find that a utility has rent used. How would you treat this on
purchased a large stock of materials and the books of the company? State your rea
supplies during the past year at low prices SOrl S.

and at the time of inventory prices have 5. How, in your opinion, should premium
increased to a considerable extent. You on stock sold be recorded on the books of
find that the utility carries the supplies on the company? State reasons.
its books at the increased inventory value 6. In checking the annual reports of pub

to lic
and records the margin of increase as a utilities, what relation would you expect

or
credit to surplus. How would you treat find between (a) the yearly surplus
this entry in auditing? Give your reasons. deficit appearing the balance sheet? (b)

in
3. (a) What do you understand by a The annual charges for depreciation and

as
the depreciation appearing

in
trial balance, and what are its uses to an reserve

in to of
the balance sheet? (c) The relation the
auditor? (b) What does the trial balance
prove regarding detail nlant and equipment accounts the
the correctness of the property and plant account appearing
books? Explain. the
balance sheet?
4. In auditing the accounts of a utility 7. Give the primary operating expense
you find that the practice of the company
accounts outlined for steam roads by the
has been to charge all utility equipment Interstate Commerce Commission.
expenses, such as the expenses of automo (a) Give the detail accounts included
8.

biles, horses, etc., to operating expenses.


the primary maintenance ac
of
under one
The facts show, however, that all utility (b) Give the classes

of
counts. revenues
equipment was used not only for operating
outlined the Interstate Commerce Com
in

purposes, such as repair work, etc., but also mission's railroad classification.
in connection with construction, reconstruc
tion, and also for the delivery of material 9-10. (a) List the primary operating
revenue, operating expense and plant and
sold, as gas stoves, electric appliances, etc. equipment accounts, prescribed by the
as
Outline the manner in which you would
treat such costs in determining the normal
State Public Utilities Commission of Illi
C,

nois, for Class Telephone Companies.


operating expenses of the utility. (This the Interstate Com
as

the same
is

5. Distinguish between cash collections merce Commission classification for Class


from customers and earnings. What are Telephone Utilities.)

of
Give each the
C

the dangers that may result from a failure primary accounts number. (b) Indicate
a

to properly differentiate between them? by number under which primary account


Explain fully. as listed under “a” above each of the fol
State the method you would follow lowing items belongs: Refunds subscrib
to

6.
in auditing: (a) Cash, (b) Accounts receiv ers for line being out order; rental from
of

able, (c) Accounts payable. subscribers for rural service; replacing


broken insulators; installation of switch
board; office supplies; land used for store
ACCOUNTING METHODS AND PROB house; revenues from messenger service;
LEMS. labor setting new poles; profits on materials
1. A telephone company has revenue of and supplies; labor installing telephones;
salaries; cost patent rights;
of

$1,000 per year and a plant value of $3,500. onerators'


insurance; legal fees organization; bat
in

In your judgment how much detail should tery expenses; chief operator's desk; repair
be required in the accounts as to operating cords; automobile; repair
revenues, operating expenses, and property ing switchboard
ing automobile; lighting expense central
at

and plant? Why?


office; law expenses; charges for extra di
2. Discuss the importance of providing rectory insertions; office rent; payments
to

for depreciation and state the difference other companies for rent pole line; earn
of

between the straight line and sinking fun i ings from rental pole line other com
of

to

methods. Illustrate by journal entries the Danies; charge made for installing phone;
steps necessary to provide for depreciation power purchased; pay and expenses
of

cost
under these methods. messengers; expenses changing loca
of

of

A telephone company replaces phone; office salaries:


of

3. a tion subscriber's
switchboard which had an original cost of repair relay racks;
of

discounts allowed
$1,000, by one costing $1,500. $100 is real subscribers for prompt payment; expense
ized from the sale of the old board and printing directories: repair operator's
of

of

an adequate depreciation reserve has been chair; expense billing subscribers; ad


of

established to meet the replacement require vertising expenses; charges telegraph


to

ments at the original cost of all units of companies for making collections.

94
CERTIFIED PUBLIC ACCOUNTAINT—ANSWERS TO
NEW YORK STATE EXAMINATION QUESTIONS.

NOTE:—In the examination which fol Ques. 1:—John Doe commenced business
lows Mr. Max Schlesinger, B.C.S., C.P.A., with a cash capital of $15,000. At the close
made the high percentage of 95. The an of his fiscal period the ledger accounts
swers as here given are an exact reproduc were: accounts receivable $4,312.50; mer
tion of those given at the examination. chandise debit balance $5,062.50; accounts
On all but Question 4 he received the payable $5,375; expense $900. Doe's total
full rating of 25; on Question 4 he received loss was $2,775.
20. Prepare a statement of assets and liabil
The Author takes this occasion to thank ities and the profit or loss.
Mr. Schlesinger for his kind permission to
publish the answers, which add a valuable
chapter to the book. Ques. 2:—A & B, commission merchants,
suspect their cashier of embezzlement.
University of the State of New York.
37th Accountant Examination. From the following data determine whether
or not their suspicions are well founded,
PRACTICAL ACCOUNTING—PART I. and produce a balance sheet and profit and
loss statement to prove or disproce the sus
Tuesday, June 23, 1914—1.15 to 4.15 P. M. picion:
only.
Sales . . . . . . . . . . . . . . . . . . . . . .... .. $42,400
The Practical Accounting paper consists
of Part I and Part II. Cash receipts, customers. . . $42,000 .. .. ..
Freight ... . . . ... .. .. .. .. . . 4,240 2,480
“The Regents of the University shall Duty . . . . . . . 2,120 1,240
.. .. .. .. .... . .
make rules for the examination of persons
applying for certificates under this act, and Dock charges .. . .. .. .. .. . . 212 124
may appoint a board of three examiners for Custom house charges. . . . . 90 45

the purpose. .”—Laws of 1896, Ch. Interest (account sales at


312, Sec. 2. 6%) . . . . . . . . . . . . . . . . . . .. .. .. 248

Answer two questions but no more. An Commission (5% on sales)


Office expense . . . . . . . . . . . .
.. . .. .
2,000
1,240
. ... ..
swers in excess of the number required will Documentary
not be considered. Do not repeat questions advances.... 20,000 12,000
Acceptances against ship
but write answers only, designating by num ments . . . . . . . . . . . . . . . . 12,000 20,000
ber as in question paper. Check (V) the
number of each one of the questions you Analysis of account sales ledger debits,
have answered. Each complete answer will
receive 25 credits. Papers entitled to 75 or duty $875, freight $1,560, dock charges $70,
more credits will be accepted. custom house charges $40.
ANSWER (1):

John Doe
Statement of Assets and Liabilities as of Date.
Assets: Liabilities:
Cash . . . . . . . . . . . . . . . . $10,100. Accounts Payable. . . $5,375.00
Mdse. Inventory...... 3,187.50 John Doe Capital—
Accounts Receivable... 4,312.50 less loss of $2,775..12,225.00

Total Assets . . . . . . . . . . . . $17,600.00 Total Liabilities. . . . . . . . $17,600.00

Statement of Profit Loss.

(1) Cost of goods sold—


-
Sales . . . . . . . . . . . . . . . . . . . . . . . . . . . . . $4,312.50

Purchases . . . . . . . . . . $9,375.00
Less Moise. on hand. 3,187.50

Cost of goods sold. . . . $6,187.50

Gross Profit (Loss) on


Sales . . . . . . . . . . . . . . . . . . . . . . . 1,875.00
(2) Expenses ..................... 900.00

Net (Profit) Loss............. $2,775.00

95
Statement Proving Inventory on Hand.
Total Loss . . . . . . . . . . . . . . . $2,775.00 The Balance on the Mdse. account
Deduct expenses included. 900. is . . . . . . . . . . . . . . . . . . . . . . . . . . . . $5,062.50
Deduct loss on trading............ 1,875.00
Loss on trading........... $1,875.00
Being the excess of purchases The Balance being goods on hand
price over the sales price. at closing time. . . . . . . . . . . . . . . . $3,187.50

Ques. 3:—John Adams lost his stock of Statement of Profit Loss, proving the net
merchandise May 1, 1914, through a flood
profits applying, inventory as ascertained.
Sales . . . . . . . . . . . . . . . . . . . . . . . . . . . $81,688.04
in the Mississippi river.
Cost of goods sold:
Adams applied to the local Mutual Flood Inventory Dec., 1911. . $1,568.62
Insurance Society for reimbursements, Purchases . . . . . . . . . . . . 55,415.82
claiming a loss of $5,886.35 on merchandise Labor Productive..... 19,499.58
Sundry factory exp.... 3,201.92
stock. From the following data ascertain Repairs ... ... .. . .. . .. 16.00
his merchandise inventory:
Total . . . . . . . . . . . . . . . $79,701.94
Net profits May 1, 1914, $4,452.91; draw
ings $1,598; legal expenses Less Inventory May 1,
$17.50; interest 1914 . . . . . . . . . . . . . . . 4,384.61
debit $313; advertising $14; commissions
debit, $961.01; insurance $196,23; sales $81, Cost of goods sold........... 75,317.33

688.04; inventory December, 1911, $1,568.62;


Gross Profits on Sales........ $6,370.71
purchases $55,415.82; labor productive $19, Selling Expenses:
499.58; telephone $416.06; sundry factory ex Commissions .. .... . . . $961.01
penses $3,201.92; repairs $16; surplus May Advertising .. . ... . . .. 14.00 975.01
1, 1914, $2,854.91.
Selling Profit . . . . . . . . . . .. .. .. $5,395.70

ANSWER (3): Administration Expenses:


Legal expense......... $17.50
Re John Adams. Telephone . .. .. .. .. . .. 416.06
Insurance .. . ... .. . .. . 196.23 629.79
Statement Showing Mdse. Inventory on the
1st of May, 1914. Ordinary profits from operations $4,765.91
Deductions from Income:
(1) Net Profits, May 1st, 1914. . . $4,452.91 Interest debit . . . . . . . . . . . . . . . . 313.00
Add items which were deducted
to arrive at the Net Profits: Net Profits, May 1st, 1914. . . . . . . $4,452.91
Legal expenses. . . . . . . . $17.50 Less drawings . . .. .. .. ... . .. . .. . 1,598.00
Interest debit . . . . . . . . . 313.00
Advertising . .. .. ... .. 14.00 Surplus May 1st, 1914. . . . . . . . . . . $2,854.91
Commissions debit.... 961.01
Insurance .. .. .. .. . .. . 196.23
Telephone . . . . . . . . . . . . 416.06 1,917.80 NOTE:—In solving this problem I as
sumed that John Adams has not closed his
Gross Profits Re-established... $6,370.71 books since December, 1911, as there is
nothing indicative to the contrary, and on
(2) Total Sales . . . . . . . . $81,688.04 this assumption I based my solution.
Deduct Gross Profits. 6,370.71
PRACTICAL ACCOUNTING—PART II.
Cost of goods sold. . . . . . . . . . . . $75,317.33
Wednesday, June 24, 1914—9.15 A. M. to
(3) Total cost of goods: 12.15 P. M. only.
Inventory Dec., 1911. . $1,568.62
Purchases . . . . . . . . . . . . 55,415.82 Answer two questions but no more. An
Labor Productive. . . . . 19,499.58 swers in excess of the number required
Sundry factory exp. ... 3,201.92 will not be considered. Do not repeat
questions but write answers only, designat
Repairs .. . .. ... . .. . .. 16.00
ing by number as in question paper. Check
Total . . . . . . . . . . . . . . . . . . . . . . $79,701.94 (V) the number of each one of the ques
tions you have answered. Each complete
Deduct cost of goods sold..... 75,317.33 answer will receive 25 credits. Papers en
Inventory May 1st, 1914. . . . . . . . . $4,384.61 titled to 75 or more credits will be accepted.
Amount claimed by John Adams 5,886.35 Ques. 4:—A contracts with a textile es
tablishment to sell the mill's annual output
Excess claimed over inventory on the following conditions:
ascertained $1,501.74 The mill is to bill the output to A at cost.
A is to finance the mill to the extent of 75%
96
of cost on receipt of goods. The balance Prepare A's financial statement.
is to be remitted by A, as the various ship
ments are sold, less 5% and advances. At
the end of a year an analysis of A's affairs
reveals the following, as shown by his Ques. 5:—X and Y purchase an invoice
books, the goods being sold at 10% profit
factory cost (mill shipments $7,327, of coffee for $12,000. X contributes $7,500
##,
Debits Credits
and Y $4,500. They sell Z a one-third in
Mill advances. . . . . . . . . $5,545,938 $5,000,000 terest in the venture for $6,000. How much
Mill sales. . . . . . . . . . . . . 6,400,000 7,840,710
Freight and cartage... 90,000 80,000 of the $6,000 should X and Y receive, re
Customers ... . . . . . . . . . 7,840,710 7,632,200 spectively, X, Y
in order to make and Z
Cash . . . . . . . . . . . . . . . . . 7,610,200 5,635,938
Discounts . . . . . . . . . . . . 22,000 equally interested?
Commission . . . . . . . . . . .. .. .. . . 320,000
Mill account........ . . . . . .. .. . . 1,000,000

$27,508,848 $27,508,848 ANSWER (4):

A's Financial Statements.


(1) Pro Forma Account Sales:
Sales for which I am account
able . . . . . . . . . . . . . . . . . . . . . . . . $6,400,000.00
Charges against mill:
Freight & Cartage.. $80,000
Commissions - - - - - - - 320,000
Advances applicable
thereto - - - - - - - - - - - 5,000,000

Total charges 5,400,000.00

Balance due to mill. . . . . . . . . . . $1,000,000.00

A's Balance Sheet.


(2) Assets: Liabilities and Capital:
Mill advances to be deducted $545,938.00 Balance of sales unaccounted $1,440,710.00
Freight cartage unapplied... 10,000.00 Mill account . . . . . . . . . . . . . . . 1,000,000.00
Customers . . . . . . . . . . . . . . . . . . 208,510.00 A's Capital and Surplus. . . . . 370,035.50
Cash . . . . . . . . . . . . . . . . . . . . . . . 1,974,262.00
Commission receivable on
balance of sales. . . . . . . . . . . 72,035.50

Total Assets . . . . . . . . . . . . . . . $2,810,745.50 Total Liabilities Capital..... $2,810,745.50

(3) Dr. A's Profit & Loss Account. Cr


Discounts lost ... ... .. . .. . .. $22,000.00 Commissions deducted....... $320,000.00
Balance ... .... .. . .. . ... .... 370,035.50 Commissions receivable. . . . . 72,035.50

Total .. .. . . .. . .. . .. . . .. ... . $392,035.50 $392,035.50

NOTE:—This problem is solved on the


assumption that discounts be charged to
the agent.

Ques. 6:—On January 1, 1914, The Arling During the period from January 1 to De
ton Company's records show the following cember 31, 1914, purchases of raw materials
condition of its accounts: amounted to $241,249.35; factory pay rolls
Inventory of raw materials $46,864.26; $377,381.70; superintendence $114,300; fac
accrued factory pay roll, applied and dis tory expenses including wages not applied
tributed, $2,495.34; goods in process at prime to cos taccounts $74,538; interest paid on
cost $191,665.32;the further value of $24, notes $3,600; dividends received $15,012.
111.51 for the factory overhead, also $36, During the period mentioned the opera
224.76 to cover superintendence; finished tions in the factory comprised: raw ma
goods in stock show a total cost of $64, terials requisitioned for consumption $239,
968.03. 461.02; wages applied and distributed to
97
manufacturing cost $360,751.20, and to fac On December 31, 1914, the goods in
tory expenses $17,878.17 included in the sum process included, in addition to prime cost,
stated in the paragraph above. factory overhead amounting to $25,317.06
There were also forwarded from the fac and superintendence $38,035.98, and accrued
tory to the warehouse finished goods at factory pay roll applied and distributed
prime cost, covering materials $235,627.74 amounting to $3,743.01.
and labor $355,001.25. The cost of goods Show the cost controlling accounts as
sold during the year was $755,849.70 and they would appear in the general ledger,
the proceeds from goods sold $907,019.64. their operation and the resulting net profit.
ANSWER (6):
Raw Materials.

1/1/14 Balance .... .. .. ... . $46,864 26 12/31 Goods in Process.... $239,461 02


Creditors . ... . .... .. 241,249 35 Balance . . . . . . . . . . . . . 48,652 59

$288,113 61 $288,113 61

1/1/15 Balance .. .. . . ... .. . $48,652 59

Goods in Process.

1/1/14 Balance . . . . . . . . . . . . $191,665 32 12/31 Finished Goods. . . . . . $590,628 99


Raw Materials. . . . . . 239,461 02 Balance . . . . . . . . . . . . . 201,248 55
Wages . . . . . . . . . . . . . 360,751 20

$791,877 54 $791,877 54

1/1/15 Balance . . . . . . . . . . . . $201,248 55

Factory Overhead.

1/1/14 Balance .. .. ... ... .. $24,111 51 12/31 Finished Goods. .... . $73,332 45
12/31 Factory Payroll..... 17,878 17 Balance .. . .. . .. .. . . . 25,317 06
Cash .. . .. ... ... .. .. 56,659 83

$98,649 51 $98,649 51

1/1/15 Balance ............ $25,317 06

Superintendence.

1/1/14 Balance . . . . . . . . . . . . $36,224 76 12/31 Finished Goods. . . . . . $112,488 78


12/31 Cash . . . . . . . . . . . . . . . 114,300 00 Balance .. . .. . .. . .. . . 38,035 98

$150,524 76 $150,524 76

1/1/15 Balance .. .. . . .. .. . . $38,035 98

Factory Payrolls.

12/31/14
--
Cash . . . . . . . . . . . . . $377,381 70 1/1/14 Goods in Process... $2,495 34
Balance . . . . . . . . . . 3,743 01 12/31 Goods in Process... 360,751 20
{{
Factory Overhead... 17,878 17

$381,124 71 $381,124 71

1/1/15 Balance .. ... . ... .. . $3,743 01

Finished Goods.

1/1/14 Balance . . . . . . . . . . . . $64,968 03 12/31 Sales . . . . . . . . . . . . . . . . $755,849 70


Goods in Process... 590,628 99 Balance . . . . . . . . . . . . . 85,568 55
Factory Overhead... 73,332 45
Superintendence .... 112,488 78

$841,418 25 $841,418 25

1/1/15 Balance ............ $85,568 55 -


98
Interest Paid on Notes.
12/31/14 Cash ... .. . ... .. .. $3,600 00 12/31 Profit Loss. . . .. . .. . . $3,600 00

$3,600 00 $3,600 00

Dividends Received.

12/31/14 Profit Loss....... $15,012 00 12/31 Cash . ... . .. . .. . ... . . $15,012 00

$15,012 00 $15,012 00

-
Sales.

12/31/14 Finished Goods... $755,849 70 12/31 Accounts Receivable... $907,019 64


Profit Loss. . . . . . . 151,169 94

$907,019 64 $907,019 64

Profit Loss Account.

12/31/14 Interest Paid...... $3,600 00 12/31 Sales . . . . . . . . . . . . . . . . $151,169 7.

—r
Surplus .. . .. . . .. . 162,581 d)4 Dividends Rec. . . . . . . 15,012 00

(64
$166,181 404 $166,181

NOTE:—The Profit Loss account ar


&

to
introduced this solution
in

order

at in
is

the net profits from operation.


at

This result could also


be

rive arrived statement

in
from starting with the gross profits given by the problem and other income, deduct
as

charges against income, and get the result transferable surplus.


to
ADDITIONAL BOOKS TO STUDY.
The following additional books will be BOOKKEEPER, United States Service.
found valuable to study for the various po —The first five subjects of this examination
sitions as enumerated; all of these books are the regular Federal First Grade Clerical
may be obtained from the Civil Service test. For these five subjects, study the
Chronicle, No. 23 Duane Street, New York: Chronicle's specially prepared book, “Fed
BOOKKEEPER, New York City Service.
eral First Grade Clerical Examination In
—“Bookkeeping Simplified” (Ney); cloth, struction,” described in Book List in the
$1; by mail, $1.10. “Bookkeeping Self rear of this book; $1.50, mail $1.60. As auxil
Taught” (Goodwin); cloth, $1; by mail, iary aids it also will be helpful to study
(Griffith); “Champion Spelling Book,” 25 cents, by
$1.10. “Practical Bookkeeping” mail, 30 cents, and “150 Lessons in Plain,
288 pages, 140 illus.; covers Theory of Ac
counts, Single Proprietorship. Accounts, Rapid Penmanship,” 25 cents, by mail, 30
Partnership Accounts, Wholesale Accounts, cents. For the Bookkeeping branch of this
Brokerage Business, etc.; test, study the same bookkeeping books as
Commission and
are mentioned under Bookkeeper, New
288 pages, 140 illus.; cloth, $1.50; by mail,
$1.65. “Filing Systems” (Cope); 192 pages; York City Service.
cloth, $1; by mail, $1.10. “Sixty Lessons in
Business Arithmetic,” 50 cents; by mail, 60 .
BOOKKEEPER, New Jersey Service.—
cents. “150 Lessons in Plain, Rapid Pen Same books as for Bookkeeper, New York
manship,” 25 cents; by mail, 30 cents. The State Service.
Chronicle's book, “Senior Grade Clerical ACCOUNTANT, New York City Service.
Examination Instruction,” $1.50 (mail $1.60),
contains valuable chapters on Arithmetic
—“Handbook of Municipal Accounting”
(Metz); 32 folders and charts; $2.25; by
and also covers Government. See descrip mail, $2.40. “Municipal Accounting” Eggles
tion of this book in Book List in the rear
ton); 456 pages; half-leather, $4; by mail,
of this book.
BOOKKEEPER, New York State Ser $4.20. “C. P. A. Auditing Questions” (Bent
ley); 247 pages; cloth, $2; by mail, $2.15.
vice.—The Chronicle's book, “Senior Grade “Corporation Accounts and the Voucher
Clerical Examination Instruction,” $1.50 System” (Griffith); 160 pages, 50 illus.;
(mail $1.60), excellently covers Arithmetic cloth, $1; by mail, $1.15. “First Lessons in
and includes chapters on Spelling and Pen Finance” (Cleveland); cloth, $1.25; by mail,
manship. For Penmanship, also study “150 $1.35. “An Introduction to the Study of
Lessons in Plain, Rapid Penmanship,” 25 Statistics” (Yule-Undy); 382 pages, 53 fig
cents, by mail, 30 cents. For Spelling, ures; cloth, $3.50; by mail, $3.70.
“Champion Spelling Book,” 25 cents, by
mail, 30 cents. Also the Filing and the ACCOUNTANT, New York State Ser
Bookkeeper books mentioned for Bookkeep vice.—Study same books as for Accountant,
er, New York City Service.
New York City Service. But for Account
BOOKKEEPER AND STENOGRA ant, Public Service Commission, study also
PHER, New York State Service.—Same “Railroad Accounting” (Hooper); $2.25; by
books as for Bookkeeper, New York State mail, $2.41.
Service, with the addition of the Chronicle's
book, “Stenographer and Typist,” described ACCOUNTANT, New Tersey Service.—
on Book List in the rear of this book; price, Study same books as for Accountant, New
60 cents; by mail, 65 cents. York City Service.

CIVIL SERVICE CHRONICLE CO-OPERATIVE


- LEAGUE 23 Duane Street, New York.
Fill this out and mail it to us with $2.00, and we will mail you the Membership Card.

In consideration of the payment of a membership fee of $2.00, the receipt of which is


hereby acknowledged,
Name
Street
City and State
is entitled to the Civil Service Chronicle mailed to his or her address in the United States,
weekly, postpaid, for one year, beginning with the issue of
and ending with the issue of.................................
And to a discount of 10 per cent from the regular retail prices of all books published
or, sold by the Civil Service Chronicle, or any book on any subject published by any pub
lisher who allows trade discounts, upon presentation of this card:
And to free Notarial services in matters relating to the civil service;
And to the privileges of the Civil Service Chronicle's Information Bureau, Civil Service
Reference Library, Questions and Answers Service, and of Inspecting and Copying Exam
ination Questions.

100
CO N TE N TS.
PAGE
Information regarding examinations for Bookkeeper and Accountant
positions in the New York City, New York State, New Jersey
and United States Services. . . . . . . . . . . . . . . . . . . . . . . . . . . . . . . . .
Bookkeeper, New York City Service—Answers to Examination Ques
tions and Specimen Questions for 2nd, 3rd, 4th and 5th Grades.
Bookkeeper, New York State Service—Answers to Examination
Questions and Specimen Questions for 4th, 5th and 6th Grades. 34

Bookkeeper, United States Service—Answers to Examination Ques


tions and Specimen Questions................. • * * * * * * * * * * * * 47

Bookkeeper and Junior Bookkeeper, New Jersey and Illinois Ser


vices—Answers to Examination Questions and Specimen Ques
tions . . . . . . . . . . . . . . . . . . . . . . . . . . . . . . . . . . . . . . . . . . . . . . . . . . . . 52

Board of Education Examination Questions for License to Teach


Accounting and Business Practice in High Schools and Evening
Schools, New York City. . . . . . . . . . . . . . . . . . . . . . . . . . . . . . . . . . . 57

Practice Questions and Answers on Bookkeeping, Commercial Prac


tice and Terms. . . . . . . . . . . . . . . . . . . . . . . . . . . . . . . . . . . . . . . . . . . 60

Accountant, New York City Service—Answers to Examination Ques-


tions and Specimen Questions for 4th and 5th Grades. . . . . . . . . 67

Accountant, New York State Service—Specimen Examination Ques


tions for Accountant and Junior Accountant, Public Service
Commission, and Expert Accountant, Dept. of Efficiency and
Economy . . . . . . . . . . . . . . . . . . . . . . . . . . . . . . . . . . . . . . . . . . . . . . . . - 80

Digest of Uniform Accounting Systems Prescribed by the Public


Service Commission, First District, New York. . . . . . . . . . . . . . . 84

Accountant, New Jersey and Illinois Services—Specimen Examina


tion Questions for Accountant and Statistician, Public Utilities
Commission, and Accountant, Rank 1. . . . . . . . . . . . . . . . . . . . . . .
Certified Public Accountant—Answers to New York State Examina
tion Questions . . . . . . . . . . . . . . . . . . . . . . . . . . . . . . . . . . . . . . . . . . .

101
D EC EMBE R 1 st , 19 1 7.

CIVIL SERVICE B00KS


Published or for Sale by the

23 Duane Street - - - - - - - New York


IN OT I C E .
In addition to books specially prepared for Civil Service Examinations, this list Includes
handbooks and textbooks that come nearest to serving as substitutes. The specially pre
pared Civil Service books, with a few exceptions, are publications of the Civil Service
Chronicle. The handbooks and textbooks not specially prepared for civil service examina
tions are indicated by (*) marked after the price. They have been selected after an ex
haustive research of the catalogues of hundreds of publishers, and each such book recorm
mended is as nearly as possible the most suitable, the simplest in manner of treating the
subject, and most popular in price.
The Publishers have in their office files a descriptive card catalogue containing SEVERAL
THOUSAND OTHER SUCH BOOKS. If you do not find what you want in this Book List,
let us know for what position you desire to study and we shall be pleased to recommend
the most suitable books.

FIRE DEPARTMENT. N. . Y., Fire College. Much of the instruction


is in the form of Questions and Answers. 35,000
words. Pa., $0.75; mail $0.80.
FIRE DEPT. PROMOTION EXAMINATION
INSTRUCTION. For all ranks and bureaus. FIRE ENGINEERs', Quizz Book. By Capt.
750 Civil Service Ques. and Ans. and 1,000 W. Benedict Watts. Contains 225 Questions and
Specimen Questions. 300,000 words of instruc Answers, and includes descriptions of engines in
tion for Fire Engineer, Fire Lieutenant, Fire use in N. Y. Fire Dept. Pa., $0.50; mail $0.55.
Captain, Battalion Chief, Deputy Chief, Chief,
Fire Prevention Inspector, Fire Marshal and STEAM APPARATUS IN USE IN THE N. Y.
Asst. Fire Marshal, Fire Alarm Telegraph FIRE DEPT. A complete description. Paper,
Bureau, Official Instruction of N. Y. Fire Col $0.75; mail $0.80. Set of 7 Blue Prints, illus
lege and of the Boston Fire Dept., . Reports, trating descriptive matter in the book, $1.25
etc. (2d enlarged Ed.), cl., $3.50, mail $3.65; pa.,
extra; mail $1.40.
$3, mail $3.15. FIRE PREVENTION EXAMINATION IN
FIRE MAN CIVIL SERVICE EXAMINATION STRUCTION. By Samuel Rosenblum, B. S., C.
INSTRUCTION. A complete course of instruc E., ex-Chief Examiner, Bur. Fire Prevention,
tion for entrance examinations. 100,000 words of City of New York. 650 Ques. and Ans. for Civil
simple instruction. Answers to all N. Y. City Service Examinations for Fire Prevention In
previous Examination Questions—500 Questions spector, etc., with 200 Sections of New York
and Answers, covering Fire Fighting and Duties Fire Prevention Laws, Regulations, etc. 90,000
—Rules of N. Y. Fire Dept.—N. Y. City, , N. words. Pa., $2; mail $2.10.
Y. State and Federal Government—Arithmetic– FIRE PREVENTION AND PROTECTION
Memory Test—Reports–First Aid to Injured— (Hutson). For firemen, inspectors, engineers,
Medical and Physical Requirements. Pa., $1; architects, etc., covering protection from and
mail $1.10. extinguishment of fires. 750 pages (illus.). $4.25;
mail $4.40.*
FIRE DEPT. HYDRAULIC PROBLEMS AND
How To work THEM. By Charles Blum. FIRES AND FIRE FIGHTERS. By John Ken
simple Rules and Methods of Finding Square lon, Chief of New York Fire Dept. 434 pp.
Root; Friction Loss in Fire Hose, Water Mains, (illus.). Cl., $2.50; mail $2.65.”
Standpipes and Fittings; Nozzle Discharge; En ENGINES,
gine and Nozzle, Pressure; Water. Tower, Dis FIRE MODERN STEAM (Roper).
charge; Height of Streams, Pump Slip and Pump Descriptions and illustrations of all best types,
Displacement; Pump Capacity; Horse-power of and treats extensively on hydraulics. $3.50;
Fire Engines; Automatic Sprinkler Discharge; mail $3.70.*
Fire Hydrant Discharge; Volume: Siamese Con FIRE ENGINEERS’ VEST POCKET CALCU
nections: Underwriters' and Other Tables; Civil LATOR. It will tell you instantly discharge,
Service Examination Problems, etc.; 21 full-page friction loss, effective reach of streams, and
plates. Pa., $2; mail $2.10. enable you to estimate what pressure you need,
FIRE DEPT. MoToR APPARATUS. Official (It is not a book.) Waterproof; $1; mail $1.05."
Description and Equipment of every type of Mo Two PLATOON BRIEF FOR FIREMEN. By
tor Apparatus in use in the New York, Fire Soiomon Hecht, Editor, Civil Service Chronicle,
iSept..." with Official N.Y. Fire Dept. Instruc
and Capt. James D. Clifford, Pres. Firemen's
tion on Operation and Care; Questions and An Mut. Benevolent Assn. N. Y. City. It is the
swers on the Theory of Auto Engine Operation: most exhaustive argument ever presented on
Defects and Cost of Maintenance of . Motor the subject of why Firemen should be emanci
Pumping Engines, etc. Pa., $1; mail $1.10. pated from the slavery of 21 hours' duty a day
HAN D Book of INSTRUCTIONS FOR FIRE and be given a system of working in two shifts.
Single copies $0.50; mail $0.55. Ten copies or
LIEUTENANTS AND FIRE CAPTAINS. . Ex over, 10 cents a copy, postpaid.
clusively composed of official instruction of the

102
CIVIL SERVICE BOOKS.

POLICE DEPARTMENT. Government—Spelling—Specimen


Sheets—In the Examination
Examination
Room—Require
Pol-ICE DEPT. PROMOTION EXAMINA ments for Senior Clerk, N. Y. State; 2nd, 3rd
TION INSTRUCTION. 175,000 words of instruc and 4th Gr. Clerk, N. Y. City—Answers to ex
tion, 1,000 questions and answers (taking in annina Lion questions and specimen questions
a score of past examinations) for . Sergeant, -(2d Ed.). (NOTE: The first 96,000 words of
Lieutenant, Captain and Inspector; 91 Reports; this book are identical with “Federal First
. Y. Police College Instruction. Pa., $2.50; Grade Clerical Examination Instruction.” The
mail $2.60. remaining 34,000 words are devoted to examina
How TO GET ON THE POLICE FORCE. tions of a corresponding grade in other serv
A complete course of instruction for candidates ices.) 130,000 words. Pa., $1.50; mail #1.60.
for Pătrolman, Police Matron, and Police POST OFFICE DEPT. ExAMINATION IN
woman. 100,000 words of simple instruction in STRUCTION. By Solomon Hecht, Editor, Civil
cluding: 725 Questions and Answers, Arithmetic, Service Chronicle. A complete course of instruc
Memory Test, Official Instruction of the School tion, largely made up of past examination ques
of Recruits, Official Rules of N. Y. Police Dept., tions, difficult to obtain, with full explanations
Answers to all previous New York Examina and all processes shown, for Postal Clerk, Let
tion Questions, Reports, Definitions of Crime; ter Carrier, Railway Mail Clerk and Rural
City, State and Federal Government, First Aid Carrier, with information and aids for 4th Class
to Injured, Requirements; Form of Application. Postmaster and lnspector. 110,000 words.
(3d Ed.) Pa., $1; mail $1.10.
Covers Copying from, I'lain Copy–Letter Writ
Pol-ICE SERGEANTS’ CATECH ISM. An ing–Arithmetic—Spelling—Pennanship— Punc
swers to Questions asked at 6 New York, Ex tuation—1teading Addresses—Answers to Past
aminations held some years ago. Pa., $0.25; Examination Questions—Questions and Answers
mail 30.30. on Grammar—How to Fill Out P. O. Application
POLICE LIEUTENANTS’ AND CAPTAINS' Blank. (3d Ed.), Pa., $1; mail $1.10. (See also
CATECHISM. Answers to 8 sets of New York “Champion Spelling Book,” “150 Lessons in
Plain, Rapid Penmanship,' and “Sixty
Examination Questions, held some years ago, in Lessons
cluding Inspector. Pa., $0.25; mail $0.30. in Business Arithmetic.”)
THE POLICE PROMOTER. By Inspector POST OFFICE INSPECTOR. A course
Richard O'Connor, N. Y. Police Dept. A simple
digest of all New York laws
of instruction. Includes 8 separate pamphlets
lating to Police duty.
and ordinances re covering Duties, Civics, Arithmetic, Geography,
Spelling, Letter Writing, Penmanship and Gen
An invaluable pocket
companion for all ranks. (5th Ed.) Cl., $1.75; eral lnstructions, and 400 Ques. and Ans. On
mail $1.85. Postal laws and regulations. (Sold only by the
POLICE PRACTICE AND PROCEDURE. set.) Complete set $3; mail $3.25. In connec
(Insp. C. F. Cahalane, N. Y. City Police Lºept;) tion with this set it is advised to study “Federal
CôNTENTS: Discipline and Deportment; Physi First Grade Clerical Examination Instruction,”
cal Condition; Patrol; Fire Arms; Observation: as most of the subjects of this examination
Fires; Accidents; First, Aid: Arrests; Children; are the regular First Grade.
Handiing Demented Persons; Fugitives, frºm
justice; Evidence, Court Procedure, and Dis: THE U. S. POST OFFICE, by ex-Asst. Post
position of Property; Criminal Identificatiºn and master General Daniel C. Roper. Valuable for
Finger Prints; Traffic and Street. Conditions; the information of Postmasters and P. O. In
Public Hacks; Animals: Crime Classification and Spectors and all who seek promotion in the
Criminals; Crimes Classified; Manner in Whigh service. Covers every phase of the workings
Thieves Operate; Confidence Games; Public of the Department. 382 pages (illus.). Cl., $1.50;
Morals: Gambling; Prostitution; Enforcement mail #1.65."
of Sabbath Laws; Liquor Tax Law; Election MIDDLE GRADE CLERICAL Exam I NA
Laws; Investigations and Reports; Co-operatiºn TION INSTRUCTION. By Solomon Hecht,
with 'City Departments; Qrdinances; Permits. Editor, Civil Service Chronicle. Covers Arith
241 pp. ; iſſus. Cl., $1.50; mail $1.65.” Writing—Copying
Inetic—Letter from Plain
FINGER PRINT INSTRUCTOR, by Sergt. Copy-Pennanship—Punctuation—Spelling. Also
Fred. Kuhne, Bur. Crim. Identification, N. . Y: valuable chapters on Government and Grammar.
Police Dept. Based on Henry system; a full Answers to examination questions, and 500
and clear explanation. 160 pages; profusely specimen examination questions for 2nd gr.
illustrated. Cl. $2; mail $2.10.* Clerk, 3rd gr. Clerk, Junior Clerk, Temporary
Clerk, Record Clerk, Office Asst., Indexer, Cata
loguer, etc., U. S., N. Y. City, N. Y. State,
CLERICAL. New Jersey. (2d Ed.) 125,000 words. (NOTE:
The first six chapters of this book are identical
FEDERAL FIRST GRADE cLERICAL EX with “P. O. Dept. Examination Instruction.”
AMINATIon By Qr. Samuel The remaining chapters are for examinations
fºsenberg, INSTRUCTION.
formerly of a similar grade of difficulty in other services.)
of Examining Staff, of U. I’a., $1; mail #1.10.
S. Civil Service Comm. at Washington. . A com
largely
plete course of instruction. It is made
up of answers to past, examination questions,
MINOR CLERICAL EXAMINATION IN
difficult to obtain. with full explanations and
STRUCTION. A complete course of instruc
showing all processes. Arithmetic-Pennanship tion for the positions enumerated. Covers Arith
Report Writing—Copying , and Correcting
metic-Letter Writing—Spelling—Copying from
Manuscript (Rough praft)–Condensation—ºe Plain Copy–Penmanship—Ques. and Ans. on
Grammar–N. Y. City, N. Y. State and U. S.
ography and Civil Government-Spelling-An Government—Answers to examination questions
swers to Past Examination Questions-Specimen and specimen questions—Requirements for 1st
Examination. Sheets—in the Examination Room Gr. Clerk (Office Boy) N. Y. City; Federal Sub
frequirements for 1st Gr. Clerk, Department Clerical (Messenger, Watchman and Skilled
Clerk. Inspector, Stenographer, and is for all ex Laborer), Messenger, Boy, Messenger, Page, etc.,
aminations for which the Federal First Grade N. Y. City, N. Y. State, New Jersey, Chicago
Gierical Educational test is given-Answers to and Federal–90,000 words. (2d Ed.) Pa., $0.75;
200 Exclusive Examination Questions-9 plates— mail ș0.85.
72d Ed.)—100,000 words. Pa., $1.50;, mail $1.50.
(See also “Ques, and Ans. ...in Geography, COMPLETE COURSE IN THE U. S. CIVIL
*Champion Spelling Book,” “150. Lessons in SERVICE. For 1st, 2nd and 3rd Grade Federal
Plain. itapid I'enmanship,” and “Sixty Lessons Clerical Examinations. Contains a large num

jº.
in Business Arithmetic.”) ber of problems and other matter appropriate
to each branch of these examinations, and in
SENIOR GRADE CLERICAL EXAMINA cludes: 1st, 2nd and 3rd Grade Arithmetic: 1st,
TION INSTRUCTION. By Dr. Samuel Rosen 2nd and 3rd Grade Copying from Plain
bérg, formerly of the Examining Staff of U. Copy; Heading Addresses; Composition and
S. Givil Service Comm. at Washington. Arith Letter Writing: Rough Draft; Conversion of
metic Penmanship—Report Writing (Condensa Currency; Geography.

º's".
Cl., $1.25; mail $1.35.”
tion)—Copying and, Correcting Manusºil (NOTE: The title is somewhat misleading, as it
(Rough Draft)—U. S. S. Civil is not a complete course. However, it contains
Government—N. Y. City and N. Y. State much valuable material for practice.)

103
CIVIL SERVICE BOOKS.

fº.
CIVIL SERVICE MANUAL No. 1–ARITH INDEXING
- AND FILING (Hudders). Ex
M ETIC. Covers every phase of Arithmetic plains

-
basi systems. Cl., $3.00;

*
used at Civil Service Examinations. Measure mail $3.15% 292 pp.

c
ments—Number Reading—Addition—Reduction
--Subtraction—Multiplication —Division —Com F1 LING SYSTEMS (Cope). Their principles

ºp
and application

to
plex. Fractions—Common Fractions Changed to modern office
Decimals—Measurements —Percentage —com 189 pages. Cl., $1; mail $1.10.*
mercial Paper—Conversion of Currency—Square !NDEX SYSTEM (Byles). Its prin
Includes a large
Root–Miscellaneous Problems.
Fºer
uses, operations and componen parts
Hºt

30
of questions and answers. Cl., $1; mail 108 pp.; illus. CI., $0.50; mail
ACC.90NTING,
By Hermat: MUNICIPAL HAND Book


CIVIL SERVICE MANUAL NO. 2–ENGLISH of. Metz, ex-Comptroller, City

A.
AND ALLIED SUBJECTS. Punctuation–Dic scientific system

of
9ſ. New York.

...
account".

A
tation — Penmanship — Synonyms — Homo and, reporting for municipal utilities.”:1

in
8
nyms—Correct. Use of Words—Letter Writing CI., $2.25; mail $2.40."

85
pp., with pp. index.
and Composition—Plain Copy—Rough Draft– ACSQUNTING, MUNICIPAL. By Eggles
#

D.
C.
Reading Addresses — Abbreviations — Stenog

º
ton, C.P.A., formerly Finance Dept., Of

of
raphy–Typewriting—Report Writing—Copying New York.
and Correcting Manuscript. Cl., $1; mail #1.10. Gives method for handling every

of
kind financial transaction Clty. 4:3t;

of

a
CIVIL SERVICE MANUAL NO. 3–GEOG pp. Lea., $4.00; mail $4.20.*
RAPHY, U. S. CIVIL GOVERNMENT, SPELL
ING. Geography and U. S. Trial Ex 99RPQRATION ACCOUNTS AND vouch - ER
-
SYSTEM 160 pp.; Cl.,

50
(Griffith). pp.; illus.
amination Papers—Spelling—Civil Government.
(ºººººººººº,
$1.00; mail
Cl., $1; mail $1.10.
STATISTICAL ELEME METHOD,
STENOGRAPHER AND TYP1ST. Manual o

of
preparation for New York, New Jersey and U. S. FLEMENTs
Civil Service Examinations, by Chas. L. Frank, STATISTICS, INTRODUCTION TO THE-
Instructor in Stenography, New York Univer STUDY OF (Yule). 382 pp., 53 figs.
igs. Cl., $3.50;

-

,
)
sity. Requirements, specimen questions, mail $3.70.*
...

etc.
Pa., $0.60; mail $0.70. (In connection with this ..FINANCE, FIRST LEssons, IN. By

A.
F.
book study “Federal First Grade Clerical Ex Qleveland, ex-Director Bur. Municipal Research,
amination Instruction,” as most of the Stenog N. City. Cl., $1.25; mail $1.35.”

Y.
tº ºt)
rapher subjects are regular Federal First Grade
INVESTIGATING AN INDUSTRY (Kent).

A
subjects.) diagnosis
Scientific of the diseases of IIlanage
QUESTIONS AND ANSWERS IN GEOG ment. 126 pp. Cl., $1.00; mail $1.10.*
RAPHY. By Emil Gutman, Specially prepared QVER HEAD EXPENSE AND PERCENTAGE
for Federal First Grade Clerical and Railway METHODS (Baillet). 128 pp. Ul.,
24 tables.

.;
Mail Examinations. Covers this troublesome ables.
subject completely. Pa., $0.35; mail $0.40.
150 LESSONS IN PLAIN, RAPID PEN MAN DEPRECIATION, PRINcIPLES OF (Saliers).
SHIP. Pa., $0.25; mail $0.30.* From the accountant's standpoint. 200 pp. Half
lea., $2.50; mail $2.65.”
TRUE COSTS AND RELATIVELY TRUE
BOOKKEEPING, ACCOUNTANCY, SEL-L-ING PRICES, THE PREDETERMINA.
-

TION Q.F. (Parkhurst). Includes stores Keep


FILING, INDEXING, FINANCE. ing and Perpetual Inventory.
Cl., $1.25; mail $1.35.”
104 pp.; illus.

BOOKKEEPER AND ACCOUNTANT CIVIL


SERVICE EXAMINATION INSTRUCTION. By
Mark Wolff, B.C.S., C.P.A., Chief Accountant ENGLISH LANGUAGE.
Dept. Water Supply, Gas and Electricity, an Book, IN writing
Instructor Bookkeeping and Accounting, Eve: ENGLISH, FIRST
in

(Lewis). Presents an understandable way


in

ning High Schools, City 85,000


of

New York. principles writing that high school


words. Includes Ans. to 250 exam. Ques, and 500
of

the a
yºuth may, reasonably be expected employ.

to
specimen ques. Information regarding examina
293 pp. Cl., $0.80; mail $0.90.*
tions for Bookkeeper and Accountant positions
N. City, N. Y. State, New Jersey, Illinois ENGLISH, GRAMMAR SIMPLIFIED (Fern
Y.
in

and U. Bookkeeper, N. Y. City ald). For those whose knowledge the parts
S.

Services.
of

Service: Answers to exam. ques. and specimen imperfect. 282 pp. $0.75; mail
ºech
is
§

ques. for 2d, 3d, 4th and 5th Grades. Book


keeper, N. Y. State Service: Answers to exam. STYLE BOOK OF BUSINESS ENGLISH.
§e.
ues. and specimen ques, for 4th, 5th and 6th
Bookkeeper, U. Service: Answers to
Includes card indexing, letter filing and duties
Private Secretary. 238 pages. Cl., $0.85;
S.

of
a

exam, ques, and specimen ques. Bookkeeper mail $0.95.”


and Junior Bookkeeper, New Jersey and Illinois BUSINESS ENGLISH AND CORRESPOND
Services: Answers to exam. ques. and specimen ENCE (Barrett). 216 pages. Cl., $1; mail
ques. Accountant, N. City Service: Answers
Y.

$1.15.”
to exam. Ques. and specimen ques. for 4th and
5th Grades. Accountant, N. Y. State Service: MANUSCRIPT PRE PARATION AND PROOF
Specimen ques, for Accountant and Junior Ac READING (Esenwein). Clear directions on
prepare manuscript, punctuate, capital
to

countant, Public Service Com’n, and Expert how


Accountant, Dept. Efficiency and Economy. c ize, hyphenate, spell and read proof. $1.60;
A

countant, New Jersey and Illinois Services: mail $1.70.*


Specimen ques. for Accountant and Statistician, MANUSCRIPTS-PREPARATION OF, FOR
Public Utilities Conn'n, and Accountant, Rank THE PRINTER (Vizetelly). Preparation of
1.

Practice ques. and ans. on Bookkeeping, Com copy, correcting proofs and suggestions on sub
mercial Practice and Terms. Board of Educa mitting manuscripts for publication. 148 pp.
Cl. $0.75; mail $0.85.”
tion specimen exam. ques. for License Teach
to

Bookkeeping, N. Y. City High Schools and Eve PROOFREADING PUNCTUATION


A

D
N

ning Schools. Ans. N. Y. State exam. ques. (Smith). textbook for schools and aid to
to

for Certified Public Accountant. Digest of Uni Printers and Proofreaders. 187 pp.; illus. $1.00;
form Accounting Systems Prescribed by the mail $1.10.*
Public Service Com’n, 1st Dist., N. Y. Pa., PUNCTUATION, HANDBOOK OF (Bigelow).
$1.50; mail $1.60. For Printers, Authors, Teachers and Scholars.
BOOKKEE PING, SELF-TAUGHT (Goodwin). $0.50; mail $0.55.”
Cl., $1.00; mail $1.13.” PUNCTUATION. Simple lessons. Pa., $0.25;
Book KEEPING, PRACTICAL (Griffith). mail $0.28.*
A

working handbook of elementary bookkeeping CHAMPION SPELLING BOOK. Words graded


and approved methods of modern accounting. according difficulty. For Federal Sub-Clerical
to

153 pp.; illus. Cl., $1.50; mail $1.65.” study pages 1-110; for P. O. Clerk-Carrier, 85
Book KEEPING SIMPLIFIED (Ney). Cl., 198; for Fed. 1st Gr, Clerk, 123-238. 238 pages.
$1.00; mail $1.10." Cl., $0.25; mail $0.30."

104
CIVIL SERVICE BOOKS.

BUSINESS DICTATION BOOK AND LEGAL PRISON KEEPER AND POLICE DOORMAN
FORMS, Twent ETH CENTURY (Pitman).
Boards, $0.75; mail $0.85."
RULES AND TEXAMINATION FABERS"HoH.
294 pp. examination questions for N. Y. City,
Specimen
State, New Jersey and Chicago, for

N.
1 C T I O N A R Y,

Y.
COMMERCIAL D VEST
POCKET_ (Pitman). 384 pp. Boards, $0.25; Keeper, Guard, Officer, Watchman, Tibéputy

ºp
Warden and Police Doorman; Rules"

.
Y.

of
mail $0.28.*
§§

-
and N. State Prisons. Pa., $0.50; maii

Y.
MATHEMATICS. JANITOR- ENGINEER, INSTRUCTION FOR.
Answers 100 Civil Service Examination Ques.

to
SIXTY LESSONS IN BUSINESS ARITH ME tions, and many specimen questions for Janitor
TIC (Mull.). Suitable for all clerical grades in Engineer, Janitor, Janitor Steam Heating, Sta
the civil service. Contents: Numbers—Notation tionary Engineer and Stationary Engineman;
Numeration—Addition—Subtraction —Mul Janitors, etc.

of
Board Education Instruction

to
and
tiplication—Division—Divisors —Multiples—Can Pa., $0.75; mail $0.80.
cellation—Fraction—Decimals—Aliouot
Measures—Denominate Numbers —Time —Per
Parts— MALE ATTENDANT, WATCH MAN, MES
SENGER AND BRIDGE ENESEHT N.T.Y.

ºr

in
centage—Profit and Loss—Commission and City. Examination instruction.
Brokerage—Trade 150 Ques. and
Discount—Insurance—Taxes Ans. Pa., $0.50; mail $0.55.
—Duties or Customs—Stocks and Bonds—Do
mestic Exchange—Foreign Exchange—Bills— ATTEN DANT—MALE AND FEMALE–SUP
Common Interest—Exact Interest—Problems in PLEMENT. 150 additional Ques. and Ans.; also
Interest—Bank Discount—Partial Payments— Regulations for Baths, Beaches, Parks' and
Equation of Accounts—Settlement of Accounts— Recreation Piers. Pa., $0.50; maii 50.55.
Current Partnership—Ratio and Proportion— TELEPHONE OPERATOR ExAMINATION
Metric System. 90 pp. $0.50; mail $0.60.* INSTRUCTION. Ans. Ques, asked at N.

to

6
MATH E MATICS, PRACTICAL City tests, with specimen New Jersey exam.
Y.
(Hobbs).
Fundamental Processes, Fractions, Denominate questions. Official instruction N. Y. Tele.

of
Numbers, Advanced Processes, Practical Ge phone Co. for operation Private Branch Ex

of
ometry, Logarithms, Curve Plotting. 172 pp.; change switchboards. (Illus.) $0.50; mail 30.55.
70 illus. Cl., $1.00; mail $1.15.” TELEPHONE TROUBLES, AND HOW TO
MATHEMATICS, VOCATIONAL (Dooley). 341 FIND THEM. Includes line, cable, instrument
pp.; illus. Cl., $1.00; mail $1.15.” and switchboard, troubles on both the magneto
and common battery systems. (Illus.) Pa.,
(See also “Civil Service Manual No. 1" under $0.25; mail $0.30.
Clerical books.)
INSPECTOR, INVESTIGATOR,
ATTENDANT AND CUSTODIAN. CLAIM ADJUSTER.
COURT ATTEN DANT AND COURT CLERK
Exam INATION INSTRUCTION (1917 edition). (NOTE: For Civil Engineering Inspec
By Solomon , Hecht, Editor, Civil Service
.
...

Chronicle, and Henry M. Schiffer. Contains tion, see “Civil Engineering.”)


703 Ques. and Ans., including N. Y., State LICENSE INSPECTOR EXAMINATION IN
6.

and County examinations and N. Y. City ex STRUCTION. covering


7

aminations; General and Miscellaneous instruc 100 Ques, and Ans.


tion on Law and Courts; Organization and Ju Previous Examinations, Duties, Laws, Reports,
Forms, etc., and all N. City Laws and Or.
Y.

risdiction of N. Y. City, County, N. Y. State and


Federal Courts, and Duties of Court Attendants
and Court Clerks; Juries and Definitions of
dinances
Pa., $0.50; mail *...*s';
Licenses. 50,000 words.

Jury Terms; Civil Procedure and Definitions of TAXICAB INSPECTOR EXAMINATION IN


Terms; Criminal Procedure and Definitions of STRUCTION. Ques. and Ans. on Duties of
Terms; Warrants, Writs, Orders, Processes and Taxicab Inspectors, Descriptions and Illustra
tions the various types Meters
in
of

of

Service of Papers. Also, 451 Specimen use


in

Civil City, Rates Fare, Forms Application


Service Examination Questions for Asst. Clerk,
of

of
Y.
N.

Deputy Clerk, Special Deputy Clerk and Asst. for Licenses, Ordinances, Rules, etc. Illus.
(Illus.)

º:
Special Deputy Clerk, Court of Claims, Supreme Pa., $1; mail $1.10.
Court and Court of General Sessions in the DOCKMASTER INSTRUCTION FOR CIVIL
N. Y. State and Services; for Clerk, pre
SERVICE EXAMINATIons. Answers
to

Asst. Clerk and Deputy Clerk, Court of Special vious Examination Questions, Official Rules
Sessions, Magistrates' Court, City Court and
of N. Y. Dock Dept., Instructions Dock
to

Municipal Court, N. Y. City; for Court. Attend masters, Reports,


ant, Court Crier, Sergeant-at-Arms, Clerk and 100 Practice Questions, etc.
,

Pa., $0.50; mail $0.55.


District Court Clerk in the New Jersey Service.
Information for candidates for Court Attendant METER AND WATER CONSUMPTION IN
in N. Y. City, N. Y. State and County Services. SPECTORS' MANUAL. With Ques. and Ans.
80,000 words. Pa., $2.00; mail $2.10. (Illus.) Boards, $0.50; mail $0.60.
COURT ATTEN DANT EXAMINATION IN
STRUCTION (1914 edition). Contains answers HOW TO REDUCE WATER TAxES: THE
to sets of N. Y. City and N. Y. State exam INSPECTOR:... By Jacob Klein, Chief Inspector,
N. Y. City Water Dept. (Suitable for Water
9

ination questions, with 150 Practice Ques. and Meter Inspectors.) Includes Examination Ques
Ans. and 400 Specimen examination questions tions and Answers. Pa., $0.65; mail $0.70.
in the N. Y. City, N. Y. State and New Jersey
services for Asst. Court Clerk, Deputy Clerk, GAS AND GAS METERS, PRACTICAL TEST
Spl. Asst. Deputy Clerk, District Court Clerk, ING OF. By H. Stone, Chief Insp. Gas,
C.

of

Court Crier and Sergeant-at-Arms. 45,000 words.


.

Pub. Service Comn., 2nd Dist., N. Y. 337 pp.;


Pa., $1; mail $1.10. 51 figs. Cl., $3.25; mail $3.40.*
COURT ATTEN DANT MANUAL (Rourke CLAIMS: FIXING THEIR VALUES.
A

Elwood). Gives in nutshell Duties, Legal


a

manual by G. W. Johnson.
F.

F.

Knowledge. Forms, Court Procedure, etc. Pa., Deiser and


A ,

ex-Asst. Gen]... Claim gent, Phila. Rapid


...

...

$1.00; mail $1.05.


Transit Co. 165 pp. $2.00; mail. $2.10.*
PRISON KEEPER EXAMINATION IN
STRUCTION. Answers to Examination Ques ADJUSTER'S MANUAL (Harbaugh). For
settlement, accident and health claims. Lea.
of

tions and 173 Practice Questions and Answers


on Rules of the N. Y. City Prison and Work $2.50; mail $2.65.”
house, and 300 Specimen Examination Questions
for Keeper. Hall Keeper, Asst. Principal Keeper, LIABILITY INVESTIGATOR'S HAND Book
Principal Keeper, Guard, Female Guard, Woman (Dilg.). For accident cases, by the N. Y.T.Chief
Investigator prominent liability company.
of

Officer and Matron, N. Y. City, N. Y. State and


a

Lea., $1.50; mail $1.60.*


New Jersey. This book supplement
is

to
a

“Rules and Flxamination Papers for Prison SPECIAL AGENTS’ AND... ADJUSTERs'
Keeper and Police Doorman.” Paper, $0.75; mail AND BOOK (Steeb. (For Fire Insurance.)
H

).

$0 Lea., $1.50; mail $1.63.”

105
CIVIL SERVICE BOOKS.

FIRE INSURANCE AGENTS’ HANDBOOK MENTAL DEFICIENCY (Tredgold). Covers


(Weed). Theory and practice. $1.50; mail $1.63.” incidence, pathology, mental and physical char
acteristics, social relationship, diagnosis, prog
FACTORY INSPECTOR. See “The Modern nosis and, treatment. (2d Ed.) 511 pp.; illus.
Factory,” under Social books. $4.25; mail $4.45.”
NSTITUTIONS, COTTAGE AND CONGRE

|
GATE (Hart). 136 pp.: illus. $1.00; mail $1.10.*
SOCIAL AND REFORMATORY. INSTITUTIONs, cost. Accounting For
(Cole). For hospitals, hotels, municipal institu
PROBATION AND PAROLE OFFICER EX tions, 248 pp.
etc. $2.50; mail $2.65.”
AMINATION INSTRUCTION (1917 edition). By THE MODERN FACTORY. By Dr. Geo. M.
James J. Flynn, Chief Parole Officer, N. Y. City Price. Covers safety, sanitation and welfare,
Parole Commission; Edwin J. Cooley, Chief Pro including factory inspection and legislation and
bation Officer, City Magistrates' Courts; Bernard fire drill instruction. 574 pages (illus.). Cloth,
J. Fagan, Chief Probation Officer, Children's $4; mail $4.15.”
Court; Daniel J. White and Mrs. Anna W.
Hochfelder, B.A., LL.B., D.J., Probation Offi
cers. Children's Court: "Patrick J. Sheily. Pro SANITARY AND MEDICAL.
bation Officer, City Magistrates' Courts; Wm.
B. Allis, Probation Officer, Court of Special Ses
sions; Julius Hochfelder, LL.M., TEN EMENT HOUSE AND SANITARY IN
Attendance SPECTOR EXAMINATION INSTRUCTION. 310
Officer, Board of Education; Charles L. Frank,
B.S., LL.B.; and Solomon Hecht, Editor, Civil Ques. and Ans. for Inspector and Chief Inspec
tor of Tenements and Lay Sanitary Inspector,

.
Service Chronicle. for N. City, New Jersey and Chicago. His

Y.
A practical handbook for civil service candi tory of N. Y. Tenement House Dept., and Ad
dates, for Probation Officers in the service, and
Candidates by I\r., George M. Price.
vice Digest

to

A
for Social Workers generally. 165,000 words, 1916 of New York Laws and Ordinances
including 700 Questions and Answers, and 400 governing the regulation of tenements. Amend
Specimen Questions, etc. ments to the Tenement House Law from 1912
Covers N. Y. laws relating to Probation and to 1916. Practice Ques. and Ans. on Laws and
Parole; Duties of Probation and Parole Officers;
Duties by experts of the Tenement House Dept.
Probation and Parole methods in practice in Answers to Civil Service Examination Ques
- . City; Organization and Jurisdiction of tions, and Specimen Questions. Report Writing.
N. Y. City, County and State Courts; Criminal Pa., $1.00; mail $1.10.
Procedure and Definitions of Crimes; 210 Topics
discussed at State Probation Commission Con MILK AND FOOD INSPECTOR. By Charles
ferences; Probation Rules of the Children's L. Frank, B.S., LL.B. 359 Ques. and Ans., Re
orts, etc., on milk, groceries, meats, etc., for
Court of N. Y. City; Information for candidates; ivil Service Examinations, and 221 Rules of
Investigation Work and Report Writing, in
cluding explanation and use of the various N. Y. City Health Dept. 50,000 words. Pa.,
Forms in use; Answers to 10 sets of Past Ex
$1; mail $1.10.
amination Questions and many Specimen Ques FOOD INSPECTORS' HANDBOOK (Vacher).
tions for hief Probation Officer, Probation practical guide for health officers, meat in
A

Officer, Chief I’arole Officer, Parole Officer, spectors, army officers, etc. Covers all branches.
Parole Officer, and Parole Agent in the 340 pp.; 98 illus. Cl., $3.00; mail $3.15."
Spegal
. Y. City, N. Y. State and New Jer
§"É. FOOD INSPECTION (MacEwen). practical

A
sey and Illinois Services. Second dition. Pa., handbook. 264 pp.; illus. Cl., $2.50; mail $2.65.”
$3.00; mail $3.15. FOODS, PURE: THEIR ADULTERATION,
350 QUES. & ANS. FOR PROBATION OFFI NUTRITIVE VALUE AND COST (Olsen). 210
CER. By Chas. L. Frank, B.S., LL.B., (1913 pp., illus. Cl., $0.80; mail $0.90.*
edition). 65,000 words. Pa., $1 (reduced from FOODS AND THEIR ADULTERATIONS. By
$2); mail $1.10. Harvey W.

ex-Chief Chemist, U. Dept.

S.
PENAL AND REForMATORY INSTITU: Agriculture. 641 pp.; illus. Cl., $4.00; mail
(Sylvester). 345 pp.; illus. $2.50; mail $4.20.* -
195's
$2.70. MEAT INSPECTORS, GUIDE FOR (Wilcox).
ATTEN DANCE OFFICER EXAMINATION For Veterinary and Meat Inspectors. 258 pp.;
INSTRUCTION (TRUANT OFFICER). By 158 illus. Cl., $2.50; mail $2.60.*
Julius Hochfelder, LL.M., Attendance Officer, MILK AND MILK TESTING, QUESTIONS
Board of Education, City of New York. 500 AND ANSWERS ON (Publow). 100 pp.; illus.
Ques, Ans., Reports, Forms. Specimen
and Ex Cl., $0.50; mail $0.60.*
amination Questions, etc. Pa., $2; mail $2.10.
NURSE INSTRUCTION FOR CIVIL SERV
CHARITY INSPECTOR AND SOCIAL IN ICE EXAMINATIONS. Ans. to Examination
VESTIGATOR ExAMINATION INSTRUCTION. Questions for Nurse; Specimen Questions for
By soiomon Hecht, Editor, Civil Service Chron Nurse; Trained, Hospital, Field, Visiting, Tu
icie, and Julius Hochfelder, LL.M. For candi School; Nurse's Assistant;
Social In
berculosis. Head
dates for Institutional Inspector, Nurse, Supervising Nurse, Assistant Superin
vestigator, Charity Application Investigator. In City,
spector State Board of . Charities, etc. , Con tendent and Superintendent, for
.

N. Y. State, New Jersey, Chicago and Federal.


tains 1500 Official Inspection Questions and 1000 Pa., $0.50; mail $0.55.
Questions and Answers covering all New York
State Laws relating to charities, answers to STATE BOARD QUES. ANS. FOR NURSES
&

past Civil service. Examinations, recent refºrms Foote). Actual questions by 31, State exam
Cl.,
(

in the practice of public charity, in New York ination boards, with answers. 365 pages.
City, déscription of a model, orphanage, exist $2.50; mail $2.65.”
ing defects in private charitable institutions, CARE AND FEED ING OF CHILDREN
etc. 150,000 words. Pa., $3; mail $3.15 (Holt). Contains about 800 Ques. and Ans. for
CHARITY, PRAcTICE OF. By Edward T. Nurses. 197 pages. Cl., $0.85; mail $0.95."
'y.

IDevine, N. School of Philanthropy. Cl., $0.75; NURSE, THE JUNIOR (Hrowne). guide
A

mail $0.85.* for the earlier months of training, giving prac


PHILANTHROPY, MODERN. By Wm. H. 208 pp. Cl., $1.50; mail $1.63.”

§º
tical rudiments.
Allen, Director, Bur. of Municipal Research, NURSING, QUIZ BOOK FOR TEACHERS
City. 456 pp.; illus. $1.50; mail $1.70." AND STUDENTS (Pope). great help for
A

DELINQUENT chil LD AND THE HOME those studying for examinations. Cl., $2.00;
(Breckenridge and Abbott). 355 pp. $2.00; mail mail $2.15.”
NURSEs, TEXTBOOK FOR TRAINING
A

$2.15.”
CHILD HELPING ORGANIZATIONS, ELE SCHOOLS FOR. By Dr. M. Wise, late Pres.
P.

MENTS of REcoRD KEEPING FOR (Ralph). N. Y. State Lunacy Comn.


,,

vols., each $1.25,


-

iº, pp. $1.50; mail $1.60."


$2.50; mail $2.70."
DEPENDENT, DEFECTIVE AND DELIN ETETICS, HOME (Fish). Course designed
D

for classes supervised by Amer. Red Cross Nurs


|

pp. $1.50;
QUENT cuAsses (Henderson). 397 ing Service. 127 pp.; illus. $1.00; mail $1.10.*
mail $1.63.”

106
CIVIL SERVICE BOOKS.
DIETETICS, ESSENTIALS IN (Pope). A text INSPECTOR OF Pue Lic works. By Walter
Engr., City N.Y. 200 Answers

of
book for Nurses and a
practical dietary house Seely, Asst.
Cl., $1.25; mail $1.38.* Civil Service Exam. Ques, and 400 Specimen

to
hold guide.
OFFICERS’ MANUAL (McNutt). Exam. Ques. Pa., $0.50; mail $0.55.
HEALTH
INSPECTOR OF water suppl-Y.
By Walter
Cl., $3.00; mail #3.15.”
650 pp. Seely, Asst. Engr., City N. Y. 100 Answers

of
HEALTH AND MEDICAL INSPECTION OF Ques, and 200 Specimen
By Dr. W. S. Cornell, Civil Service Exam.


SCHOOL CHILDREN. Schools Exam. Ques. Pa., $0.50; mail $0.55.
Director, Medical Inspection of Public By Wm.
#. $14 pp.; 200 engravings. Cl., $3.00; mail


PUBLIC WORKS INSPECTION. New York. Spe.
Goldsmith, Asst. Engr., City

of
3.20.
INSPECTION OF SCHOOLS cially prepared for Civil Service Examinations.
MEDICAL Cl., $1.50; mail $1.63." Covers Duties, Methods of Reports,
(Gulick). 224 pp.; illus. Mathematics, Specifications, ormulas. Pa.,
MEDICAL AND SANITARY INSPECTION OF $1.50; mail $1.55.
SCHOOLS. By Dr. . Newmayer, in charge
RQPMAN (JUNIOR ENGINEER) INSTRUC
S.
Hygiene, Bur. of Health, Phila.
of Div. of Child
$2.50; mail $2.65.” TION FOR CIVIL SERVICE ExAMINATIONs.
High
l.,

318 pp.; 85 illus.


(Hornsby). By M. Perlman, Asst. Engr. N. Y. State
HOSPITAL, THE MODERN way Dept. 100 N. Y. State Civil

to
Answers
Covers construction, conduct, duties of officers 350 Specimen

.
Questions,
training school, etc. 644 pp.; Service Examination.City, State, New Jer

Y.
Questions for N. N.

Y.
and staff, nurses'
Cl., $7.00; mail $7.25.” Duties, survey
207 illus. Sey and Chicago, etc. Includes
R. Curves, Concrete Construction,Roa S,
Hy
HOSPITAL ACCOUNTING AND STATISTfGS ing,

R.
(Thorne). $1.25; mail $1.35.” draulics, Mechanics, Structures, Derricks,
Trigonome:
Piles,
By Dr. Geo. Vol. Computation, Materials,
HAN BOOK OF SANITATION. Pa., $1.25; mail
§§ Logarithms, Tables, etc.
D

edition. Covers sanitary science,

º
M. Price. 3rd
sanitary inspection; tene ...So.
sanitary practice and
ment houses; factories; foods; disinfection; AXEMAN. Four Civil Service Examination Pa., $0.60;
questions. papers, Questions and Answers.
specimen civil service examination

2
343 pages. Cloth, $1.50; mail $1.65.” mail $0.65.
BUILDINGS (Ger CHAINMAN AND RODMAN. Three Civil
SANITATION OF PUBLIC Churches, and papers, 102 Questions and
hard). Hospitals, Theatres Service Examination mail,
Schools. 262 pp. Cl., $1.50; mail $1.65.” Answers. Pa., $0.60; $0.65.
REFUSE, MUNICIPAL–DISPOSAL OF (Par LEVELER ENGINEER). Six Civil
186 pp.; 73 figs. Cl., $2.00; mail $2.15." Questions and

88
sons). Service Examination papers,
st REET clean ING, Mope RN, METHOD
By Geo. A. Soper, Chief Engr. Metropolitan
Answers. Pa., $0.60; mail $0.65.
OF. comparison of TRANSITMAN AND COMPUTER. Six Civil
Questions and

67
of Y. Service Examination papers,
A

Sewerage Comn.
200 pp.; 100 Pa.,
.

American and
#Tºº
illus. $3.00; mail $3.15.”
methods.
Answers. Appendix.
$0.60; mail $0.65.
seful formulas.
PLUMBING, SANITARY, QUES. AND $1.00; ANS. volume

in
THE SURVEYOR. Combines one
Vol. and Rod
(Starbuck). Drainage and Venting, the four books, “Axeman,” “Chainman
I,

mail $1.10.*II, Hot Water Supply and Circula man,’’ ‘‘Leveler,” “Transitman and Computer";
tion, Vol. $1.00; mail $1.10.* Steam and Cl., $2.00; mail $2.20.
Hot Water Heating, $1.00; mail $1.10.* GENERAL APPENDICES. Information U.and S.,

of
the civil service
in

examination papers
State, Boston, New Orleans, etc. 1,000
LEGAL.
Y.

N. Cl., $2;
specimen ques. mail $2.20.
LAW CLERK, 150 QUESTIONS AND AN DRAFTSMAN AND DRAFTSMAN's HELPER.
swers For civil SERVICE EXAMINA Eighteen Civil Service Examination papers. 120
L

Questions and Answers. Pa.,


in

Examination Questions $1; mail #1.10.


to

TIONS.CityAnswers
N. Y. and N. Y. State; Practice Questions THE INSPECTOR. Building, Mason and
and Answers; Specimen Examination Questions; Carpentry, Steel, Regulating, Grading and Pav
Definitions; Arithmetic; Letter Writing; Spell Civil Service Examination papers, 222
11

ing.
,
,

ing. Pa., $0.75; mail $0.85.


Questions and Answers. Pa.,
,

$0.75; mail $0.85.


PRocess SERVER, QUESTIONS AND AN: ASSISTANT ENGINEER RAPID TRANSIT
Answers to Questions asked at
5

SVVERS. Practice COMMISSION. 10 Civil Service Examination


N. Y. Civil Service Examinations; Pa., $0.,
of

Questions and Answers.


90

Extracts from the N. papers,


Questions and Answers; Pa., mail $0.85.
Y. Civil Code, etc. $0.50; mail $0.55 Aque
MAN ASSISTANT ENGINEER—GENERAL.
NOTARIES" AND COMMISSIONERS’
19

Appointment, duct, Docks, Sewers and Highways. Civil


to

UAL (Snyder). Particulars asunder N. Y. and Service Examination papers, 218 Questions and
Rights, Powers and Duties Answers. Pa., $1.25; mail $1.35.
$1.90; Pa., $1.50;
Federal laws. Cl., $1.75; mail CIVIL ENGINEERING EXAMINATION
mail $1.63." papers and 1100
COMMERCIAL LAW (Chamberlain). 336
pp. QUESTIONS. 60 specimen the serv.
in

Civil Service Examination Questións


York, Buffalo,
$2.00; mail $2.15." the United States, New
of

CORPORATION LAW (Abbott). Private Cor ices


Boston, New Orleans, etc. Cl., $2; mail $2.20.
porations, Public Service Corporations,Legisla Inter DRAWING, How TO READ (Getty). Me
64A

state Commerce Law and pp. Anti-Trust


62

Cl., $3.00; mail chanical and Architectural.


pp.; illus.
tion and Litigation. 473 Cl., $1.00; mail $1.10.*
$3.25.” DRAwl NG, SELF
L
A

For
T
E

PAR LIAMENTARY PROCEDURE (Felt).series


H
T

R
U
A

C
R
C

TAUGHT., (Hodgson). 336 pp.; 250 illus. TCI., -


|

Clubs, Societies, General Meetings, etc.


A

Cl., $0.60; mail $2.00; mail $2.15.”


of practical lessons. 200 pp.
ARCHITECTURAL DRAwl NG AND LET
$0.70.* practical in
of

By R. TERING (Bourne). manual


A

TAx

ſº
INCOME PROCEDURE (1917). Cl., $1.50;
H. Montgomery, C.P.A. Full directions for pre struction. .192 pp.; 100 drawings.
Corporation mail $1.65.”
paring returns, with comments on
Tax and Munitions Tax. 461
pp. Cl., ARCHITECTURE, EASY STEPs To (Hodg
CI.,
$2.50; mail $2.65.” son). For, home, study. 350 pp.; 230 illus.
$1.50; mail $1.63.”
ARCHITECTURE, EASY IN
N
o
S
S
L
E
S

CIVIL ENGINEERING AND (Mitchell). 150 engravings. $0.50; mail $0.58.*

ARCHITECTURAL. BUILDING SUPERINTENDENCE for Brick,


Stone Buildings (Nichols). 224 pp.;
Frame andCl.,
INSPECTOR OF BUILDINGS, MASONRY
By Walter Seely, Asst. 250 illus. $1.50; mail $1.65.”
AND Foun DATIONS. SUPERINT ENDENCE
Civil Ser BUILDING
ii
O
R
F
to

Engr., City N. Y. 200 Answers


of

84

and 350 Specimen STEEL, STRUCTURES (Belden). 107 pp.;


vice Examination Questions
Pa., $0.50; mail $0.55. lus. Cl.,
$1.00; mail $1.15.”
Examination Questions.

107
CIVIL SERVICE BOOKS.

ºº
STEEL CONSTRUCTION, P R A C T 1C A L
(Hodgson). 150 pp.; illus. $0.50; mail $0.60.* MECHANICAL ENGINEERING.
STRUCTURAL DRAFTING (Dufour). Draft Steam; Electrical; Gas; Automo
ing room equipment and practice; Detailing
General Instructions; Detaiſing Methods. 119
biles; Elevators.
pp.; 88 illus. Cl., $1.00; mail $1.15.” FNGINEERING, PRACTICAL–A N s w E R s
STRUCTURAL
(Scott).
DESIGNERS’ HANDBOOK
It aims to shorten computations, etc.
2N,
earn and electrical.
mall $1.15.”
250
ques.
.; 3.
90 pp.; i
and ans. on
illus.- - $1.00;-
Cl.,
51 diagrams; 35 tables. $2.00; mail
#"isº"; -- ENGINEERs' EXAMINATIONS (Hawkins).
Covers Steam Boilers, Engines, Pumps, Elec
ENGINEERING work
IN PUBLIC BUILD trical and Refrigerating Machines in ques. and

ſº
INGS (Allsop). Power, Lighting, Heating, Ven ans. form. $2.00; malſ $2.15.”
tilation, Water Supply. 158 pp.; 75 illus. Cl., RQTLERS, STEAM: THEIR construction
3.50; mail $3.65.”
HANDBOOK FOR SUPTS. OF CONSTRUC
cáñé ÁNúLea.,
194 illus.
CEEATSN'ſsº”:'''.
$1.50;
mail #1.65.
TION, ARCHITECTS, BUILDERS AND BUILD
ING INSPECTORS. By H. G. Richey, Supt. of
Construction, U. S. Public Bldgs.
figures. Morocco, $4.00; mail $4.20.*
74 pp.; 357
ENGINEERS' ARITH MET1c (Colvin-Chen
Pocketbook for Steam
$0.5u; mail 30.60.*
112 i. º: -

EXAMINATIONS, GUIDE TO
CONCRETE INSPECTION (Hill). 186 pp. Cl.,
#Fºrklºh
(Taylor).
With specimen ques, and ans.T IIs
$1.00; mail $1.10.* Pp. ; illus. Cl., $0.15; mail $0.85.”
CEMENT WORKERS' HAND Book (Baker). AUTOMOBILE GUIDE (Audel). Ques. and
Ans. for . Owners, Operators and
To meet the requirements of the common work 100 pp.; 550 illus. $1.00; mail $1.65.Machinists.
man. Cl., $0.50; mail $0.60."
AUTOMOBILE CATECH ISM AND REPAIR
PLASTERS AND PLASTERING, MORTARS MANUAL (Swingle). 165 pp.; illus. Lea., $1.29;
AND CEMENTS: HOVW TO MAKE AND How mail $1.35."
TO USE (Hodgson). Cl., $0.50; mail $0.60.* MOTORCYCLE MANUAL (Sauer). 85 pp.;

...
MUNICI PAL ENGINEERING AND SANITA illus. Pa., $0.25; mail $0.30.*
TION (Baker). 317 pp. $1.25; mail $1.35.” GAS ENGINES, PRACTICAL HAND Book
9N, CARE AND MANAGEMENT of (Lieck
PUBLIC WORKS, UNITED STATES (Black). Non-technical. 103 pp.; illus. Cl., $0.50;

...
Summary of Methods of Construction and feld).
mail $0.60.*
Character of Materials and Plant used in U.
S. Public Works of various departments. An MECHANICAL DRAUGHTING. Students'
lustrated practical guide. $0.50; mail $0.60."
il
important book of reference. 276 pp.; 56 draw
ings. Cl., $5.00; mail $5.20.* MECHANICAL. DRAWING course, con
EST MATORS' PRICE BOOK AND POc KET CISE (Gueth). The explanations form the basis
Cooper
9f

govered by the classes

at
the course
COMPANION (Hicks). For Architects and Institute,

50
Y. 120 pp.; plates. $1.00;
N.
Cl.
218 pp.; 92 tables. Cl., $1.00; mail
Hºrs.
1.10.
mail #1.10.*
MECHANICAL DRAWING (Kenison). prac

A
ESTI MATING (Nichols). tical manual self-instruction. 176 pp.; 120
of

Includes a detailed
estimate of a residence. 128 pp.; 22 illus. Cl., illus. Cl. $1.00; mail $1.15.*
$1.00; mail $1.15.” MACHINE SHOP PRIMER (Colvin). 148 pp.;
508 illus. $1.00; mail $1.10.*
cost KEEPING, Fo R contRAcTors
alker). 273 pp.; 159 figures. Cl., $2.00; mall MACHINE SHOP CATECH ISM. Over 1,000
2.15.”
{} ques. and ans. 221 pp.; 138 illus. $2.00; mail
$2.15."
writing, ELEMENTS QF
SPECIFICATION engineers. MECHANIC's own Book (Spon). 702 pp.;
(IKirby). For civil 125 pp. Cl., $1.25; 1420 illus. Cl., $2.50; mail $2.65.”
mail #1.35.” (Jallings). Hand-power- belt
ELEVATORS
contRACTS AND S P E C I F I C A T I O N S power, Steam, hydraulic, electric. 217 pp.; 17.2
(Nichols). 110 pp.; illus. Cl., $1.00; mail $1.15.” illus. Cl., $1.50; mail $1.65.”
HIGHVVAY ENGINEERS' HANDBOOK. By MARINE ENGINEERING (Hawkins). About
W. G. Harger, C.E., and E. A. Bonney, Con 800 practical ques. and ans. new edition. $3.00;
;

sulting Engr., N. Y. State Highway Dept. 609 mail $3.15.”


pp., illus. Lea., $3.00; mail $3.15.” NAVAL MACHINERY, HANDBOOK FoR
THE CARE AND OPERATION OF (Dinger).
SEw ERS AND DRAINS (Marston). , Sys 312 pp.; illus. 2.00; mail $2.15.”
tems; calculation, and design; drainage and sani
tation; construction and maintenance; disposal.
208 pp.; 73 illus. Cl., $1.00; mail $1.15.”
PARKs AND PARK ENGINEERING (Lyle).
TRADES AND VOCATIONS.
130 pp.; 38 plates. Cl., $1.25; mail $1.40.*
MACHINE SHOP MANAGEMENT, HAND
Dock AND HARBOR ENGINEERS’ REF BOOK OF (Van Deventer). 374 pp.; 245 illus.
$2.50;mail $2.65.”
ERENCE BOOK (Brysson). 319 pp.; 144 figs. CAR PENTRY practical
$3.00; mail $3.15." (Townsend). A.
treatise on simple building construction. In
SURVEYING (Finch). Elementary discussion cludes Timber its natural state, and Car
in

of principles, instruments, methods of making penters' Tools and Building Problems. 258 pp.;
land, topographic, hydrographic drafting. 75 pp.; 368 illus. Cl., $1.50; mail $1.65.”
150 illus. Cl., $1.50; mail $1.65.” CAR PENTRY, BEGINNERS’ GUIDE To
99

(Jarvis). 128 pp.; illus. Boards, $0.50; mail


SURVEYING, PLANE (Tracy). 792 pp.; illus. $0.60.*
Morocco, $3.00; mail $3.20.* BOOKBINDING, PRACTICAL (Pearce). 132
N A V A L
ARCHITECTS’ AND SHIP pp.; Boards, $0.50; mail $0.60.*
92 illus.
BUILDERS". Poç KEI. Book, OF. FoEMULAE;
RULES AND TABLES, and Marine Engineers
BASKETRY, PRACTICAL (Gill). Instruction
from the simplest form to the most elaborate.
and Surveyors' Handy Book of Reference (Mack Illus. $1.00; mail $1.10.*
row ). pp.
742 Lea., $5.00; mail $5.20.*
constructor, BASKET work (Hasluck). 189 illus. $0.50;
K

NAVAL THE (Simpson). A mail $0.60."


vade mecum of Ship Design for Students, Naval

-
Architects. Shipbuilders and Owners, Marine DRESSMA KING, MANUAL OF. By Janes
Supts., Engineers and Draughtsmen. 816 pp.; Fales, Prof. Household Arts, Teachers' College,
36, illus. Lea., $5.00; mail $5.20.* Columbia Univ., N. Y. $1.25; mail $1.38.*

108
CIVIL SERVICE BOOKS.

TELEGRAPHY FOR BEGINNERS (Jones). ports, and official papers. 402 Pp., with tables
Authoritative instruction on standard methods. and forms. $5.00; mail $5.20.
58 pp.; 19 illus. Cl., $0.50; mail $0.60.*
MUNICIPAL. UTILITIES, REGULATION OF
WIRELESS LICENSE EXAMINATIONS, U. Prepared from material of Nat. Mu

§:
(King).
S.: HOVW TO PASS (Bucher). 118 actual govt. nicipal League. Cl., $2.00; mail $2.15.”
ques. answered. 72 pp.; illus. $0.50; mail
RAILROAD RATES, AMERICAN (Noyes).
FLUMBING. (See under “Sanitary and
Fundamental legal principles which must con
regulation. 277 pp. $1.50; mail
Medical.”) ºvernment

...
too.
RAILROAD EXPENSEs, HANDBOOK

F
O
559 pp. Lea., $3.00; mail $3.15.”
GOVERNMENT. (Eaton).
GOVERNMENT: NEW YORK CITY NEW
YORK STATE AND FEDERAL. Specially pre AGRICULTURE AND FORESTRY.
for Civil Service Examinations. Includes
pºeg
00 Ques. and Ans. 45,000 words. Pa., $0.50; AGRICULTURE, CATECHISM OF (Atkeson).
mail $0.55. 96 pp.; Cl.,
illus. $0.50; mail $0.60.*
CIVICS OF NEW JERSEY (Knowlton). 368 AGRICULTURE THROUGH THE LABORA
Cl., $1.00; mail $1.15.”

-
pp. TORY AND SCHOOL GARDEN (Jackson). Ele
ACTUAL GOve RNMENT OF NEW YORK mentary textbook for schools. 450 pp.; illus.
Cl., $1.50; mail $1.65.”
(Boynton). A manual of local, municipal, State
and Federal government for use in schools. For ESTRY, ELEMENTs of. By Moon,

F.
F.
Covers the work outlined by N. Y. State Educa Prof. of Forest Engrg., N. Y. State College of
JPept. 368 pages (illus.). Cl., $1; mail Forestry. $2.00; mail $2.15.”
##
THE AMERICAN GOVERNMENT (Haskin). FOREST VALUATION (Chapman). 310 pp.
every department
Cl., $2.00; mail $2.15.”
Covers of the U. S. Govt.
Endorsed by high government officials. 398 pages FEDERAL FARM Loa SYSTEM (Myrick).

N
(illus.). Cl., $1; mail $1.15.* 240 pp. Cl., $1.00; mail $1.13.”
UNITED STATES DEPT. OF STATE (Hunt). WEATHER FORECASTING, PRACTICAL
Formation and development of the Department METHODS OF. By E. B. Dunn, formerly Govt.
§s s chief duties. 468 pp. Cl., $2.25; mail
Forecaster in New York. Presented in popular,
untechnical form. Cl., $1.75; mail $1.90.*
CITY PLANNING (Nolen). Cl., $2.00; mail
$2.15.”
CITY PLAN, CARRYING OUT THE (Shurt MISCELLANEOUS.
leff and Olmsted). 349 pp. $2.00; mail $2.15.”
PLAN NING OF THE MODERN CITY. By NAVIGATORS' Pocket BOOK. By Capt.
Howard Patterson. Arranged for immediate
Nelson P. Lewis, Chief Engr. Bó. of Flstimate. reference to any navigation subject. complete
§§ Cl., $3.50; mail

A
of N. Y. 439
pp.; illus. Lea., $2.00; mail $2.13.”
guide and instructor.

CITY GOVERNMENT BY COMMISSION


SHIPMASTERS, PRACTICAL DUTIES OF

§
Description of the System, argu (Wilkes-Harry). 119 pp.; illus. Cl., $1.25; mail
(Woodruff).
ments for and against, accounts of actual opera $1.40.*
charters, etc. Illus. $2.00; mail GAME PROTECTION AND PROPAGATION
.results.
AMERICA (Chase). handbook of prac
A
N
|

Gover NMENT, THE NEW CITY. By Henry tical information for officials and others. 238
Bruere, Director Bur. Municipal Research, pp. Cl., $1.25; mail $1.40."
N.
Y. City. Based on researches in 10 cities hav ANIMAL USBAN DRY, BEGINNINGS IN
H

Ing Commission government. Cl., $2.00; mail (Plumb). 393 pp.; illus. $1.25; mail $1.35."
$2.15.“
REAL ESTATE METHODS, PRACTICAL. By
CITY MANAGER, THE (Toulmin). Com covering all branches. $2.50; mail
plete summary of this form of government as ºperts.
2.70.*
§

adopted in many cities. Also deals with ad


vantages and disadvantages of Commission gov BUSINESS MANAGEMENT (Russell). Billing
ernment. $2.00; mail $2.15.” and Order Recording: Shipping Dept.; Corre
spondence Filing; Business Statistics. 240 pp.;
122 illus. Cl., $1.50; mail $1.65.”
PUBLIC UTILITIES. EFFICIENCY METHODS,
N

T
A

G
N
L
L
S
I

(Knoeppel). The exact description of practice.


(See also under “Bookkeeping.”) 260 pp. $3.00; mail $3.15.”
MUNICIPAL FRANCHISEs. By Delos F.
Wilcox, ex-Chief Bur of Franchises, Pub. Serv.
Com., N. Y. Vol. I: Pipe and Wire Fran
chises: 710 pp.: $5.00; mail $5.25.” Vol. II: CIVIL SERVICE CHRONICLE.
Transportation Franchises, Taxation and Con The National Journal of the Civil Service.
trol of Public Utilities; 885 pp.; $5.00; mail Published weekly at 23 Duane Street, New York.
$5.25.” Every issue contains Free Instruction for many
PUBLIC SERVICE CORPORATIONS, VAL popular Civil Service FXaminations, with list
ina

pending and anticipated examinations, ad


of

UATION OF (Whitten). Complete legal and


treatment, with authorities and pre dition generally covering the field of news
to

economic
of interest Civil Service employes and candi
$2 to

cedents. Cl., $5.50; mail $5.70.*


dates for City, State and Federal Government
PUBLIC UTILITY PROPERTIES, VALUA positions. year: $1.15 for months;
60
6
a

TION OF (Floy). Unit price, definite figures cents for months; cents copy. Sample
3

and facts, with court decisions, records, re copy free.

-
New York
23 Duane Street -
-
|

You might also like